Brunzel Ans Key

You might also like

Download as pdf or txt
Download as pdf or txt
You are on page 1of 88

CHAPTER 1 Quality Assessment and Safety 15

6. Clinical and Laboratory Standards Institute (CLSI): Training 15. Occupational Safety and Health Administration: Directives:
and competence assessment: approved guideline, ed 3, CLSI enforcement procedures for the occupational exposure to
Document QMS03-A3, CLSI, Wayne, PA, 2009. bloodborne pathogens, CPL 2-2.69D, US Department of Labor,
7. College of American Pathologists: Clinical Microscopy Survey November 27, 2001.
Kit Instructions, CMP 2016. College of American Pathologists, 16. Garner JS: Guideline for isolation precautions in hospitals. Infect
Northfield, IL, 2016. Control Hosp Epidemiol 17:53–80, 1996.
8. Clinical and Laboratory Standards Institute (CLSI): Protection 17. Centers for Disease Control and Prevention: 2007 guideline
of laboratory workers from occupationally acquired infections: for isolation precautions: preventing transmission of
approved guideline, ed 4, CLSI Document M29-A4, CLSI, infectious agents in healthcare settings (website): www.cdc.gov/
Wayne, PA, 2014. hicpac/2007IP/2007isolationPrecautions.html. Accessed
9. Clinical and Laboratory Standards Institute (CLSI): Clinical July 7, 2011.
laboratory safety: approved guideline, ed 3, CLSI Document 18. Occupational Safety and Health Administration: NFPA OSHA
GP17-A3, CLSI, Wayne, PA, 2012. label comparison quick card (website): https://www.osha.gov/
10. Clinical and Laboratory Standards Institute (CLSI): Clinical dsg/hazcom/. Accessed February 18, 2016.
laboratory waste management: approved guideline, ed 3, CLSI
Document GP05-A3, CLSI, Wayne, PA, 2011. BIBLIOGRAPHY
11. Lynch P, Jackson M, Cummings M, Stamm W: Rethinking the
role of isolation precautions in the prevention of nosocomial Clinical and Laboratory Standards Institute (CLSI): Urinalysis:
infections. Ann Intern Med 107:243–246, 1987. approved guideline, ed 3, CLSI Document GP16-A3, CLSI,
12. Lynch P, Cummings M, Roberts P, et al: Implementing Wayne, PA, 2009.
and evaluating a system of generic infection precautions: Hazardous materials, storage, and handling pocketbook, Alexandria,
body substance isolation. Am J Infect Control 18:1–12, 1987. VA, 1984, Defense Logistics Agency.
13. Occupational Safety and Health Administration: Occupational National Fire Protection Association: Hazardous chemical data,
exposure to bloodborne pathogens; final rule. Federal Register Boston, 1975, National Fire Protection Association, No. 49.
56:64003–640182 (codified as 29 CFR 1910.1030), December 6, Occupational exposure to hazardous chemicals in laboratories, final
1991. rule, Federal Register 55:3327–3335, 1990.
14. Occupational Safety and Health Administration: Directives: Schweitzer SC, Schumann JL, Schumann GB: Quality assurance
enforcement procedures for the occupational exposure to guidelines for the urinalysis laboratory. J Med Technol 3:570, 1986.
bloodborne pathogens, CPL 2-2.44D, US Department of Labor,
November 5, 1999.

STUDY QUESTIONS
1. The ultimate goal of a quality assessment program 5. The purpose of quality control materials is to
is to A. monitor instrumentation to eliminate downtime.
A. maximize the productivity of the laboratory. B. ensure the quality of test results obtained.
B. ensure that patient test results are precise. C. assess the accuracy and precision of a method.
C. ensure appropriate diagnosis and treatment of D. monitor the technical competence of laboratory staff.
patients. 6. Why are written procedures necessary?
D. ensure the validity of laboratory results obtained. A. To assist in the ordering of reagents and supplies for a
2. Which of the following is a preanalytical component of a procedure
QA program? B. To appropriately monitor the accuracy and precision
A. Quality control of a procedure
B. Turnaround time C. To ensure that all individuals perform the same task
C. Technical competence consistently
D. Preventive maintenance D. To ensure that the appropriate test has been ordered
3. Which of the following is a postanalytical component of a 7. Which of the following is not considered to be an analyt-
QA program? ical component of QA?
A. Critical values A. Reagents (e.g., water)
B. Procedures B. Glassware (e.g., pipettes)
C. Preventive maintenance C. Instrumentation (e.g., microscope)
D. Test utilization D. Specimen preservation (e.g., refrigeration)
4. Analytical components of a QA program are procedures 8. Which of the following sources should include a protocol
and policies that affect the for the way to proceed when quality control results
A. technical testing of the specimen. exceed acceptable tolerance limits?
B. collection and processing of the specimen. A. A reference book
C. reporting and interpretation of results. B. A procedure
D. diagnosis and treatment of the patient. C. A preventive maintenance manual
D. A specimen-processing protocol
CHAPTER 1 Quality Assessment and Safety 15

6. Clinical and Laboratory Standards Institute (CLSI): Training 15. Occupational Safety and Health Administration: Directives:
and competence assessment: approved guideline, ed 3, CLSI enforcement procedures for the occupational exposure to
Document QMS03-A3, CLSI, Wayne, PA, 2009. bloodborne pathogens, CPL 2-2.69D, US Department of Labor,
7. College of American Pathologists: Clinical Microscopy Survey November 27, 2001.
Kit Instructions, CMP 2016. College of American Pathologists, 16. Garner JS: Guideline for isolation precautions in hospitals. Infect
Northfield, IL, 2016. Control Hosp Epidemiol 17:53–80, 1996.
8. Clinical and Laboratory Standards Institute (CLSI): Protection 17. Centers for Disease Control and Prevention: 2007 guideline
of laboratory workers from occupationally acquired infections: for isolation precautions: preventing transmission of
approved guideline, ed 4, CLSI Document M29-A4, CLSI, infectious agents in healthcare settings (website): www.cdc.gov/
Wayne, PA, 2014. hicpac/2007IP/2007isolationPrecautions.html. Accessed
9. Clinical and Laboratory Standards Institute (CLSI): Clinical July 7, 2011.
laboratory safety: approved guideline, ed 3, CLSI Document 18. Occupational Safety and Health Administration: NFPA OSHA
GP17-A3, CLSI, Wayne, PA, 2012. label comparison quick card (website): https://www.osha.gov/
10. Clinical and Laboratory Standards Institute (CLSI): Clinical dsg/hazcom/. Accessed February 18, 2016.
laboratory waste management: approved guideline, ed 3, CLSI
Document GP05-A3, CLSI, Wayne, PA, 2011. BIBLIOGRAPHY
11. Lynch P, Jackson M, Cummings M, Stamm W: Rethinking the
role of isolation precautions in the prevention of nosocomial Clinical and Laboratory Standards Institute (CLSI): Urinalysis:
infections. Ann Intern Med 107:243–246, 1987. approved guideline, ed 3, CLSI Document GP16-A3, CLSI,
12. Lynch P, Cummings M, Roberts P, et al: Implementing Wayne, PA, 2009.
and evaluating a system of generic infection precautions: Hazardous materials, storage, and handling pocketbook, Alexandria,
body substance isolation. Am J Infect Control 18:1–12, 1987. VA, 1984, Defense Logistics Agency.
13. Occupational Safety and Health Administration: Occupational National Fire Protection Association: Hazardous chemical data,
exposure to bloodborne pathogens; final rule. Federal Register Boston, 1975, National Fire Protection Association, No. 49.
56:64003–640182 (codified as 29 CFR 1910.1030), December 6, Occupational exposure to hazardous chemicals in laboratories, final
1991. rule, Federal Register 55:3327–3335, 1990.
14. Occupational Safety and Health Administration: Directives: Schweitzer SC, Schumann JL, Schumann GB: Quality assurance
enforcement procedures for the occupational exposure to guidelines for the urinalysis laboratory. J Med Technol 3:570, 1986.
bloodborne pathogens, CPL 2-2.44D, US Department of Labor,
November 5, 1999.

STUDY QUESTIONS
1. The ultimate goal of a quality assessment program 5. The purpose of quality control materials is to
is to A. monitor instrumentation to eliminate downtime.
A. maximize the productivity of the laboratory. B. ensure the quality of test results obtained.
B. ensure that patient test results are precise. C. assess the accuracy and precision of a method.
C. ensure appropriate diagnosis and treatment of D. monitor the technical competence of laboratory staff.
patients. 6. Why are written procedures necessary?
D. ensure the validity of laboratory results obtained. A. To assist in the ordering of reagents and supplies for a
2. Which of the following is a preanalytical component of a procedure
QA program? B. To appropriately monitor the accuracy and precision
A. Quality control of a procedure
B. Turnaround time C. To ensure that all individuals perform the same task
C. Technical competence consistently
D. Preventive maintenance D. To ensure that the appropriate test has been ordered
3. Which of the following is a postanalytical component of a 7. Which of the following is not considered to be an analyt-
QA program? ical component of QA?
A. Critical values A. Reagents (e.g., water)
B. Procedures B. Glassware (e.g., pipettes)
C. Preventive maintenance C. Instrumentation (e.g., microscope)
D. Test utilization D. Specimen preservation (e.g., refrigeration)
4. Analytical components of a QA program are procedures 8. Which of the following sources should include a protocol
and policies that affect the for the way to proceed when quality control results
A. technical testing of the specimen. exceed acceptable tolerance limits?
B. collection and processing of the specimen. A. A reference book
C. reporting and interpretation of results. B. A procedure
D. diagnosis and treatment of the patient. C. A preventive maintenance manual
D. A specimen-processing protocol
16 CHAPTER 1 Quality Assessment and Safety

9. Technical competence is displayed when a laboratory 15. Match the mode of transmission with the laboratory activity.
practitioner
A. documents reports in a legible manner. Mode of
B. recognizes discrepant test results. Laboratory Activity Transmission
C. independently reduces the time needed to perform a 3 A. Not wearing gloves when
__ 1. Inhalation
procedure (e.g., by decreasing incubation times). handling specimens 2. Ingestion
I
D. is punctual and timely. __ B. Centrifuging uncovered 3. Direct contact
10. Quality control materials should have specimens
2 C. Smoking in the laboratory
__
A. a short expiration date.
3 D. Being scratched by a
__
B. a matrix similar to patient samples.
broken beaker
C. their values assigned by an external and unbiased
__
3 E. Having a specimen
commercial manufacturer. splashed into the eyes
D. the ability to test preanalytical variables. 2 F. Pipetting by mouth
__
11. Within one facility, what is the purpose of performing
duplicate testing of a specimen by two different laborato- 16. Which of the following is not considered personal protec-
ries (i.e., in-house duplicates)? tive equipment?
A. It provides little information because the results are A. Gloves
already known. B. Lab coat
B. It saves money by avoiding the need for internal qual- C. Disinfectants
ity control materials. D. Goggles
C. It provides a means of evaluating the precision of 17. Which of the following actions represents a good labora-
a method. tory practice?
D. It can detect procedural and technical differences A. Washing or sanitizing hands frequently
among laboratories. B. Wearing lab coats outside the laboratory
12. Interlaboratory comparison testing as with proficiency C. Removing lab coats from the laboratory for launder-
surveys provides a means to ing at home in 2% bleach
A. identify critical values for timely reporting to D. Wearing the same gloves to perform venipuncture on
clinicians. two different patients because the patients are in the
B. ensure that appropriate documentation is being same room
performed. 18. Which of the following is not an acceptable disposal practice?
C. evaluate the technical performance of individual lab- A. Discarding urine into a sink
oratory practitioners. B. Disposing of used, empty urine containers with
D. evaluate the performance of a laboratory compared nonhazardous waste
with that of other laboratories. C. Discarding a used, broken specimen transfer pipette
13. The primary purpose of a Standard Precautions policy in with noninfectious glass waste
the laboratory is to D. Discarding blood specimens into a biohazard container
A. ensure a safe and healthy working environment. 19. Which of the following is not part of a Chemical Hygiene
B. identify processes (e.g., autoclaving) to be used to Plan?
neutralize infectious agents. A. To identify and label hazardous chemicals
C. prevent the exposure and transmission of potentially B. To educate employees about the chemicals they use
infectious agents to others. (e.g., providing material safety data sheets)
D. identify patients with hepatitis B virus, human immu- C. To provide guidelines for the handling and use of
nodeficiency virus, and other infectious diseases. each chemical type
14. Which agency is responsible for defining, establishing, D. To monitor the handling of biological hazards
and monitoring safety and health hazards in the 20. Which of the following information is not found on a
workplace? safety data sheet (SDS)?
A. Occupational Safety and Health Administration A. Exposure limits
B. Centers for Disease Control and Prevention B. Catalog number
C. Chemical Hygiene Agency C. Hazardous ingredients
D. National Fire Protection Association D. Flammability of the chemical
CHAPTER 1 Quality Assessment and Safety 17

Case 1.1
Both a large hospital and its outpatient clinic have a laboratory 1. Which of the following conditions present in the hospital lab-
area for the performance of routine urinalyses. Each laboratory oratory could cause the observed findings in this case?
performs daily QA checks on reagents, equipment, and proce- 1. The urinalysis centrifuge had its brake left on.
dures. Because the control material used does not have sedi- 2. The urinalysis centrifuge was set for the wrong speed or
ment components, each laboratory sends a completed time setting.
urinalysis specimen to the other laboratory for testing. After 3. Microscopic examination was performed on an unmixed
the urinalysis has been performed, results are recorded, com- or inadequately mixed specimen.
pared, and evaluated. The criterion for acceptability is that all 4. Microscopic examination was performed using nonopti-
parameters must agree within one grade. mized microscope settings for urine sediment viewing
(e.g., contrast was not sufficient to view low-refractile
Results components).
One day, all results were acceptable except those of the micro- A. 1, 2, and 3 are correct.
scopic examination, which follow: B. 1 and 3 are correct.
C. 2 and 4 are correct.
D. 4 is correct.
Hospital Laboratory Clinic Laboratory E. All are correct.
RBCs/hpf: 5–10 RBCs/hpf: 25–50 2. Which of the following actions could prevent this from hap-
WBCs/hpf: 0–2 WBCs/hpf: 0–2 pening again?
Casts/lpf: 0–2 hyaline Casts/lpf: 5–10 hyaline A. The microscope and centrifuge should be repaired.
B. The laboratory should participate in a proficiency survey.
On investigation, the results from the clinic were found to be
C. A control material with sediment components should be
correct; the hospital had a problem, which was addressed and
used daily.
remedied immediately.
D. All results should be reviewed by the urinalysis supervisor
before they are reported.

hpf, High-power field; lpf, low-power field; RBC, red blood cell; WBC, white blood cell.
2
Urine Specimen Types, Collection,
and Preservation

LEARNING OBJECTIVES
After studying this chapter, the student should be able to: • Suprapubic aspiration
1. State at least three clinical reasons for performing a routine • Pediatric collection
urinalysis. 5. Describe materials and procedures used for proper
2. Describe three types of urine specimens, and state at least collection and identification of urine specimens.
one diagnostic use for each type. 6. Identify six reasons for rejecting a urine specimen.
3. Explain the importance of accurate timing and complete 7. State the changes possible in unpreserved urine, and
collection of timed urine specimens. explain the mechanism for each.
4. Describe the collection technique used to obtain the 8. Discuss urine preservatives, including their advantages,
following specimens: disadvantages, and uses.
• Random void 9. List and justify at least three tests that assist in determining
• Midstream “clean catch” whether a fluid is urine.
• Catheterized

CHAPTER OUTLINE
Why Study Urine?, 19 Suprapubic Aspiration, 22
Specimen Types, 19 Pediatric Collections, 22
First Morning Specimen, 19 Reasons for Urine Specimen Rejection, 22
Random Urine Specimen, 20 Urine Volume Needed for Testing, 22
Timed Collection, 20 Urine Specimen Storage and Handling, 22
Collection Techniques, 21 Containers, 22
Routine Void, 21 Labeling, 23
Midstream “Clean Catch,” 21 Handling and Preservation, 23
Catheterized Specimen, 21 Is This Fluid Urine?, 25

K E Y T E R M S*
catheterized specimen suprapubic aspiration
first morning specimen timed collection
midstream “clean catch” specimen urine preservative
random urine specimen

*Definitions are provided in the chapter and glossary.

The purposes of performing a routine urinalysis are (1) to aid in changed because of improper storage conditions, testing will
the diagnosis of disease; (2) to screen for asymptomatic, congen- produce results that do not reflect the patient’s condition. In
ital, or hereditary disease; (3) to monitor disease progression; such situations, the highest quality reagents, equipment, exper-
and (4) to monitor therapy effectiveness or complications.1 tise, and personnel cannot compensate for the unacceptable
To obtain accurate urinalysis results, urine specimen integrity specimen. Therefore written criteria for urine specimen types,
must be maintained. If the urine specimen submitted for testing instructions for proper collection and preservation, appropriate
is inappropriate (e.g., if a random specimen is submitted instead specimen labeling, and a handling timeline must be available to
of a timed collection) or if the specimen composition has all personnel involved in urine specimen procurement.

18
CHAPTER 2 Urine Specimen Types, Collection, and Preservation 25

and (3) time delays and transportation conditions (e.g., the laboratory of two specimens from the same patient in identi-
sample is sent to a reference laboratory). Some laboratories cal sterile containers for testing, but the fluid source is not
may perform an assay daily in-house and require only refrig- clearly evident on either container. In these varied situations,
eration of the sample during the timed collection. In contrast, a a few simple and easily performed tests can aid in determining
small laboratory may send the assay to a reference facility that whether the fluid is actually urine.
requires that a chemical preservative be used during the collec- The single most useful substance that identifies a fluid
tion to ensure analyte stability. Each urinalysis laboratory must as urine is its uniquely high creatinine concentration (approx-
have in its procedure manual a protocol for the collection of all imately 50 times that of plasma). Similarly, concentrations of
timed urine specimens. The protocol should include the name urea, sodium, and chloride are significantly higher in urine
of the analyte; a description of the appropriate specimen col- than in other body fluids. Note that in urine from healthy
lection technique; the appropriate preservative required; label- individuals, no protein or glucose is usually present. In con-
ing requirements, including precautions for certain chemical trast, other body fluids such as amniotic fluid or plasma
preservatives; the location at which the test is performed; ref- exudates contain glucose and are high in protein.
erence ranges; and the expected turnaround time.
Timed urine collections should be transported to the labora-
tory as soon as possible after completion of the collection. The REFERENCES
total volume is determined, the specimen is well mixed to ensure 1. Clinical and Laboratory Standards Institute (CLSI): Urinalysis:
homogeneity, and aliquots are removed for the appropriate approved guideline, ed 3, CLSI Document GP16-A3, CLSI,
tests. At no point during a timed collection can urine be removed Wayne, PA, 2009.
or discarded, even if the volume is recorded. This would inval- 2. Schumann GB, Schumann JL: A manual of cytodiagnostic
idate the collection because the concentration of the analyte in urinalysis, Salt Lake City, 1984, Cytodiagnostics Company.
any removed aliquot cannot be determined and corrected for. 3. Roberts SB, Lucas A: Measurement of urinary constituents and
output using disposable napkins. Arch Dis Child 60:1021–1024,
1985.
IS THIS FLUID URINE? 4. Schumann GB, Friedman SK: Specimen collection, preservation
and transportation. In Wet urinalysis, Chicago, 2003, ASCP
At times it is necessary to verify that the fluid present in a urine Press.
container is in fact urine. This may occur in laboratories that 5. Culhane JK: Delayed analysis of urine. J Fam Pract 30:473–474,
perform urine testing for illicit drugs (e.g., amphetamine, 1990.
cocaine, tetrahydrocannabinol [THC], steroids). In these situ- 6. Meers PD, Chow CK: Bacteriostatic and bactericidal actions of
ations, particularly when the urine collection is not witnessed, boric acid against bacteria and fungi commonly found in urine.
the individual may have the opportunity to add a substance to J Clin Pathol 43:484–487, 1990.
the urine collection (e.g., an adulterated specimen). Another 7. Kouri T, Vuotari L, Pohjavaara S, Laippala P: Preservation of
possibility is that the liquid in the container is not urine. urine for flow cytometric and visual microscopic testing. Clin
Specific gravity, pH, and temperature can be helpful in iden- Chem 48:900–905, 2002.
8. Weinstein MP: Clinical evaluation of a urine transport kit with
tifying urine specimens to which additional liquid has been
lyophilized preservative for culture, urinalysis, and sediment
added. The physiologically possible range for urine pH in a microscopy. Diagn Microbiol Infect Dis 3:501–508, 1985.
fresh urine specimen is 4.0 to 8.0 and for specific gravity is 9. Becton Dickinson VACUTAINER Systems: Understanding
1.002 to 1.035. In a normal healthy individual, the temperature additives: urine preservatives. In Lab notes—a newsletter from
of a urine specimen immediately after collection is usually Becton Dickinson VACUTAINER Systems, Vol 4 No 2,
between 32.5 °C and 37.5 °C. If this range is exceeded and Franklin Lakes, NJ, 1993, Becton Dickinson VACUTAINER
the temperature is lower or higher, the urine has been altered Systems.
in some way, or the fluid is not urine. Note that urine specific
gravity can exceed 1.035 if the patient has had a recent infusion
of radiographic contrast media (x-ray dye).
BIBLIOGRAPHY
Occasionally, when an amniocentesis is performed, con- McPherson RA, Ben-Ezra J: Basic examination of urine.
cern may be raised regarding whether the fluid collected is In McPherson RA, Pincus MR, editors: Clinical diagnosis and
amniotic fluid or urine aspirated from the bladder. Another management by laboratory methods, ed 22, Philadelphia, 2011,
circumstance that may be encountered is receipt in the Saunders Elsevier.

STUDY QUESTIONS
1. Which of the following is the urine specimen of choice for 2. Which of the following specimens usually eliminates
cytology studies? contamination of the urine with entities from the external
A. First morning specimen genitalia and the distal urethra?
B. Random specimen A. First morning specimen
C. Midstream “clean catch” collection B. Midstream “clean catch” specimen
D. Timed collection C. Random specimen
D. 4-hour timed collection
26 CHAPTER 2 Urine Specimen Types, Collection, and Preservation

3. Substances that show diurnal variation in their urinary 8. A urine specimen containing the substance indicated is
excretion pattern are best evaluated using a kept unpreserved at room temperature for 4 hours.
A. first morning specimen. Identify the probable change to that substance.
B. midstream “clean catch” specimen.
C. random specimen. Substance Change
D. timed collection. __
C Bacteria A. Decrease
4. Which of the following will not cause erroneous results in __
A Bilirubin B. No change
a 24-hour timed urine collection? A Glucose
__ C. Increase
A. The collection starts and ends in the evening. __
A Ketones

B. Two first morning specimens are included in the %


__ pH
__ Protein
collection. A
__ Urobilinogen
C. Multiple collection containers are not mixed together
before specimen testing. 9. Which of the following is the most common method used
D. A portion of the collection is removed before total to preserve urine specimens?
volume measurement. A. Acid addition
5. A 25-year-old woman complains of painful urination and B. Thymol addition
is suspected of having a urinary tract infection. Which of C. Freezing
the following specimens should be collected for a routine D. Refrigeration
urinalysis and urine culture? 10. If refrigeration is used to preserve a urine specimen,
A. First morning specimen which of the following may occur?
B. Timed collection A. Cellular or bacterial glycolysis will be enhanced.
C. Midstream “clean catch” specimen B. Formed elements will be destroyed.
D. Random specimen C. Amorphous crystals may precipitate.
6. A 35-year-old diabetic woman is suspected of developing D. Bacteria will proliferate.
renal insufficiency. Which of the following specimens 11. Which of the following urine preservatives is acceptable
should be obtained to determine the amount of creatinine for both urinalysis and urine culture?
being excreted in the urine? A. Boric acid
A. 2-hour postprandial B. Chlorhexidine
B. 12-hour timed collection C. Dowicil 200
C. 24-hour timed collection D. Formalin
D. Midstream “clean catch” 12. How much urine is usually required for a manually per-
7. An unpreserved urine specimen collected at midnight is formed routine urinalysis?
kept at room temperature until the morning hospital A. 5 to 10 mL
shift. Which of the following changes will most likely B. 10 to 15 mL
occur? C. 20 to 30 mL
A. Decrease in urine color and clarity D. 50 to 100 mL
B. Decrease in pH and specific gravity 13. Which of the following substances is higher in urine than
C. Decrease in glucose and ketones in any other body fluid?
D. Decrease in bacteria and nitrite A. Chloride
B. Creatinine
C. Glucose
D. Protein
3
The Kidney

LEARNING OBJECTIVES
After studying this chapter, the student should be able to: 6. Describe the three secretory mechanisms that the kidney
1. Identify and state the primary functions of the uses to regulate the acid-base equilibrium of the body.
macroscopic structures of the kidney and urinary tract. 7. Explain tubular transport capacity (Tm), and discuss its
2. Diagram the structure and state the function of each relationship to renal threshold.
portion of the nephron. 8. Compare and contrast the countercurrent multiplier
3. Describe renal blood circulation and its role in renal mechanism, the countercurrent exchange mechanism,
function. and the urea cycle, and their roles in urine formation and
4. Discuss the components and the process of glomerular concentration.
filtration and urine formation, including the anatomic 9. Briefly summarize the relationship of water reabsorption
structures, the filtration forces, and the substances to antidiuretic hormone and the relationship of sodium
involved. reabsorption to renin and aldosterone.
5. Describe the transport mechanisms of tubular reabsorption
and tubular secretion, including the substances involved.

CHAPTER OUTLINE
Renal Anatomy, 28 Glomerulus, 32
Renal Circulation, 30 Tubules, 35
Renal Physiology, 32 Tubular Function, 37
Urine Formation, 32

K E Y T E R M S*
active transport maximal tubular secretory capacity
afferent arteriole mesangium
antidiuretic hormone nephron
basement membrane osmosis
collecting duct passive transport
countercurrent exchange mechanism peritubular capillaries
countercurrent multiplier mechanism podocytes
distal convoluted tubule proximal tubule
efferent arteriole renal threshold level
glomerular filtration barrier renin
glomerulus (also called renal corpuscle) shield of negativity
isosmotic titratable acids
juxtaglomerular apparatus tubular reabsorption
kidneys tubular secretion
loop of Henle urea cycle
maximal tubular reabsorptive capacity vasa recta

*Definitions are provided in the chapter and glossary.

27
CHAPTER 3 The Kidney 47

STUDY QUESTIONS
1. Beginning with the glomerulus, number the following 7. Which of the following is a characteristic of renin, an
structures in the order in which the ultrafiltrate travels enzyme secreted by specialized cells of the juxtaglomer-
for processing and excretion in the kidney. ular apparatus?
10
__ A. Bladder A. Renin stimulates the diffusion of urea into the renal
__
7- B. Calyces interstitium.
__
6 C. Collecting tubule B. Renin inhibits the reabsorption of sodium and water
__
5 D. Distal tubule

__I E. Glomerulus
GPLJ / in the nephron.
C. Renin regulates the osmotic reabsorption of water by
__
4 F. Juxtaglomerular apparatus the collecting tubules.
__
3

__
G. Loop of Henle
2 H. Proximal tubule O
D. Renin causes the formation of angiotensin and the
secretion of aldosterone.
__
8 I. Renal pelvis 8. The glomerular filtration barrier is composed of the
__
9 J. Ureter A. capillary endothelium, basement membrane, and
__ CE, BNN , Pooh
11 K. Urethra
podocytes.
2. How many nephrons are found in the average kidney? B. mesangium, basement membrane, and shield of
A. 13,000 negativity.
B. 130,000 C. capillary endothelium, mesangium, and juxtaglomer-
C. 1.3 million - ular apparatus.
D. 13 million D. basement membrane, podocytes, and juxtaglomeru-
3. Ultrafiltration of plasma occurs in glomeruli located in lar apparatus.
the renal 9. The ability of a solute to cross the glomerular filtration
A. cortex. barrier is determined by its

I
B. medulla. 1. molecular size.
C. pelvis. 2. molecular radius.
D. ureter. 3. electrical charge.
0
4. Which component of the nephron is located exclusively
in the renal medulla?
4. plasma concentration.
A. 1, 2, and 3 are correct.
A. Collecting tubule B. 1 and 3 are correct.
B. Distal tubule C. 4 is correct.

-0
C. Loop of Henle D. All are correct.
D. Proximal tubule 10. The epithelium characterized by a brush border owing to
5. Which of the following is not a vascular characteristic of numerous microvilli is found in the
the kidney? A. collecting tubules.
A. The afferent arteriole has a narrower lumen than the B. distal tubules.
efferent arteriole. C. loops of Henle.
B. The arteries are primarily end arteries, supplying D. proximal tubules.
specific areas of tissue, and they do not interconnect. 11. The kidneys play an important role in the

E
C. The arterioles subdivide into a capillary network, 1. excretion of waste products.
rejoin as an arteriole, and subdivide into a second 2. regulation of water and electrolytes.
capillary bed. 3. maintenance of acid-base equilibrium.
D. The vasa recta vessels deep in the renal medulla form 4. control of blood pressure and fluid balance.
the beginning of the venous renal circulation. A. 1, 2, and 3 are correct.
O
6. Formation of the ultrafiltrate in the glomerulus is driven
by the
B. 1 and 3 are correct.
C. 4 is correct.
A. hydrostatic blood pressure. OD. All are correct.
B. oncotic pressure of the plasma proteins.
C. osmotic pressure of the solutes in the ultrafiltrate.
O
12. What percent of the original ultrafiltrate formed in the
urinary space actually is excreted as urine?
D. pressures exerted by the glomerular filtration barrier. O A. 1%
B. 10%
C. 25%
D. 33%
48 CHAPTER 3 The Kidney

13. What differentiates tubular reabsorption from tubular 21. When too much protein is presented to the renal
secretion in the nephron? tubules for reabsorption, it is excreted in the urine
0
A. The direction of movement of the substance being because
absorbed or secreted is different. A. the renal threshold for protein has not been exceeded.
B. Reabsorption is an active transport process, whereas B. the maximal tubular reabsorptive capacity for protein
secretion is a passive transport process. has been exceeded.
C. Cell membrane–binding sites are different for the C. protein is not normally present in the ultrafiltrate and
reabsorption and secretion of a solute. cannot be reabsorbed.
D. The location of the epithelium in the nephron deter- D. the glomerular filtration barrier allows only abnor-
mines which process occurs. mal proteins to pass.
14. During tubular transport, the movement of a solute 22. More than 66% of filtered water, sodium, and chloride
against a gradient and 100% of filtered glucose, amino acids, and proteins
A. is called passive transport. are reabsorbed in the
B. requires little to no energy. A. collecting tubules.
0
C. involves specific cell membrane—binding sites.
D. may occur paracellularly—that is, between cells
B. distal tubules.
C. loops of Henle.
aaa

through intercellular spaces. D. proximal tubules.


15. Substances bound to plasma proteins in the blood can be 23. Water reabsorption occurs throughout the nephron
eliminated in the urine by except in the
A. glomerular secretion. A. cortical collecting tubules.
B. glomerular filtration. B. proximal convoluted tubules.
C. tubular secretion. C. ascending limb of the loops of Henle.
D. tubular reabsorption. D. descending limb of the loops of Henle.
16. Which statement characterizes the ability of the renal sys- 24. The process solely responsible for water reabsorption
tem to regulate blood pH? throughout the nephron is
0
A. The renal system has a slow response with complete
correction of the pH to normal.
/ A. osmosis.
B. the urea cycle.
B. The renal system has a fast response with complete C. the countercurrent exchange mechanism.
correction of the pH to normal. D. the countercurrent multiplier mechanism.
C. The renal system has a slow response with only par- 25. Hypertonicity of the renal medulla is maintained by
tial correction of the pH toward normal. 1. the countercurrent multiplier mechanism.
D. The renal system has a fast response with only partial 2. the countercurrent exchange mechanism.
correction of the pH toward normal. 3. the urea cycle.
17. The kidneys excrete excess alkali (base) in the urine as 4. osmosis.
A. ammonium ions. A. 1, 2, and 3 are correct.
B. ammonium salts. B. 1 and 3 are correct.
C. sodium bicarbonate. C. 4 is correct.
D. titratable acids. D. All are correct.

tubular lumen to eliminate hydrogen ions?


0
18. Which of the following substances is secreted into the 26. Which of the following is not a feature of the renal coun-
tercurrent multiplier mechanism?
A. Ammonia (NH3) A. The ascending limb of the loop of Henle is imperme-
B. Ammonium ions (NH+4 ) able to water.
C. Disodium phosphate (Na2HPO4) B. The descending limb of the loop of Henle passively
D. Monosodium phosphate (NaH2PO4) reabsorbs water.
19. Urine titratable acids can form when the ultrafiltrate C. The descending limb of the loop of Henle actively
contains 00
reabsorbs sodium and urea.
A. ammonia. D. The fluid in the ascending and descending limbs of
B. bicarbonate. the loop of Henle flows in opposite directions.
C. phosphate. 27. The purpose of the renal countercurrent multiplier

:
D. sodium. mechanism is to
20. The renal threshold level for glucose is 160 to 180 mg/dL. A. concentrate the tubular lumen fluid.
This corresponds to the B. increase the urinary excretion of urea.
A. rate of glucose reabsorption by the renal tubules. C. preserve the gradient hypertonicity in the medulla.
B. concentration of glucose in the tubular lumen fluid. D. facilitate the reabsorption of sodium and chloride.
C. plasma concentration above which tubular reabsorp-
tion of glucose occurs.
D. plasma concentration above which glucose is
excreted in the urine.
CHAPTER 3 The Kidney 49

28. Which vascular component is involved in the renal coun- 30. Which of the following describes the tubular lumen fluid
tercurrent exchange mechanism? that enters the collecting tubule compared with the tubu-
A. Afferent arteriole lar lumen fluid in the proximal tubule?
B. Efferent arteriole A. Hypo-osmotic
C. Glomerulus B. Isosmotic Is osmotic

D. Vasa recta C. Hyperosmotic

0
29. Antidiuretic hormone regulates the reabsorption of D. Counterosmotic
A. water in the collecting tubules. 31. The final concentration of the urine is determined within
B. sodium in the collecting tubules. the
C. sodium in the distal convoluted tubule. A. collecting ducts.
D. water and sodium in the loop of Henle. B. distal convoluted tubules. CD
C. loops of Henle.
APH D. proximal convoluted tubules.
4
Renal Function

LEARNING OBJECTIVES
After studying this chapter, the student should be able to: 6. Calculate osmolar clearance and free-water clearance
1. State the volume and solute composition of normal urine. results using data provided.
2. Differentiate between the solute amount (osmolality) and 7. Compare and contrast the creatinine clearance test and
the solute mass (specific gravity) in urine and describe the inulin clearance test for assessment of glomerular
ways in which they are measured. filtration.
3. Discuss the effects that diet, disease, and some exogenous 8. Describe a protocol for a creatinine clearance test, and
substances (e.g., x-ray contrast media) have on solute discuss factors that can influence the results obtained.
composition measurements. 9. Calculate creatinine clearance and estimated glomerular
4. Discuss physiologic factors involved in determining the filtration rate (eGFR) results using data provided.
volume of urine excreted. 10. Describe the p-aminohippurate clearance test for
5. Describe a protocol and one purpose for each of the assessment of renal plasma flow.
following procedures: 11. Discuss briefly the relationship of renal tubular secretory
• Fluid deprivation test function to the urinary excretion of acids.
• Osmolar clearance determination 12. Describe the oral ammonium chloride test for the
• Free-water clearance determination assessment of tubular function.

CHAPTER OUTLINE
Urine Composition, 51 Clearance Tests, 57
Solute Elimination, 51 Alternate Approaches to Assessing Glomerular Filtration
Measurements of Solute Concentration, 51 Rate, 60
Osmolality, 51 Estimated GFR (eGFR), 60
Specific Gravity, 53 Cystatin C and β2-Microglobulin, 61
Urine Volume, 54 Screening for Albuminuria, 62
Assessment of Renal Concentrating Ability/Tubular Assessment of Renal Blood Flow and Tubular Secretory
Reabsorptive Function, 55 Function, 62
Osmolality Versus Specific Gravity, 55 Determination of Renal Plasma Flow and Renal Blood
Fluid Deprivation Tests, 56 Flow, 62
Osmolar and Free-Water Clearance, 56 Assessment of Tubular Secretory Function for Acid
Assessment of Glomerular Filtration, 57 Removal, 63
Renal Clearance, 57

K E Y T E R M S*
anuria (also called anuresis) osmolar clearance
creatinine clearance test polydipsia
free-water clearance (also called solute-free water clearance) polyuria
glomerular filtration rate renal blood flow
nocturia renal clearance
oliguria renal plasma flow
osmolality specific gravity

*Definitions are provided in the chapter and glossary.

50
64 CHAPTER 4 Renal Function

5. Boothby WM: Nomogram to determine body surface area. 15. Lennon EJ, Lemann J, Litzow JR: The effect of diet and stool
Boston Med Surg J 185:337, 1921. composition on the net external acid balance of normal subjects.
6. Rock RC, Walker WG, Jennings CD: Nitrogen metabolites and J Clin Invest 45:1601–1607, 1966.
renal function. In Tietz NW, editor: Fundamentals of clinical 16. First MR: Renal function. In Kaplan LA, Pesce AJ, editors:
chemistry, ed 3, Philadelphia, 1987, WB Saunders. Clinical chemistry: theory, analysis, and correlation, ed 2,
7. Myers GL, Miller WG, Coresh J, et al: National Kidney Disease St Louis, 1989, Mosby.
Education Program Laboratory Working Group: Recom-
mendations for improving serum creatinine measurement: a
report from the Laboratory Working Group of the National
BIBLIOGRAPHY
Kidney Disease Education Program, Clin Chem 52:5–18, 2006. Alpern RJ, Hebert SC, editors: Seldin and Giebisch’s the kidney:
8. Coresh J, Stevens LA: Kidney function estimating equations: physiology and pathophysiology, ed 4, Amsterdam, 2008,
where do we stand? Curr Opin Nephrol Hypertens 15:276–284, Academic Press.
2006. Anderson SC, Cockayne S: Clinical chemistry: concepts and
9. Levey AS, Stevens LA, Schmid CH, et al: A new equation to applications, New York, 2003, McGraw-Hill.
estimate glomerular filtration rate. Ann Intern Med 150: Goldman L, Schafer AI, editors: Goldman-Cecil medicine, ed 25,
604–612, 2009. Philadelphia, 2015, Saunders.
10. Levey AS, Coresh J, Balk E, et al: National Kidney Foundation Hall JE: Guyton and Hall textbook of medical physiology, ed 12,
practice guideline for chronic kidney disease evaluation, Philadelphia, 2011, Saunders.
classification, and stratification. Ann Intern Med 139:137–147, Kumar V, Abbas AK, Aster JC, editors: Robbins and Cotran
2003. pathologic basis of disease, ed 9, Philadelphia, 2014, Saunders.
11. Laterza OF, Price CP, Scott MG: Cystatin C: an improved Patton KT, Thibodeau GA: Anatomy and physiology, ed 7,
estimator of glomerular filtration rate? Clin Chem 48:699–707, Philadelphia, 2010, Saunders.
2002. Pincus MR, Bock JL, Bluth MH: Evaluation of renal function, water,
12. Pucci L, Triscornia S, Lucchesi D, et al: Cystatin C and estimates electrolytes, and acid-base balance. In McPherson RA,
of renal function: searching for a better measure of kidney Ben-Ezra J, editors: Clinical diagnosis and management by
function in diabetes patients. Clin Chem 53:480–488, 2007. laboratory methods, ed 22, Philadelphia, 2011, Saunders.
13. American Diabetics Association: Nephropathy in diabetes Schrier RW, Gottschalk CW: Diseases of the kidney and urinary
(position statement). Diabetes Care 27(Suppl 1):S79–S83, 2004. tract, ed 7, Philadelphia, 2001, Lippincott Williams & Wilkins.
14. Dustan H, Corcoran A: Functional interpretation of renal tests.
Med Clin North Am 39:947–956, 1955.

STUDY QUESTIONS

molar amounts in urine? 0


1. Which of the following solutes are present in the largest 0
5. The osmolality of a solution containing 1.0 mole of urea
is equal to that of a solution containing
A. Urea, chloride, and sodium - ucs A. 1.0 mole of HCl.
B. Urea, creatinine, and sodium B. 1.0 mole of H2PO4. 1.0 mole 0.5
=

C. Creatinine, uric acid, and ammonium C. 0.5 mole of NaCl. mile NaCl
D. Urea, uric acid, and ammonium D. 0.5 mole of glucose.

500
2. Renal excretion is not involved in the elimination of 6. The maximum osmolality that urine can achieve is deter-
A. electrolytes and water. mined by the
B. normal byproducts of fat metabolism. A. quantity of solutes ingested in the diet.
C. soluble metabolic wastes (e.g., urea, creatinine). B. presence of antidiuretic hormone in the collecting
D. exogenous substances (e.g., drugs, x-ray contrast tubules.
media). C. osmolality of the medullary interstitium.
3. The concentration of which substances provides the best D. osmolality of fluid entering the collecting tubules.
O
means of distinguishing urine from other body fluids? 7. Serum osmolality remains relatively constant, whereas
A. Creatinine and urea the urine osmolality ranges from
B. Glucose and protein A. one-third to one-half that of serum.
C. Uric acid and ammonia B. one-third to equal that of serum.
D. Water and electrolytes C. one to three times that of serum.
O
4. What is the definition of the osmolality of a solution? D. three to five times that of serum.
A. The density of solute particles per liter of solvent
B. The mass of solute particles per kilogram of solvent
O
8. Another name for excessive thirst is
A. polydipsia.

solvent

C. The number of solute particles per kilogram of B. polyuria.
C. hydrophilia.
> 3L excessive
earth a

D. The weight of solute particles per liter of solvent D. hydrostasis.

number
osmolality
=
CHAPTER 4 Renal Function 65

O
9. Specific gravity measurements are not affected by
A. temperature.
O
18. All of the following conditions may produce nocturia except
A. anuria.
B. solute charge. B. pregnancy.
C. solute mass. C. chronic renal disease.
D. solute number. D. fluid intake at night.
10. Osmolality is a measure of solute
A. density.
O
19. Which renal function is assessed using specific gravity
and osmolality measurements?
B. mass. A. Concentrating ability
Osmolality number
=

C. number. B. Glomerular filtration ability


D. weight. C. Tubular excretion ability
11. Which of the following solutes, if added to pure water, D. Tubular secretion ability
affects the specific gravity of the resultant solution more 20. A fluid deprivation test is used to
than it affects its osmolality? A. determine renal plasma flow.
A. Sodium B. investigate the cause of oliguria.
B. Chloride C. assess renal concentrating ability.
C. Potassium D. measure the glomerular filtration rate.
☆ D. Glucose 21. A fluid deprivation test involves the measurement of
12. Occasionally the specific gravity of a urine specimen serum and urine

exceeds that physiologically possible (i.e., >1.040). A. density.
osmolality =
fluid
Which of the following substances when found in urine
could account for such a high value?
B. osmolality.
C. specific gravity. deprivation
A. Creatinine D. volume.
B. Glucose 22. The volume of plasma cleared per minute in
/Mannitol
C. ingestion of ✗
rag dyes excess of that required for solute elimination is called the

0
D. Protein A. creatinine clearance.
13. The excretion of large volumes of urine (>3 L/day) is B. free-water clearance.
called C. osmolar clearance.
A. glucosuria. D. renal clearance.
B. hyperuria. 23. A free-water clearance value of !1.2 would be expected

↳ jammed
C. polydipsia. from a patient experiencing
D. polyuria. A. polyuria. Mary
14. The daily volume of urine excreted normally ranges from B. dehydration.
A. 100 to 500 mL/day. C. water diuresis.
B. 100 to 1800 mL/day. D. excessive fluid intake.
C. 500 to 1800 mL/day. 24. Calculate the osmolar and free-water clearances using the
D. 1000 to 3000 mL/day. following patient data.
O
15. When the body is dehydrated, the kidneys
A. excrete excess solutes in a constant volume
Serum osmolality: 305 mOsm/kg
Urine osmolality: 250 mOsm/kg
of urine. Urine volume: 300 mL/2 hours

÷
B. excrete solutes in as small a volume of urine as A. Is this individual excreting more water than is neces-
possible. sary for solute removal? Yes/No
C. decrease the quantity of solutes excreted and decrease B. Is the osmolar clearance “normal” (i.e., 2.0 to
the urine volume. 3.0 mL/min)? Yes/No
D. increase the quantity of solutes excreted while hold- C. From the free-water clearance result obtained, is the
ing the urine volume constant. urine hypo-osmotic or hyperosmotic?
16. The excretion of less than 400 mL of urine per day is 25. Which of the following is an endogenous substance used
called to measure glomerular filtration rate?
m
-

A. anuria. < 4W mL A. Urea


B. hypouria. B. Inulin Cra
endo
-
-

C. nocturia. night origine C. Creatinine


-

D. oliguria. D. p-aminohippurate
17. The ultrafiltrate in the urinary space of the glomerulus 26. Renal clearance is defined as the volume of

0
has a specific gravity of A. urine cleared of a substance per minute.
A. 1.005 and a lower osmolality than the blood plasma. B. plasma cleared of a substance in a time interval.

0
B. 1.010 and the same osmolality as the blood plasma.
C. 1.015 and a higher osmolality than the blood plasma.
D. 1.035 and a higher osmolality than the blood plasma.
C. plasma flowing through the kidney per minute.
D. plasma containing the same amount of substance in
1 mL of urine.
66 CHAPTER 4 Renal Function

0
27. Creatinine is a good substance to use for a renal clearance
test because it
33. A 45-year-old female African American had her serum
creatinine determined using a creatinine method that is
A. is exogenous. Crea Constant
=
not calibrated to an IDMS reference method. Her serum
B. is reabsorbed. creatinine was 1.5 mg/dL; what is her eGFR using the
C. is affected by fluid intake. appropriate MDRD equation?
D. has a 0 constant plasma concentration. A. 40 mL/min/1.73 m2
28. Which of the following groups would be expected to have B. 48 mL/min/1.73 m2
the greatest 24-hour excretion of creatinine? C. 51 mL/min/1.73 m2
A. Infants D. 54 mL/min/1.73 m2
B. Children 34. The glomerular filtration rate is controlled by
C. Women A. the renal blood flow.
/ GFR RBF
=

D. Men B. the renal plasma flow.


29. Creatinine clearance results are “normalized” using an C. the countercurrent mechanism.
individual’s body surface area to account for variations D. hormones (e.g., aldosterone, antidiuretic hormone).
in the individual’s 35. For measurement of renal plasma flow, p-aminohippurate
A. age. is an ideal substance to use because it
B. sex. A. is easily measured in urine and plasma.
C. dietary intake. B. is endogenous and does not require an infusion.
Mhk w
D. muscle mass. C. is secreted completely in its first pass through the
30. The following data are obtained from a 60-year-old kidneys.
female who is 40 800 tall and weighs 88 lb: D. maintains a constant plasma concentration through-
Plasma creatinine: 1.2 mg/dL out the test.
Urine creatinine: 500 mg/L
Urine volume: 1440 mL/24 hr

O
A. Calculate the creatinine clearance. 42mL / min
the kidneys?
A. 8%
O
36. What percentage of the total cardiac output is received by

O
B. Calculate the normalized creatinine clearance. (Deter- B. 15%
mine the body surface area using Equation 4.6.) 58mL / min C. 25%
C. Are these results normal for this patient? (Use refer- D. 33%
O
ence intervals provided in Table 4.3.) Yes
31. A 24-hour urine collection is preferred for determina-
37. Measuring the quantity of hydrogen ion excreted as
titratable acids and ammonium salts in urine provides
tion of creatinine clearance because of diurnal variation a measure of
in the A. tubular secretory function.
A. glomerular filtration rate. B. tubular reabsorptive function.
B. plasma creatinine. C. glomerular filtration ability.
C. creatinine excretion.
GER D. renal concentrating ability.
D. urine excretion. 38. The oral ammonium chloride test evaluates the ability of
32. Which of the following situations results in an erroneous the tubules to secrete
creatinine clearance measurement? A. ammonium and chloride.
A. A 24-hour urine collection from an individual on a B. phosphate and sodium.
vegetarian diet C. bicarbonate and chloride.
B. A 24-hour urine collection maintained at room tem- D. ammonia and hydrogen.
perature throughout the collection
C. A plasma sample drawn at the beginning instead of
during the 24-hour urine collection
D. Creatinine determinations made using the nonspeci-
fic alkaline picrate method (Jaffe reaction)
CHAPTER 4 Renal Function 67

Case 4.1
A 52-year-old woman with a 25-year history of type 1 diabetes 1. Calculate this patient’s body surface area using Equation 4.6. 1.80
mellitus submits a 24-hour urine collection for testing. A blood 2. Calculate the normalized creatinine clearance result using the
sample was also collected when she brought the urine speci- data provided. 55mL / min
men to the laboratory. The following information and results 3. Calculate the albumin excretion in milligrams per day
are obtained: (mg/day). 95mg / day
4. Calculate the albumin excretion in micrograms per minute
Patient Information (μg/min). 66h91min
Height: 50 500 (165 cm) 5. Calculate the urine albumin-to-creatinine ratio in micrograms
Weight: 160 lb (72.7 kg) albumin per milligram of creatinine (μg albumin/mg creati-
nine). 50mg 1mg creatine
Results Reference Intervals
Creatinine, serum: 2.3 mg/dL 0.8 – 1.3 mg/dL
Urine volume, 24-hour: 1000 mL 600 – 1800 mL/day
Creatinine, urine: 190 mg/dL Varies with hydration
Albumin, urine: 9.5 mg/dL Varies with hydration

Case 4.2
A 24-year-old man who had previously sustained a severe head 3. This patient’s polyuria should be classified as
injury in a car accident is seen by his physician. He complains of A. oncotic diuresis.
polydipsia and polyuria. Neurogenic diabetes insipidus is sus- B. psychosomatic diuresis.
pected, and tests are performed to rule out compulsive water C. solute diuresis.
ingestion. The following routine urinalysis is obtained. D. water diuresis.
4. In patients with neurogenic diabetes insipidus, if antidiuretic
Results: hormone is given intravenously, the urine osmolality should
A. remain unchanged.
Physical Examination Chemical Examination B. decrease.
Color: colorless SG: 1.005 C. increase.
Clarity: clear pH: 6.0 5. Which of the following tests should be used to evaluate this
Odor: — Blood: negative patient?
Protein: negative A. Free-water clearance test
LE: negative B. Fluid deprivation test
Nitrite: negative C. Glucose tolerance test
Glucose: negative D. Osmolar clearance test
Ketones: negative
Indicate whether each of the following statements is true (T) or
Bilirubin: negative
false (F).
Urobilinogen: normal F 6. Patients with diabetes insipidus often have glucose pre-
___
Ascorbic acid: —
sent in the urine.
F 7. Patients with diabetes insipidus often have a high urine
___
1. Explain briefly the cause of polyuria in patients with diabetes specific gravity.
insipidus. F 8. Patients with diabetes insipidus often have urinary
___
2. Without fluid restrictions, this patient’s urine osmolality most ketones present because of an inability to use the glucose
likely is present in the blood.
T 9. Diabetes is a general term referring to disorders charac-
___
A. less than 200 mOsm/kg.
B. greater than 200 mOsm/kg. terized by copious production and excretion of urine.

LE, Leukocyte esterase; SG, specific gravity.


5
Physical Examination of Urine

LEARNING OBJECTIVES
After studying this chapter, the student should be able to: 7. Identify two variables involved in determining urine
1. State the importance of using established terminology for concentration.
describing urine color and clarity. 8. Compare and contrast the specific gravity and osmolality
2. Discuss the origin of the following pigments and their determinations for the measurement of urine concen-
effects on urine color: tration.
• Bilirubin 9. State the principle of each of the following specific gravity
• Urobilin determination methods:
• Urochrome • Harmonic oscillation densitometry
• Uroerythrin • Reagent strip method
3. List appropriate color terms and the substances that can • Refractometry
produce the colors, and identify those substances that • Urinometer method
indicate a pathologic process. 10. Differentiate between direct and indirect measures of
4. List appropriate clarity terms, their definitions, and the urine specific gravity, and compare the limitations of
substances that can cause clarity changes, and identify each method.
those substances that indicate a pathologic process. 11. State the principle of the following osmometry methods:
5. Describe the effects that increased amounts of protein • Freezing point depression
and bilirubin can have on urine foam. • Vapor pressure depression
6. Discuss the cause of normal urine odor, identify 12. Discuss factors that affect urine volume and the terms
conditions that change this urine characteristic, and list used to describe volume variations.
any odors associated with each condition.

CHAPTER OUTLINE
Color, 69 Concentration, 74
Foam, 71 Specific Gravity, 74
Clarity, 72 Osmolality, 79
Odor, 73 Volume, 80
Taste, 74

K E Y T E R M S*
clarity (also called turbidity) refractive index
colligative property refractometry
density urobilin
diuresis urochrome
freezing point osmometer uroerythrin
ionic specific gravity vapor pressure osmometer

*Definitions are provided in the chapter and glossary.

The study of urine is the oldest clinical laboratory test still per- “sweetness” of urine. The physical characteristics of urine
formed. Historically, only the physical characteristics of urine continue to play an important part in a routine urinalysis.
were evaluated—color, clarity, odor, and taste. The latter The presence of disease processes and abnormal urine
characteristic—taste—has not been performed for centuries components can be evident during the initial physical exam-
because of chemical methods that can be used to assess the ination of urine.

68
CHAPTER 5 Physical Examination of Urine 81

oliguria can result. Oliguria also develops with various renal 2. de Wardener HE: The kidney, ed 5, New York, 1985, Churchill
diseases, ranging from urinary tract obstruction to end-stage Livingstone.
renal disease. 3. Schweitzer SC, Schumann JL, Schumann GB: Quality assurance
Anuria is the complete lack of urine excretion. Anuria is guidelines for the urinalysis laboratory. J Med Technol 3:569,
1986.
fatal if not immediately addressed because of the accumula-
4. Clinical and Laboratory Standards Institute (CLSI): Urinalysis:
tion of toxic metabolic byproducts in the body. Any condition
approved guideline, ed 3, CLSI Document GP16-A3, CLSI,
or disease, chronic or acute, that destroys functioning renal Wayne, PA, 2009.
tissue can result in anuria. Principal among these are condi- 5. Chadha V, Garg U, Alon U: Measurement of urinary
tions that decrease the blood supply to renal tissue, such as concentration: a critical appraisal of methodologies, Pediatr
hypotension, hemorrhage, shock, and heart failure. Toxic Nephrol 16:374–382, 2001.
chemicals and nephrotoxic antibiotics can induce acute tubu- 6. Tietz NW, Pruden EL, Siggaard-Andersen O: Electrolytes, blood
lar necrosis, leading to loss of functional renal tissue and gases, and acid-base balance. In Tietz NW, editor: Fundamentals
anuria (or oliguria). In addition, hemolytic transfusion reac- of clinical chemistry, ed 3, Philadelphia, 1987, WB Saunders.
tions and urinary tract obstructions can result in anuria.
In conclusion, urine volume measurements are not per-
formed routinely. Although this information can serve as a BIBLIOGRAPHY
valuable diagnostic aid, urine volume is usually determined
with a timed urine collection and is used to calculate the con- Alpern RJ, Hebert SC, editors: Seldin and Giebisch’s the kidney:
physiology and pathophysiology, ed 4, Amsterdam, 2008,
centration of specific urine solutes or to assess renal function
Academic Press.
(e.g., the glomerular filtration rate). Chapter 4 discusses renal
Burtis CA, Ashwood ER, editors: Tietz textbook of clinical chemistry,
function and its effect on urine volume. The terms polyuria, ed 3, Philadelphia, 1999, WB Saunders.
oliguria, and anuria are usually assigned on the basis of a Goldman L, Schafer AI, editors: Goldman-Cecil medicine, ed 25,
patient’s health history and clinical observation and are not Philadelphia, 2015, Saunders.
based on timed urine collections. Table 5.7 outlines these Kumar V, Abbas A, Aster J: Robbins and Cotran pathologic basis of
urine volume terms, their definitions, and their causes. disease, ed 8, Philadelphia, 2010, Saunders.
Schrier RW, Gottschalk CW: Diseases of the kidney and urinary
tract, ed 7, Philadelphia, 2001, Lippincott Williams & Wilkins.
REFERENCES
1. Drabkin DL: The normal pigment of urine: the relationship of
urinary pigment output to diet and metabolism. J Biol Chem
75:443–479, 1927.

STUDY QUESTIONS
1. The color of normal urine is due to the pigment 4. Which of the following pigments deposits on urate and
A. bilirubin. uric acid crystals to form a precipitate described as “brick
B. urobilin. dust”?
C. uroerythrin. A. Bilirubin
D. urochrome. B. Urobilin
2. A single substance can impart different colors to urine C. Uroerythrin
depending on the D. Urochrome
1. amount of the substance present. 5. Match the colors to the urine pigment/substance. Note
2. storage conditions of the urine. that more than one color can be selected for a pigment/
3. pH of the urine. substance.
4. structural form of the substance.
A. 1, 2, and 3 are correct. Urine Pigment/ Color of Pigment/
B. 1 and 3 are correct. Substance Substance
C. 4 is correct. __ A. Bilirubin
2,3 1. Colorless
D. All are correct. __
8 B. Biliverdin 2. Yellow
__ C. Hemoglobin
4,7 3. Orange
3. Which of the following urine characteristics provides the
__) D. Myoglobin
714 4. Red
best rough indicator of urine concentration and body __) E. Porphobilinogen
614 5. Pink
hydration? __ F. Urobilin
3,7 6. Purple
A. Color __
I G. Urobilinogen 7. Brown
B. Clarity __
2 H. Urochrome 8. Green
C. Foam __
5 I. Uroerythrin

D. Volume
82 CHAPTER 5 Physical Examination of Urine

6. Which of the following criteria should one use to consis- 12. Which of the following urine specimens is considered
tently evaluate urine color and clarity? normal?
1. Mix all specimens well. A. A freshly voided urine that is brown and clear
2. Use the same depth or volume of a specimen. B. A freshly voided urine that is yellow and cloudy
3. Evaluate the specimens at the same temperature. C. A clear yellow urine specimen that changes color
4. View the specimens against a dark background upon standing
with good lighting. D. A clear yellow urine specimen that becomes cloudy
A. 1, 2, and 3 are correct. upon refrigeration
B. 1 and 3 are correct. 13. A white or beige precipitate in a “normal” alkaline urine
C. 4 is correct. most likely is caused by
D. All are correct. A. amorphous phosphates.
7. Select the urine specimen that does not indicate the pos- B. amorphous urates.
sible presence of blood or hemoglobin. C. uric acid crystals.
A. Clear, red urine D. radiographic contrast media.
B. Cloudy, brown urine 14. Match the urine odor to the condition or substance that
C. Clear, brown urine can cause it. You may select more than one odor for a
D. Cloudy, amber urine condition.
8. A urine that produces a large amount of white foam
when mixed should be suspected to contain increased Condition/Substance Urine Odor
amounts of __
b- A. Diabetes mellitus 1. Ammonia-like
A. bilirubin. __
3 B. Normal urine 2. Bleach
__ C. Old, improperly stored urine 3. Faintly aromatic
B. protein. :
__ D. Specimen adulteration 4. Pungent, fetid
C. urobilin.
__
5 E. Starvation 5. Sweet, fruity
D. urobilinogen. __ F. Urinary tract infection
1,4
9. Which of the following substances can change the color
of a urine and its foam? 15. Which of the following methods used to determine the
A. Bilirubin specific gravity of urine does not detect the presence of
B. Hemoglobin urine protein or glucose?
C. Myoglobin A. Harmonic oscillation densitometry
D. Urobilin B. Reagent strip
10. The clarity of a well-mixed urine specimen that has vis- C. Refractometry
ible particulate matter and through which newsprint can D. Urinometry
be seen but not read should be described as 16. A small ion and a large uncharged molecule have the
A. cloudy. same effect when urine concentration is determined by
B. flocculated. A. urinometry.
C. slightly cloudy. B. osmolality.
D. turbid. C. reagent strip.
11. Classify each substance that can be present in urine D. refractometry.
as indicating a (1) pathologic or (2) nonpathologic 17. Which of the following specific gravity values is physio-
condition. logically impossible?
__
I A. Bacteria (fresh urine)
A. 1.000
__ B. Bacteria (old urine)
2
B. 1.010
__
I C. Fat
C. 1.020
__ D. Powder
2
D. 1.030
__
2 E. Radiographic contrast media 18. Match the principle to the appropriate specific gravity
__
I F. Red blood cells
method. A principle can be used more than once.
__ G. Renal epithelial cells
1

__
2 H. Spermatozoa Specific Gravity Method Method
__ I. Squamous epithelial cells
2 I
__ A. Harmonic oscillation 1. Density
__
2 J. Urate crystals densitometry 2. Refractive index
__
I K. White blood cells __
3 B. Reagent strip 3. pKa changes
__
I L. Yeast __
2 C. Refractometry

__
I D. Urinometry
CHAPTER 5 Physical Examination of Urine 83

19. Which of the following methods is an indirect measure of 23. Ionic specific gravity (SGionic) measurements obtained
specific gravity? using reagent strips provide useful clinical information
1. Reagent strip because
2. Urinometry A. all of the urinary solutes present are measured.
3. Refractometry B. the quantity of nonionic solutes in urine relative to
4. Harmonic oscillation densitometry ionic solutes is significant.
A. 1, 2, and 3 are correct. C. excretion of nonionic solutes (e.g., urea, glucose, pro-
B. 1 and 3 are correct. tein) does not reflect renal dysfunction.
C. 4 is correct. D. the ability of the kidneys to concentrate urine is
D. All are correct. reflected in the reabsorption and secretion of ionic
20. The refractive index of a solution is affected by the solutes.
1. wavelength of light used. 24. Which of the following as described is not a colligative
2. size and number of the solutes present. property?
3. concentration of the solution. A. Boiling point elevation
4. temperature of the solution. B. Freezing point depression
A. 1, 2, and 3 are correct. C. Osmotic pressure depression
B. 1 and 3 are correct. D. Vapor pressure depression
C. 4 is correct. 25. An advantage of freezing point osmometry over vapor
D. All are correct. pressure osmometry is its
21. Refractometry is preferred for specific gravity measure- A. increased turnaround time.
ments because it B. use of a smaller volume of sample.
1. uses a small amount of sample. C. ability to detect volatile substances.
2. is fast and easy to perform. D. decreased interference from plasma lipids.
3. automatically compensates for temperature. 26. Osmolality measurements are considered to be a more
4. measures only ionic solutes. accurate assessment of solute concentration in body
A. 1, 2, and 3 are correct. fluids than are specific gravity measurements because
B. 1 and 3 are correct. A. all solutes contribute equally.
C. 4 is correct. B. heavy molecules do not interfere.
D. All are correct. C. they are not temperature dependent.
22. The principle of the reagent strip method for measuring D. they are less time-consuming to perform.
specific gravity is based on 27. The freezing point of a urine specimen is determined to
A. the pKa of a polyelectrolyte decreasing in proportion be !0.90°C. What is the osmolality of the specimen?
to the ionic concentration of the specimen. A. 161 mOsm/kg
B. the pH of a polyelectrolyte decreasing in proportion B. 484 mOsm/kg
to the ionic concentration of the specimen. C. 597 mOsm/kg
C. the pKa of a polyelectrolyte increasing in proportion to D. 645 mOsm/kg
the ionic concentration of the specimen. 28. Which of the following will not influence the volume of
D. the pH of a polyelectrolyte increasing in proportion urine produced?
to the ionic concentration of the specimen. A. Diarrhea
B. Exercise
C. Caffeine ingestion
D. Carbohydrate ingestion

Case 5.1
A routine urinalysis on a urine specimen collected from a hospi- 2. Which of the following actions should be taken?
talized patient revealed a specific gravity greater than 1.050 with A. Report the urinalysis results; no further action is needed.
the use of refractometry. B. Report the urinalysis results and suggest that the patient be
1. The best explanation for this specific gravity result is that the instructed to increase fluid intake.
urine specimen C. Contact the patient care unit to determine whether the
A. is old and has deteriorated. patient is taking a diuretic; if so, report the urinalysis results.
B. contains radiographic contrast media. D. Do not report the urinalysis results; request that a urine
C. is concentrated because the patient is ill and dehydrated. specimen be recollected after several hours.
D. contains abnormally high levels of sodium and other elec-
trolytes because the patient is taking diuretics.
84 CHAPTER 5 Physical Examination of Urine

Case 5.2
A prenatal examination including a routine urinalysis is per- 2. The urine specimen was placed in a refrigerator while the
formed on a 28-year-old female. Her physical examination is laboratory determined whether the physician wanted a
unremarkable. When asked about her health, she states that it microscopic examination performed (i.e., the request slip
is generally good, except for several urinary tract infections in was not appropriately completed). When the specimen
the past. In fact, she thinks she might be getting one now and was later removed to prepare an aliquot for microscopic
has been taking an over-the-counter product that lessens her dis- analysis, the specimen was still orange but was now
comfort. The following urinalysis results are obtained: cloudy. Which of the following statements best explains
Color: bright orange (like soda pop) this increase in urine turbidity?
Clarity: clear A. The delay in analysis has allowed bacteria to proliferate.
1. Which of the following statements best explains the orange B. Squamous epithelial cells in the urine have degenerated.
color of the urine? C. Because of the temperature change, normal urine sol-
A. She has a liver disorder, and bilirubin is present in the urine. utes have precipitated.
B. The urine is concentrated, which can be confirmed by the D. The specimen was contaminated with vaginal secre-
urine specific gravity. tions and yeast has propagated.
C. She has recently eaten fresh beets and is genetically
disposed to produce this abnormally colored urine.
D. The over-the-counter product contains phenazopyridine,
which imparts this characteristic color to urine.
6
Chemical Examination of Urine

LEARNING OBJECTIVES
After studying this chapter, the student should be able to: 10. Differentiate between hematuria and hemoglobinuria.
1. State the proper care and storage of commercial reagent 11. Discuss the clinical significance of myoglobin. Compare
strip and tablet tests and cite at least three potential and contrast myoglobinuria and hemoglobinuria.
causes of their deterioration. 12. Discuss the limitations of leukocyte esterase and nitrite
2. Describe quality control procedures for commercial reagent strip tests for the detection of leukocyturia and
reagent strip and tablet tests. bacteriuria.
3. Discuss appropriate specimen and testing techniques 13. Describe two physiologic mechanisms that result in
used with commercial reagent strip and tablet tests. glucosuria.
4. State the chemical principle used on reagent strips for 14. Compare and contrast the glucose reagent strip test and
measurement of the following: the copper reduction test for the measurements of sugars
• Specific gravity in urine.
• pH 15. Describe three conditions that result in ketonuria.
5. Summarize the clinical significance of the following 16. Briefly explain the metabolic pathway that results in
substances when present in urine and describe the ketone formation, state the relative concentrations of
chemical principles used on reagent strips to measure the three ketones formed, and discuss the reagent strip
them: and tablet tests used to detect them.
• Protein 17. Summarize the formation of bilirubin and urobilinogen,
• Blood discuss their clinical significance, and describe three
• Leukocyte esterase physiologic mechanisms that result in altered bilirubin
• Nitrite metabolism.
• Glucose 18. Compare and contrast the principle, sensitivity, specific-
• Ketones ity, and limitations of the following methods for detection
• Bilirubin of bilirubin in urine:
• Urobilinogen • Physical examination
• Ascorbic acid • Reagent strip test
6. Compare and contrast the sensitivity, specificity, and • Tablet test
potential interferences of each commercial reagent strip 19. Describe two chemical principles used by reagent strip
and tablet test. tests to detect urine urobilinogen and compare their
7. Compare and contrast the mechanisms for and the sensitivity, specificity, and limitations.
clinical significance of the following types of proteinurias: 20. State the importance of ascorbic acid detection in urine,
• Overflow proteinuria and describe methods used to detect ascorbic acid.
• Glomerular proteinuria 21. Identify reagent strip tests that are affected adversely
• Postural proteinuria by ascorbic acid, and explain the mechanism of
• Tubular proteinuria interference.
• Postrenal proteinuria 22. Describe the role of reflex testing in urinalysis and discuss
8. Discuss the clinical features of the nephrotic syndrome the correlation between results obtained in the chemical
and Fanconi’s syndrome, including the specific renal examination and what they imply for the microscopic
dysfunctions involved. examination.
9. Compare and contrast the chemical principle, sensitivity,
and specificity of the following tests for the detection of
proteins in the urine:
• Reagent strip protein test
• Sensitive albumin tests (i.e., microalbumin)

85
CHAPTER 6 Chemical Examination of Urine 119

22. Caraway WT, Watts NB: Carbohydrates. In Tietz NW, editor: 24. Sherwin JE: Liver function. In Kaplan LA, Pesce AJ, editors:
Fundamentals of clinical chemistry, ed 3, Philadelphia, 1987, Clinical chemistry, theory, analysis, and correlation, ed 2,
WB Saunders. St. Louis, 1989, Mosby.
23. Csako G: Causes, consequences, and recognition of false-positive 25. Kanis JA: Detection of urinary porphobilinogen. Lancet 1:1511,
reactions for ketones. Clin Chem 36:1388–1389, 1990. 1973.

STUDY QUESTIONS
1. To preserve the integrity of reagent strips, it is necessary 6. Select the primary reason why tablet (e.g., Ictotest)
that they are and chemical tests (e.g., sulfosalicylic acid precipitation
A. humidified adequately. test) generally are performed.
B. stored in a refrigerator. A. They confirm results suspected about the specimen.
C. stored in a tightly capped container. B. They are alternative testing methods for highly con-
D. protected from the dark. centrated urines.
2. Using quality control materials, one should check reagent C. Their specificity differs from that of the reagent
strip performance strip test.
1. at least once daily. D. They are more sensitive to the chemical constituents
2. when a new bottle of strips or tablets is opened. in urine.
3. when a new lot number of strips or tablets is 7. In a patient with chronic renal disease, in whom the kid-
placed into use. neys can no longer adjust urine concentration, the urine
4. once each shift by each laboratorian performing specific gravity would be
urinalysis testing. A. 1.000.
A. 1, 2, and 3 are correct. B. 1.010.
B. 1 and 3 are correct. C. 1.020.
C. 4 is correct. D. 1.030.
D. All are correct. 8. Urine pH normally ranges from
3. Which of the following is not checked by quality control A. 4.0 to 9.0.
materials? B. 4.5 to 7.0.
A. The technical skills of the personnel performing C. 4.5 to 8.0.
the test D. 5.0 to 6.0.
B. The integrity of the specimen, that is, that the spec- 9. Urine pH can be modified by all of the following except
imen was collected and stored properly A. diet.
C. The test protocol, that is, that the procedure was B. increased ingestion of water.
performed according to written guidelines C. ingestion of medications.
D. The functioning of the equipment used—for D. urinary tract infections.
example, the refractometer and the reagent strip 10. The double-indicator system used by commercial reagent
readers strips to determine urine pH uses which two indicator
4. Quality control materials used to assess the performance dyes?
of reagent strips and tablet tests must A. Methyl orange and bromphenol blue
A. be purchased from a commercial manufacturer. B. Methyl red and bromthymol blue
B. yield the same results regardless of the commercial C. Phenol red and thymol blue
brand used. D. Phenolphthalein and litmus
C. contain chemical constituents at realistic and critical 11. All of the following can result in inaccurate urine pH
detection levels. measurements except
D. include constituents to assess the chemical and A. large amounts of protein present in the urine.
microscopic examinations. B. double-dipping of the reagent strip into the
5. Which of the following is not a source of erroneous specimen.
results when reagent strips are used? C. maintaining the specimen at room temperature for
A. Testing a refrigerated urine specimen 4 hours.
B. Timing using a clock without a second hand D. allowing excess urine to remain on the reagent strip
C. Allowing excess urine to remain on the reagent strip during the timing interval.
D. Dipping the reagent strip briefly into the urine specimen. 12. Normally, daily urine protein excretion does not
exceed
A. 150 mg/day.
B. 500 mg/day.
C. 1.5 g/day.
D. 2.5 g/day.
120 CHAPTER 6 Chemical Examination of Urine

13. Which of the following proteins originates in the urinary 18. Select the correct statement(s).
tract? 1. Myoglobin and hemoglobin are reabsorbed read-
A. Albumin ily by renal tubular cells.
B. Bence Jones protein 2. Hemosiderin, a soluble storage form of iron, is
C. β2-Microglobulin found in aqueous solutions.
D. Uromodulin 3. When haptoglobin is saturated, free hemoglobin
14. Match each type of proteinuria with its description. passes through the glomerular filtration barrier.
Type of 4. Hemosiderin is found in the urine during a hemo-
Description Proteinuria lytic episode.
3 A. Defective protein reabsorption
__ 1. Overflow A. 1, 2, and 3 are correct.
in the nephrons proteinuria B. 1 and 3 are correct.
2 B. Increased urine albumin and
__ 2. Glomerular C. 4 is correct.
mid- to high-molecular- proteinuria D. All are correct.
weight proteins 3. Tubular 19. Which statement about hemoglobin and myoglobin is
3 C. Increase in low-molecular-
__ proteinuria true?
weight proteins in urine 4. Postrenal A. They are heme-containing proteins involved in oxy-
__
I D. Immunoglobulin light chains proteinuria gen transport.
in the urine B. Their presence is suspected when urine and serum
__
4 E. Proteins originating from a
appear red.
bladder tumor
__
2 F. Protein excreted only in an
C. Their presence in serum is associated with high cre-
orthostatic position atine kinase values.
__
I G. Hemoglobinuria and D. They precipitate out of solution when the urine is
myoglobinuria 80% saturated with ammonium sulfate.
__
2 H. Nephrotic syndrome 20. On the reagent strip test for blood, any heme moiety (e.g.,
__
3 I. Fanconi’s syndrome hemoglobin, myoglobin) present in urine catalyzes
A. oxidation of the chromogen and hydrogen peroxide.
15. Which of the following statements about Bence Jones B. reduction of the chromogen in the presence of
protein is correct? hydrogen peroxide.
A. The protein consists of κ and λ light chains. C. reduction of the pseudoperoxidase while the chro-
B. The protein is often found in the urine of patients mogen undergoes a color change.
with multiple sclerosis. D. oxidation of the chromogen while hydrogen perox-
C. The protein precipitates when urine is heated to ide is reduced.
100°C and redissolves when cooled to 60°C. 21. Which of the following blood cells will not be detected by
D. The protein can produce a positive reagent strip the leukocyte esterase pad because it lacks esterases?
protein test. A. Eosinophils
16. Which of the following statements best describes B. Lymphocytes
the chemical principle of the protein reagent strip test? C. Monocytes
A. The protein reacts with an immunocomplex on the D. Neutrophils
pad, which results in a color change. 22. Microscopic examination of a urine sediment revealed an
B. The protein causes a pH change on the reagent strip average of 2 to 5 white blood cells per high-power field,
pad, which results in a color change. whereas the leukocyte esterase test by reagent strip was
C. The protein accepts hydrogen ions from the indica- negative. Which of the following statements best
tor dye, which results in a color change. accounts for this discrepancy?
D. The protein causes protons to be released from a A. The urine is contaminated with vaginal fluid.
polyelectrolyte, which results in a color change. B. Many white blood cells are lysed, and their esterase
17. Which of the following aids in the differentiation of has been inactivated.
hemoglobinuria and hematuria? C. Ascorbic acid is interfering with the reaction on the
A. Urine pH reagent strip.
B. Urine color D. The amount of esterase present is below the sensitiv-
C. Leukocyte esterase test ity of the reagent strip test.
D. Microscopic examination
CHAPTER 6 Chemical Examination of Urine 121

23. Which of the following statements describes the chemical 28. Which of the following substances if present in the urine
principle involved in the leukocyte esterase pad of com- results in a negative Clinitest?
mercial reagent strips? A. Fructose
A. Leukocyte esterase reacts with a diazonium salt on B. Lactose
the reagent pad to form an azo dye. C. Galactose
B. An ester and a diazonium salt combine to form an D. Sucrose
azo dye in the presence of leukocyte esterase. 29. The glucose reagent strip test is more sensitive and spe-
C. An aromatic compound on the reagent pad combines cific for glucose than the Clinitest method because it
with leukocyte esterase to form an azo dye. detects
D. Leukocyte esterase hydrolyzes an ester on the reagent A. other reducing substances and higher concentrations
pad; then an azo coupling reaction results in the of glucose.
formation of an azo dye. B. no other substances and higher concentrations of
24. Which of the following conditions most likely accounts glucose.
for a negative nitrite result on the reagent strip despite C. other reducing substances and lower concentrations
the presence of large quantities of bacteria? of glucose.
1. The bacteria present did not have enough time to D. no other substances and lower concentrations of
convert nitrate to nitrite. glucose.
2. The bacteria present are not capable of converting 30. Which of the following statements about glucose is
nitrate to nitrite. false?
3. The patient is not ingesting adequate amounts of A. Glucose readily passes the glomerular filtration
nitrate in the diet. barrier.
4. The urine is dilute and the level of nitrite present B. Glucose is reabsorbed passively in the proximal
is below the sensitivity of the test. tubule.
A. 1, 2, and 3 are correct. C. Glucosuria occurs when plasma glucose levels exceed
B. 1 and 3 are correct. 160 to 180 mg/dL.
C. 4 is correct. D. High plasma glucose concentrations are associ-
D. All are correct. ated with damage to the glomerular filtration barrier.

:
25. The chemical principle of the nitrite reagent pad is based 31. The pass-through phenomenon observed with the Clin-
on the itest method when large amounts of glucose are present
A. pseudoperoxidase activity of nitrite. in the urine is due to
B. diazotization of nitrite followed by an azo coupling A. “caramelization” of the sugar present.
reaction. B. reduction of copper sulfate to green-brown cupric
C. azo coupling action of nitrite with a diazonium salt to complexes.
form an azo dye. C. depletion of the substrate, that is, not enough copper
D. hydrolysis of an ester by nitrite combined with an azo sulfate is present initially.
coupling reaction. D. reoxidation of the cuprous oxide formed to cupric
26. Which of the following substances or actions can produce oxide and other cupric complexes.
false-positive nitrite results? 32. The glucose specificity of the double sequential enzyme
A. Ascorbic acid reaction used on reagent strip tests is due to the use of
B. Vaginal contamination A. gluconic acid.
C. Strong reducing agents B. glucose oxidase.
D. Improper specimen storage C. hydrogen peroxide.
27. A urine specimen is tested for glucose by a reagent strip D. peroxidase.
and by the Clinitest method. The reagent strip result is 33. Which of the following ketones is not detected by the
100 mg/dL, and the Clinitest result is 500 mg/dL. Which reagent strip or tablet test?
of the following statements would best account for this A. Acetone
discrepancy? B. Acetoacetate
A. The Clinitest tablets have expired or were stored C. Acetone and acetoacetate
improperly. D. β-Hydroxybutyrate
B. A large amount of ascorbic acid is present in the 34. Which of the following can cause false-positive ketone
specimen. results?
C. A strong oxidizing agent (e.g., bleach) is contaminat- A. A large amount of ascorbic acid in urine
ing the specimen. B. Improper storage of the urine specimen
D. The reagent strip is exhibiting the pass-through phe- C. Drugs containing free sulfhydryl groups
nomenon, which results in a falsely low value. D. A large amount of glucose (glucosuria)
122 CHAPTER 6 Chemical Examination of Urine

35. Which of the following will not cause ketonemia and 42. Urobilinogen is formed from the
ketonuria? A. conjugation of bilirubin in the liver.
A. An inability to use carbohydrates B. reduction of conjugated bilirubin in bile.
B. Inadequate intake of carbohydrates -reduction of bilirubin by intestinal bacteria.
C.
C. Increased metabolism of carbohydrates D. oxidation of urobilin by anaerobic intestinal bacteria.
D. Excessive loss of carbohydrates 43. Which of the following statements about urobilinogen is
36. The ketone reagent strip and tablet tests are based on the true?
reactivity of ketones with A. Urobilinogen is not normally present in urine.
A. ferric chloride. B. Urobilinogen excretion usually is decreased after
B. ferric nitrate. a meal.
C. nitroglycerin. C. Urobilinogen excretion is an indicator of renal
D. nitroprusside. function.
O
37. Which of the following statements about bilirubin is
true? urobilin. O
D. Urobilinogen is labile and readily photo-oxidizes to

A. Conjugated bilirubin is water insoluble. 44. The classic Ehrlich’s reaction is based on the reaction of
B. Bilirubin is a degradation product of heme cata- urobilinogen with
bolism. A. diazotized dichloroaniline.
C. Unconjugated bilirubin readily passes through the B. p-aminobenzoic acid.
glomerular filtration barrier. C. p-dichlorobenzene diazonium salt.
D. The liver conjugates bilirubin with albumin to form D. p-dimethylaminobenzaldehyde.
conjugated bilirubin. 45. Which of the available chemical principles is most spe-
38. The bilirubin reagent strip and tablet tests are based on cific for the detection of urobilinogen?
A. Ehrlich’s aldehyde reaction. A. Ictotest
B. the oxidation of bilirubin to biliverdin. B. Ehrlich’s reaction


C. the reduction of bilirubin to azobilirubin. C. Azo coupling reaction
O
D. the coupling of bilirubin with a diazonium salt.
39. Which of the following are characteristic urine findings
D. Double sequential enzyme reaction
46. Which of the following reagent strip tests can be affected
from a patient with hemolytic jaundice? by ascorbic acid, resulting in falsely low or false-negative
A. A positive test for bilirubin and an increased amount results?
of urobilinogen 1. Blood
"
B. A positive test for bilirubin and a decreased amount
of urobilinogen
2. Bilirubin
3. Glucose
BBG
C. A negative test for bilirubin and an increased amount 4. Nitrite
of urobilinogen A. 1, 2, and 3 are correct.
D. A negative test for bilirubin and a decreased amount B. 1 and 3 are correct.
of urobilinogen C. 4 is correct.
40. Which of the following results shows characteristic urine D. All are correct.
findings from a patient with an obstruction of the bile 47. Which of the following best describes the mechanism of
duct? ascorbic acid interference?
A. A positive test for bilirubin and an increased amount A. Ascorbic acid inhibits oxidation of the chromogen.

O
of urobilinogen B. Ascorbic acid inactivates a reactant, promoting color

-0
B. A positive test for bilirubin and a decreased amount
of urobilinogen
-

C. A negative test for bilirubin and an increased amount


development.
C. Ascorbic acid removes a reactant from the intended
reaction sequence.
of urobilinogen D. Ascorbic acid interacts with the reactants, producing
D. A negative test for bilirubin and a decreased amount a color that masks the results.
of urobilinogen
41. Which of the following conditions can result in false-
positive bilirubin results?
A. Elevated concentrations of nitrite
B. Improper storage of the specimen
C. Ingestion of ascorbic acid
D. Ingestion of certain medications
CHAPTER 6 Chemical Examination of Urine 123

Case 6.1
A 45-year-old woman with type 1 diabetes mellitus is admitted to 1. Circle any abnormal or discrepant urinalysis findings.
the hospital and has been given a preliminary diagnosis of the 2. Which substance most likely accounts for the large amount of
nephrotic syndrome. She has not been feeling well for the past white foam observed?
week and has bilateral pitting edema in her lower limbs. Her A. Fat
admission urinalysis results follow. B. Protein
C. Glucose
Results D. Blood/hemoglobin
Physical Examination Chemical Examination 3. Explain the most likely reason for the presence of increased
red blood cells in this patient’s urine.
Color: colorless SG: 1.010
4. Is the hemoglobin present (blood reaction: small) contributing
Clarity: clear pH: 5
to the protein test result? Explain. *
Odor: — Blood: small
5. If this patient has the nephrotic syndrome, the proteinuria in
Protein: 500 mg/dL
this patient should be classified as
☐Large amount of
white foam noted
LE: negative
Nitrite: negative
Glucose: 250 mg/dL
A. glomerular proteinuria.
B. tubular proteinuria.
C. overflow proteinuria.
Ketones: negative
D. postrenal proteinuria. Albumin
Bilirubin: negative
6. In progressive renal disease, when solute discrimination by
Urobilinogen: normal
the glomerular filtration barrier is lost, monitoring which pro-
tein is most useful in identifying these glomerular changes?
7. Why is glucose present in the urine of this patient? Explain
briefly.

LE, Leukocyte esterase.

Case 6.2
An 82-year-old woman was admitted to the hospital with back 1. Circle any abnormal or discrepant urinalysis findings.
and left rib pain. Radiographic examination revealed lytic lesions 2. Explain the most probable cause for the discrepancy between
of the lumbar vertebrae and ribs, and sheets of plasma cells the reagent pad test for protein and the urine total protein
were present on bone marrow biopsy. A diagnosis of multiple result.
myeloma is made. The result of a 24-hour urine total protein 3. Which protein(s) is most likely responsible for the proteinuria
was 535 mg/24 h (reference range: <150 mg/24 h). Her admis- in this patient?
sion urinalysis results follow. A. Albumin
B. Globulins
Results C. Hemoglobin
Physical Examination Chemical Examination D. Uromodulin
4. The proteinuria in this patient would be classified as
Color: yellow SG: 1.020
A. glomerular proteinuria.
-
Clarity: slightly cloudy pH: 5.5
B. tubular proteinuria.
Odor: — Blood: negative
C. overflow proteinuria.
_ Protein: trace
D. postrenal proteinuria.
LE: negative
Nitrite: negative
Glucose: negative
Ketone: negative
Bilirubin: negative
Urobilinogen: normal

LE, Leukocyte esterase.


124 CHAPTER 6 Chemical Examination of Urine

Case 6.3
A 51-year-old woman is admitted to the hospital for a vaginal hys- 1. Circle any abnormal or discrepant urinalysis findings.
terectomy. During surgery, she is placed in Simon’s position 2. In this specimen, which substance most likely is causing the
(exaggerated lithotomy position) for 6 hours because of surgical urine to appear brown?
complications. She receives 2 units of packed red blood cells A. Bilirubin
after surgery. Twenty-four hours after surgery, a routine urinaly- B. Hemoglobin
sis, hemoglobin, hematocrit, and various chemistry tests are C. Myoglobin
performed. The chemistry and urinalysis results follow. D. A drug the patient is taking
3. Which of the following substances is causing the blood reac-
Serum Chemistry Results tion to be large?
Test Result Reference Range A. Ascorbic acid
B. Bilirubin
Creatine kinase: 5800 U/L 10-130 U/L
C. Hemoglobin
Myoglobin: 400 U/L <120 U/L Albumin
D. Myoglobin
Haptoglobin: 175 mg/dL 83-267 mg/dL
4. What protein is responsible for the trace strip result? Explain.
Urine Results 5. Suggest a cause for the myoglobinuria.
Physical Examination Chemical Examination

Color: brown SG: 1.015
Clarity: clear pH: 5.5

:
Odor: — Blood: large
Protein: trace
LE: negative
Nitrite: negative
Glucose: negative
Ketones: negative
Bilirubin: negative
Urobilinogen: normal

LE, Leukocyte esterase.

Case 6.4
A 26-year-old man is seen by his physician and reports sudden 1. Circle any abnormal or discrepant urinalysis findings.
weight loss, polydipsia, and polyuria. A routine urinalysis and 2. Explain the pass-through effect exhibited by the Clinitest
plasma glucose level are obtained. The patient was fasting method in this patient.
before blood collection. 3. What is the concern about observing the pass-through effect?
4. Is this patient showing any signs of renal damage or dysfunc-
Chemistry Results tion? Yes No
Plasma glucose: 230 mg/dL (reference ranges: fasting !110 mg/ 5. Select the diagnosis that best accounts for the glucosuria
dL; diabetic "126 mg/dL) observed in this patient.
A. Normal; glucose renal threshold exceeded
Results B. Type 1 diabetes mellitus
Physical Confirmatory C. Type 2 diabetes mellitus
Examination Chemical Examination Tests D. Impaired glucose tolerance
Color: colorless SG: 1.010 Refractometer: 6. Explain why the reagent strip ketone test is positive.
Clarity: clear pH: 5.5 1.029 7. Explain the two different specific gravity results obtained.
Odor: sweet Blood: negative Which result most accurately reflects the ability of the kidneys
(subtle) Protein: negative to concentrate renal solutes (i.e., renal concentrating ability)?
LE: negative
Nitrite: negative
Glucose: >2000 Clinitest (2-drop):
"5000
Ketones: small Clinitest (5-drop):
>2000*
Bilirubin: negative
Urobilinogen: normal

LE, Leukocyte esterase.


*The pass-through effect was noted during performance of this test.
CHAPTER 6 Chemical Examination of Urine 125

Case 6.5
A 36-year-old man sees his doctor and reports fatigue, nausea, 1. Circle any abnormal or discrepant urinalysis findings.
and concern about a yellowish discoloration in the sclera of his 2. What substance most likely accounts for the urine color and
eyes. Physical examination reveals a tender liver. The following foam color observations? Bilirubin
urinalysis results are obtained. 3. Why is the reagent strip bilirubin test negative, whereas the
Ictotest is positive?
Results 4. Should the bilirubin on this urine be reported as negative or
Physical Confirmatory positive?
Examination Chemical Examination Tests 5. Explain the physiologic process that accounts for the bilirubin
in this urine.


Color: amber SG: 1.015
6. What form of bilirubin is present in this urine: unconjugated or
Clarity: slightly pH: 6.5
conjugated? conjugated bilirubin
cloudy Blood: negative
7. Why is the urobilinogen normal and not increased?
Odor: — • Protein: trace

• Yellow coloration
of foam noted
LE: negative
Nitrite: negative
Glucose: negative
Ketones: negative
Bilirubin: negative -
Ictotest: positive
Urobilinogen: normal

LE, Leukocyte esterase.


7
Microscopic Examination of Urine Sediment

LEARNING OBJECTIVES
After studying this chapter, the student should be able to: casts with the physical and chemical examination
1. Discuss the importance of standardizing the microscopic of urine.
examination of urine and describe how this standar- 8. Describe the development of urinary crystals, including
dization is achieved in the clinical laboratory. at least three factors that influence their formation.
2. Describe microscopic and staining techniques used to 9. Describe the characteristic form of each major type of
enhance visualization of the formed elements in urinary urinary crystal; categorize each crystal type as being
sediment. found in acid, neutral, or alkaline urine; and discuss the
3. Describe the microscopic appearance and clinical clinical significance of each crystal type.
significance of erythrocytes and leukocytes in urine and 10. Identify the following formed elements found in urine
correlate their presence with the physical and chemical sediment, and discuss their clinical significance:
examination of urine. • Bacteria
4. Describe the microscopic characteristics and location of • Clue cells
each type of epithelium found in the urinary tract, that is, • Fat
squamous, transitional, and renal tubular epithelium • Fecal contaminants
(proximal, distal, and collecting duct). • Fibers
5. Summarize briefly the clinical significance of increased • Hemosiderin
sloughing of the urinary tract epithelium. • Mucus threads
6. Describe the formation, composition, and clinical • Parasites
significance of urinary cast formation. • Spermatozoa
7. State the categories into which casts are classified, • Starch
discuss the clinical circumstances that result in the • Trichomonads
formation of each cast type, and correlate the presence of • Yeast

CHAPTER OUTLINE
Standardization of Sediment Preparation, 127 Cytocentrifugation, 135
Commercial Systems, 127 Cytodiagnostic Urinalysis, 135
Specimen Volume, 128 Formed Elements in Urine Sediment, 135
Centrifugation, 128 Blood Cells, 136
Sediment Concentration, 129 Epithelial Cells, 143
Volume of Sediment Viewed, 129 Casts, 149
Reporting Formats, 130 Crystals, 159
Enhancing Urine Sediment Visualization, 131 Microorganisms in Urine Sediment, 172
Staining Techniques, 131 Miscellaneous Formed Elements, 176
Microscopy Techniques, 134 Contaminants, 180
Cytocentrifugation and Cytodiagnostic Correlation of Urine Sediment Findings With Disease, 181
Urinalysis, 135

K E Y T E R M S*
casts distal convoluted tubular cells
clue cells KOH preparation
collecting duct cells lipiduria
crystals Maltese cross pattern
cytocentrifugation oval fat bodies
cystogram proximal convoluted tubular cells

126
184 CHAPTER 7 Microscopic Examination of Urine Sediment

STUDY QUESTIONS
1. Which of the following are not standardized when com- 7. Hemoglobin is a protein and will
mercial systems are used for the processing and micro- A. not react in the protein reagent strip test.
scopic examination of urine sediment? B. interfere with the protein reagent strip test, produc-
A. Microscopic variables, such as the number of focal ing erroneous results.
planes C. always contribute to the protein reagent strip result,
B. The concentration and volume of the urine sediment regardless of the amount of hemoglobin present.
prepared D. contribute to the protein reagent strip result only
C. The volume of the urine sediment dispensed for when large concentrations of hemoglobin are present.
microscopic viewing 8. Which urinary sediment component(s) when observed
D. Identification and enumeration of formed elements microscopically can resemble red blood cells?
in the urine sediment 1. Yeasts
2. When urine sediment is viewed, stains and various 2. Air bubbles
microscopic techniques are used to 3. Oil droplets
1. enhance the observation of fine detail. 4. Calcium oxalate monohydrate crystals
2. confirm the identity of suspected components. A. 1, 2, and 3 are correct.
3. differentiate formed elements that look alike. B. 1 and 3 are correct.
4. facilitate the visualization of low-refractile C. 4 is correct.
components. D. All are correct.
A. 1, 2, and 3 are correct. 9. Which of the following is not a characteristic of neutro-
B. 1 and 3 are correct. phils found in the urine sediment?
C. 4 is correct. A. They are approximately 10 to 14 μm in diameter.
D. All are correct. B. They form “ghost cells” in hypotonic urine.
3. The microscopic identification of hemosiderin is C. They shrink in hypertonic urine but do not crenate.
enhanced when the urine sediment is stained with D. As they disintegrate, vacuoles and blebs form and
A. Gram stain. their nuclei fuse.
B. Hansel stain. 10. How do increased numbers of leukocytes usually get into
C. Prussian blue stain. the urine?
D. Sudan III stain. A. Through a renal bleed
4. When the laboratorian performs the microscopic B. By passive movement through pores in the vascular
examination of urine sediment, which of the following epithelium
are enumerated using low-power magnification? C. By active ameboid movement through tissues and
-
A. Bacteria epithelium
B. Casts D. Through damage to the integrity of the normal vas-
C. Red blood cells cular barrier
D. Renal tubular cells 11. Which statement regarding lymphocytes found in urine
5. A urine sediment could have which of the following sediment is correct?
formed elements and still be considered “normal”? A. They are not normally present in the urine.
A. Two or fewer hyaline casts B. They produce a positive leukocyte esterase test.
B. Five to 10 red blood cells C. Their number is increased in patients with drug
C. A few bacteria hypersensitivity.
D. A few yeast cells D. Their number is increased in patients experiencing
6. Which of the following statements about red blood cells kidney transplant rejection.
in urine is true? 12. Which of the following urinary tract structures is not
A. Red blood cells crenate in hypotonic urine. lined with transitional epithelium?
B. Red blood cell remnants are called “ghost cells.” A. Bladder
C. Alkaline and hypotonic urine promotes red blood cell B. Nephrons
disintegration. C. Renal pelves
D. Dysmorphic red blood cells often are associated with D. Ureters
renal tubular disease.
CHAPTER 7 Microscopic Examination of Urine Sediment 185

13. Match the number of the epithelial cell type with its char- 19. When the laboratorian is using brightfield microscopy, a
acteristic feature. Only one type is correct for each urinary cast that appears homogeneous with well-defined
feature. edges, blunt ends, and cracks is most likely a
A. fatty cast.
Characteristic Feature Epithelial Cell Type B. granular cast.
4 A. Large and irregularly
__ 1. Collecting tubular cell C. hyaline cast.
angled (flagstone); can
be anucleated
2.
3.
Distal tubular cell
Proximal tubular cell
-D. waxy cast.
20. All of the following can be found incorporated into a cast
3 B. Oblong or cigar-
__ 4. Squamous epithelial
matrix except
shaped; small cell centered or slightly
eccentric nucleus eccentric
A. bacteria.
B. crystals.
I C. Polygonal; large
__
nucleus
5. Transitional
epithelial cell -C. transitional epithelial cells.
TEE
2 D. Oval to round;
__ D. white blood cells.
small nucleus 21. Which of the following urinary casts are diagnostic of
that is centered or glomerular or renal tubular damage?
slightly eccentric A. Bacterial casts
5
__ E. Round, pear-
0 B. Red blood cell casts -
Are now
shaped, or C. Renal tubular cell casts eye rare
columnar with a D. White blood cell casts
small oval to round
22. Which of the following characteristics best differentiates
nucleus
waxy casts from fibers that may contaminate urine
14. Which of the following can be observed in the urine sediment?
sediment as an intact fragment or sheet of cells? A. Waxy casts do not polarize light; fibers do.
B. Waxy casts are more refractile than fibers.

=
1. Collecting tubular epithelium
2. Distal tubular epithelium C. Waxy casts have rounded ends; fibers do not.
3. Transitional epithelium D. Waxy casts are thicker at their margins; fibers are
4. Proximal tubular epithelium thicker in the middle.
A. 1, 2, and 3 are correct. 23. Which of the following does not affect the formation of
B. 1 and 3 are correct. urinary crystals within nephrons?
C. 4 is correct. A. The pH of the ultrafiltrate
D. All are correct. B. The diameter of the tubular lumen
15. Urinary casts are formed in C. The flow of urine through the tubules
A. the distal and collecting tubules. D. The concentration of solutes in the ultrafiltrate
B. the distal tubules and the loops of Henle. 24. The formation of urinary crystals is associated with a spe-
C. the proximal and distal tubules. cific urine pH. Match the urine pH that facilitates crys-
D. the proximal tubules and the loops of Henle. talline formation with the appropriate crystal type.
16. Urinary casts are formed with a core matrix of More than one number (pH) can be used.
A. albumin.
Crystal Type Urine pH
B. Bence Jones protein.

=
C. transferrin. __
3 A. Ammonium biurate 1. Acid
__ B. Amorphous urates
1121 2. Neutral
◦D. uromodulin.
17. Which of the following does not contribute to the size,
312__) C. Amorphous phosphates 3. Alkaline
__ D. Calcium oxalate
1. 2,3
shape, or length of a urinary cast? __
I E. Cholesterol
A. The concentration of protein in the core matrix of __) F. Cystine
112,3
the cast __
I
G. Radiographic contrast media
B. The configuration of the tubule in which the cast __1 H. Sulfonamides

is formed 312__) I. Triple phosphate


-

C. The diameter of the tubular lumen in which the cast __


I J. Tyrosine
is formed __
1
K. Uric acid
D. The duration of time the cast is allowed to form in the DM

"¥↑;;§¥hr
tubule "
18. All of the following enhance urinary cast formation except '
A. an alkaline pH.
B. urinary stasis. ma
C. an increase in the solute concentration of the
ultrafiltrate.
D. an increase in the quantity of plasma proteins in the
ultrafiltrate.
186 CHAPTER 7 Microscopic Examination of Urine Sediment

25. Match the crystal composition with the microscopic 30. Fat can be found in the urine sediment in all of the fol-
description that best characterizes it. lowing forms except
A. within casts.
Microscopic Description Crystal Composition B. within cells.
__
8 A. Colorless “coffin lid” form 1. Ammonium biurate C. as free-floating droplets.
__
6 B. Colorless hexagonal plates

__
14 C. Colorless “envelope” form
2. Amorphous urates
3. Amorphous
☆D. within hemosiderin granules.
31. Which of the following statements regarding the charac-
__
5 D. Colorless parallelogram- phosphates
teristics of urinary fat is true?
shaped plates with 4. Calcium oxalate
notched corners 5. Cholesterol
A. Cholesterol droplets stain with Sudan III stain.
I
__ E. Yellow-brown “thorny 6. Cystine •B. Triglyceride (neutral fat) stains with oil red O stain.
apple” form 7. Sulfonamides C. Cholesterol droplets do not form a Maltese cross pat-
__
9 F. Colorless to yellow; 8. Triple phosphate tern under polarized light.
diamond-shaped or 9. Uric acid D. Triglycerides (neutral fat) are anisotropic and form
rhombic; can form layers a Maltese cross pattern under polarized light.
7- G. Yellow-brown sheaves of
__ 32. Which of the following statements regarding the micro-
wheat scopic examination of urine sediment is false?
A. If large numbers of leukocytes are present microscop-
26. Which of the following crystals, when found in the urine
ically, then bacteria are present.
condition? 0
sediment, most likely indicates an abnormal metabolic
B. If urinary fat is present microscopically, then the
chemical test for protein should be positive.
✓ A. Bilirubin
C. If large numbers of casts are present microscopically,
B. Sulfonamides
then the chemical test for protein should be positive.
C. Triple phosphate
D. If large numbers of red blood cells are present micro-
D. Uric acid
scopically, then the chemical test for blood should be
27. During the microscopic examination of a urine sediment,
positive.
cystine crystals are found. The laboratorian should per-
33. The following are initial results obtained during a routine
form which of the following before reporting the pres-
urinalysis. Which results should be investigated further?
ence of these crystals?
A. Negative protein; 2 to 5 waxy casts

I
1. Perform a confirmatory chemical test
B. Cloudy brown urine; 2 to 5 red blood cells
2. Ensure that the urine specimen has an acid pH
C. Urine pH 7.5; ammonium biurate crystals
3. Assess the number of crystals per high-power
D. Clear, colorless urine; specific gravity 1.010
field
34. The following are initial results obtained during a routine
4. Check the current medications that the patient is
urinalysis. Which results should be investigated further?
taking
A. Negative protein; 0 to 2 hyaline casts
A. 1, 2, and 3 are correct.
B. Urine pH 6.0; calcium oxalate crystals
B. 1 and 3 are correct.
C. Cloudy yellow urine; specific gravity 1.050
C. 4 is correct.
D. Amber urine with yellow foam; negative bilirubin by
D. All are correct.
reagent strip; positive Ictotest
28. Mucus threads can be difficult to differentiate from
35. Which of the following when found in the urine sediment
A. fibers.
•B. hyaline casts.
C. pigmented casts.
contaminant? 0
from a female patient is not considered a vaginal

A. Fat
D. waxy casts.
B. Clue cells
29. Which of the following is not a distinguishing character-
C. Spermatozoa
istic of yeast in the urine sediment?
0
D. Trichomonads
A. Motility
B. Budding forms
C. Hyphae formation
D. Colorless ovoid forms
CHAPTER 7 Microscopic Examination of Urine Sediment 187

Case 7.1
A routine urinalysis specimen is sent to the laboratory from a 1. List any abnormal or discrepant urinalysis findings.
patient suspected of having renal calculi. When the microscopic 2. What is the most likely identity of this crystal?
examination is performed, unusual crystals that resemble cho- A. Cystine
lesterol plates are observed. The technologist is suspicious B. Cholesterol
and performs a sulfosalicylic acid precipitation test (SSA test C. Triple phosphate
for protein, see Appendix E) and checks the specific gravity by D. Uric acid, rare form
refractometry. The patient care unit is contacted for a list of cur- E. Radiographic contrast media
rent medications. The list reveals that the patient had an intrave- 3. State two results that support the crystal selection made in
nous pyelogram 6 hours earlier. The patient is taking no other question 2.
medications except those given during the intravenous pyelo- 4. Which chemical examination result does not support the pres-
gram procedure (Demerol and Xylocaine). ence of lipids in the urine? Explain.
5. Which specific gravity result best indicates this patient’s renal
ability to concentrate urine?
A. Reagent strip result: 1.020
B. Refractometer result: > 1.035

Results

Physical Examination Chemical Examination Microscopic Examination


Color: yellow SG: 1.020 RBC/hpf: 0–2
-
Clarity: cloudy Refractometry: >1.035 WBC/hpf: 0–2
Odor: — pH: 5.0 Casts: negative
Blood: negative Epithelials: few TE cells/hpf
Protein: negative
(SSA: 4 + [crystalline precipitate])
_ Crystals: moderate; type unknown per hpf

LE: negative
Nitrite: negative
Glucose: negative
Ketones: negative
Bilirubin: negative
Urobilinogen: normal

hpf, High-power field; LE, leukocyte esterase; RBC, red blood cell; TE, transitional epithelial; WBC, white blood cell.
188 CHAPTER 7 Microscopic Examination of Urine Sediment

Case 7.2
A 22-year-old woman is seen in the emergency department. She 1. List any abnormal or discrepant urinalysis findings.
complains of a painful burning sensation (dysuria) when urinat- 2. Based on the patient’s symptoms and the urinalysis results,
ing. She also states that she feels as if she has “to go” all the select the most probable diagnosis.
time. A midstream clean catch urine specimen is collected for A. Normal urinalysis
a routine urinalysis and culture. B. Urinary tract infection
C. Acute glomerulonephritis
D. Nephrotic syndrome
3. Assume that no patient information is available and that the
number of squamous epithelial cells observed microscopically
was “many.” Would your suspected diagnosis change?
4. State three reasons why the nitrite test can be negative
despite bacteriuria.
5. State two reasons why the leukocyte esterase test can be
negative despite increased numbers of white blood cells in
the urine sediment.
6. Suggest a cause for the increased number of transitional epi-
thelial cells observed in the urine sediment.
Results
Physical Examination Chemical Examination Microscopic Examination
Color: yellow SG: 1.015 RBC/hpf: 0-2
-

f-
Clarity: cloudy pH: 6.0 WBC/hpf: 10-25
Odor: — Blood: trace Casts/hpf: 2-5 hyaline

Protein: trace Epithelials: few SE cells/lpf; moderate TE cells/hpf
LE: negative Bacteria: moderate/hpf
Nitrite: negative
Glucose: negative
Ketones: negative
Bilirubin: negative
Urobilinogen: normal
hpf, High-power field; LE, leukocyte esterase; lpf, low-power field; RBC, red blood cell; SE, squamous epithelial; TE, transitional epithelial;
WBC, white blood cell.
CHAPTER 7 Microscopic Examination of Urine Sediment 189

Case 7.3
A 36-year-old man with a history of diabetes mellitus is admitted 1. List any abnormal or discrepant urinalysis findings.
to the hospital with the following complaints: decreased fre- 2. Based on the patient’s symptoms and urinalysis results,
quency of urination, constant “bloated” feeling, weight gain, select the most probable diagnosis:
puffy eyes in the morning, and scrotal swelling. Mild edema of A. Acute pyelonephritis
the ankles, abdomen, and eyes is also noted. Routine chemistry B. Nephrotic syndrome
tests reveal hypoalbuminemia and hyperlipidemia (" triglycer- C. Acute glomerulonephritis
ides and " cholesterol). Urinalysis results follow. D. Lipiduria of unknown cause
3. The proteinuria in this patient should be classified as
A. glomerular proteinuria.
B. tubular proteinuria.
C. overflow proteinuria.
D. postrenal proteinuria.
4. What substance is responsible for the large amount of white
foam observed?
A. Fat
B. Protein
C. Glucose
D. Casts
5. Explain the physiologic processes responsible for the edema
exhibited in this patient.
6. In progressive renal disease with loss of glomerular filtering
ability, which plasma protein is usually lost first? Explain why.
7. Explain the physiologic process responsible for the glucose
present in this patient’s urine.
Results 8. Why is the ketone test not positive?

Physical Examination Chemical Examination Microscopic Examination


Color: yellow SG: 1.015 °
RBC/hpf: 2-5
Clarity: slightly pH: 5.0 WBC/hpf: 0-2
cloudy Blood: small Casts/lpf: 2-5 hyaline; 0-2 fatty; 0-2 waxy
Odor: — Protein: 2000 mg/dL Epithelials: few TE cells/hpf; few OFBs/hpf
Large amount of white LE: negative bodies
foam noted. Nitrite: negative
Glucose: 100 mg/dL
Ketones: negative
Bilirubin: negative
Urobilinogen: normal

hpf, High-power field; LE, leukocyte esterase; lpf, low-power field; OFBs, oval fat bodies; RBC, red blood cell; TE, transitional epithelial; WBC, white
blood cell.
190 CHAPTER 7 Microscopic Examination of Urine Sediment

Case 7.4
A 30-year-old man is admitted to the hospital with headache, 1. List any abnormal or discrepant urinalysis findings.
anorexia, and passing red-colored urine. Examination reveals 2. What is the most likely process by which red blood cells are
mild edema of the eyes and mild hypertension. Medical history getting into this patient’s urine?
reveals that the patient’s daughter had strep throat about a 3. At this level of hematuria, is the blood that is present most
month ago and was treated successfully. Subsequently, he likely contributing to the reagent strip protein result?
developed a sore throat that lasted a few days but did not seek A. Yes
treatment. Urinalysis results follow. B. No
4. This patient is determined to have acute glomerulonephritis.
Based on this diagnosis, the proteinuria in this patient would
be classified as
A. glomerular proteinuria.
B. tubular proteinuria.
C. overflow proteinuria.
D. postrenal proteinuria.
5. Of the microscopic findings, which sediment entity specifi-
cally indicates adverse glomerular changes and the presence
of a renal disorder? RBC casts

Results

Physical Examination Chemical Examination Microscopic Examination



← Color: red
Clarity: cloudy
SG: 1.010
pH: 5.5
RBC/hpf: 25-50
WBC/hpf: 0-2
_
Odor: —
← Blood: moderate
Protein: 300 mg/dL
Casts/lpf: 2-5 hyaline; 0-2 RBCs; 0-2 granular
Epithelials: few TE cells/hpf
LE: negative Bacteria: negative
Nitrite: negative
Glucose: negative
Ketones: negative
Bilirubin: negative
Urobilinogen: normal

hpf, High-power field; lpf, low-power field; RBC, red blood cell; TE, transitional epithelial; WBC, white blood cell.

Case 7.5
An obese 58-year-old woman is seen by her physician. Her chief 1. List any abnormal or discrepant urinalysis findings.
complaint is perineal itching and soreness. When giving her 2. What is the most likely cause of this patient’s vaginitis? vaginal yeast infection
health history, she also complains of being thirsty “all the time” 3. What is the most likely origin or source of the white blood cells
and of urinating more frequently. On pelvic examination, a white in this urine? vaginal fluid contamination
discharge is noted. A urine specimen is collected for a routine 4. Which two microscopic findings suggest that this urine is not
urinalysis. a midstream clean catch specimen? Yeast .at many SE cells
5. Is this patient showing signs/symptoms of any urinary tract
disorder or dysfunction? Yes No
6. Explain the physiologic processes responsible for ketonuria.
Results
Physical Examination Chemical Examination Microscopic Examination
Color: yellow SG: 1.015 RBC/hpf: 0-2

€imen
- Clarity: cloudy
Odor: —
pH: 6.0
Blood: moderate
WBC/hpf: 10-25; clumps
Casts/lpf: 0-2 hyaline
Protein: trace Epithelials: many SE cells/lpf


LE: positive Bacteria: negative is not a clean catch
Nitrite: negative Yeast/hpf: moderate
Glucose: 500 mg/dL Crystals/hpf: few urates
Ketones: trace
Bilirubin: negative
Urobilinogen: normal

hpf, High-power field; LE, leukocyte esterase; lpf, low-power field; RBC, red blood cell; SE, squamous epithelial; WBC, white blood cell.
CHAPTER 7 Microscopic Examination of Urine Sediment 191

Case 7.6
A 48-year-old woman is admitted to a hospital for an emergency 1. List any abnormal or discrepant urinalysis findings.
appendectomy. Because of bleeding complications, she recei- 2. What is hemosiderin?
ves a unit of packed red blood cells after surgery. Two hours 3. Explain how hemosiderin gets into the urine sediment.
later, she develops fever, chills, and nausea. Two days after sur- 4. What substance most likely is causing the brown color
gery, a routine urinalysis and hemosiderin test (Rous test) are observed in this urine?
performed, and the following results are obtained. 5. Explain the physiologic mechanism that leads to increased
urobilinogen in this patient’s urine.
6. Because of the intravascular hemolytic episode experienced
by this patient, her serum bilirubin is significantly increased.
Why is her urine bilirubin still normal?
Results

Physical Examination Chemical Examination Microscopic Examination

⑤ Color: brown
Clarity: slightly cloudy
SG: 1.015
pH: 5.0
RBC/hpf: 0-2
WBC/hpf: 0-2
Odor: —
§ Blood: large
Protein: trace
↑-Casts/lpf: 5-10 granular
Epithelials: few TE cells/hpf; few RTE cells/hpf; few SE cells/hpf
LE: negative Crystals/hpf: few amorphous urates
Nitrite: negative Hemosiderin test: positive
Glucose: negative
Ketones: negative
Bilirubin: negative
my
Urobilinogen: 4 mg/dL

hpf, High-power field; LE, leukocyte esterase; lpf, low-power field; RBC, red blood cell; RTE, renal tubular epithelial; SE, squamous epithelial; TE,
transitional epithelial; WBC, white blood cell.

Case 7.7
A 19-year-old female college athlete visits the campus health The technologist performing the microscopic examination
clinic for a mandatory sports physical. She claims no health prob- checks the pH of the urine sediment and rechecks that of the
lems and is taking no medications. A clean catch urine specimen specimen in the urine cup. The following results are obtained:
is collected for a routine urinalysis, and the following results are Urine sediment: pH 5.0
obtained. Urine in specimen cup: pH 7.0
1. List any abnormal or discrepant urinalysis findings.
2. Suggest a cause for the discrepancies observed.

Results

Physical Examination Chemical Examination Microscopic Examination


Color: yellow SG: 1.020 RBC/hpf: 50-100
Clarity: clear pH: 7.0 WBC/hpf: 25-50;


Odor: — Blood: trace clumps
Protein: trace Casts/lpf: 5-10
LE: negative cellular
Nitrite: negative Epithelials: few RTE
Glucose: negative cells/hpf; few
Ketones: negative SE cells/lpf
Bilirubin: negative Crystals/hpf: few uric acid
Urobilinogen: normal
Ascorbic acid: negative

hpf, High-power field; lpf, low-power field; RBC, red blood cell; RTE, renal tubular epithelial; SE, squamous epithelial; WBC, white blood cell.
8
Renal and Metabolic Disease

LEARNING OBJECTIVES
After studying this chapter, the student should be able to: 8. Compare and contrast the causes, clinical features,
1. Discuss the pathogenesis of glomerular damage and and typical urinalysis findings in the following
describe four morphologic changes that occur in tubulointerstitial diseases and urinary tract
glomeruli. infections:
2. Describe the clinical features associated with glomerular • Acute and chronic pyelonephritis
disease and discuss factors that affect the degree to which • Acute interstitial nephritis
they are present. • Lower urinary tract infections
3. Describe briefly the morphologic appearances of the • Yeast infections
glomeruli, the mechanisms of glomerular damage, and the 9. Describe briefly the effects of vascular disease on renal
clinical presentations of the following glomerular diseases: function.
• Acute glomerulonephritis 10. Compare and contrast the causes and clinical features of
• Chronic glomerulonephritis acute and chronic renal failure.
• Rapidly progressive glomerulonephritis 11. Summarize the pathogenesis of calculus formation.
• Focal proliferative glomerulonephritis Discuss four factors that influence the formation of
• Focal segmental glomerulosclerosis urinary tract calculi, and briefly review current treatment
• IgA nephropathy options.
• Membranoproliferative glomerulonephritis 12. Describe briefly the physiologic mechanisms, clinical
• Membranous glomerulonephritis features, and roles of the urinalysis laboratory in the
• Minimal change disease diagnosis of the following amino acid disorders:
4. Describe the pathologic mechanisms of glomerular • Cystinuria and cystinosis
damage in the following systemic diseases: • Homogentisic acid (alkaptonuria)
• Systemic lupus erythematosus • Maple syrup urine disease
• Diabetes mellitus • Phenylketonuria
• Amyloidosis • Tyrosinuria and melanuria
5. State at least five clinical features that characterize the 13. Describe briefly the physiologic mechanisms, clinical
nephrotic syndrome and identify diseases that are features, and typical urinalysis findings in the following
associated with this syndrome. carbohydrate disorders:
6. Differentiate between ischemic and toxic acute • Glucosuria
tubular necrosis and discuss the clinical presentation • Diabetes mellitus
and urinalysis findings associated with this disease. • Galactosuria
7. Describe the renal dysfunction and clinical features of the 14. Describe briefly the physiologic mechanisms, clinical
following renal tubular disorders: features, and typical urinalysis findings in the following
• Cystinosis metabolic disorders:
• Cystinuria • Diabetes insipidus
• Fanconi’s syndrome • Porphyrias
• Renal glucosuria 15. Discuss the formation of porphobilinogen and its clinical
• Renal phosphaturia significance.
• Renal tubular acidosis

211
CHAPTER 8 Renal and Metabolic Disease 235

5. Cotran RS, Leaf A: Renal pathophysiology, ed 3, New York, 10. National newborn screening and global resource center
1985, Oxford University Press. (NNSGRC): National newborn screening status report (website):
6. Sharfuddin AA, Molitoris BA: Pathophysiology of acute kidney http://genes-r-us.uthscsa.edu/sites/genes-r-us/files/
injury. In Alpern RJ, Hebert SC, editors: The kidney, ed 4, nbsdisorders.pdf. Accessed July 16, 2015.
Amsterdam, 2008, Elsevier Inc. 11. Pass KA, Lane PA, Fernhoff PM, et al: US newborn screening
7. Khan SR, Glenton PA, Backov R, Talham DR: Presence of lipids system guidelines II: Follow-up of children, diagnosis,
in urine, crystals and stones: implications for the formation of management, and evaluation; statement of the Council of
kidney stones. Kidney Int 62:2062, 2002. Regional Networks for Genetic Services (CORN). J Pediatr 137:
8. Coe FL, Parks JH, Evan A, Worcester E: Pathogenesis and S1–S46, 2000.
treatment of nephrolithiasis. In Alpern RJ, Hebert SC, editors: 12. Gahl WA, Thoene JG, Schneider JA. Cystinosis, N Engl J Med
The kidney, ed 4, Amsterdam, 2008, Elsevier Inc. 347:111, 2002.
9. Hruska K: Renal calculi (nephrolithiasis). In Goldman L, 13. Sherwin RS: Diabetes mellitus. In Goldman L, Bennett JC,
Bennett JC, editors: Cecil textbook of medicine, ed 21, editors: Cecil textbook of medicine, ed 21, Philadelphia, 2000,
Philadelphia, 2000, WB Saunders. WB Saunders.

STUDY QUESTIONS
1. Which of the following statements about renal diseases is 7. Which of the following disorders is characterized by
true? cellular proliferation into Bowman’s space to form cellu-
A. Glomerular renal diseases are usually immune lar “crescents”?
mediated. A. Chronic glomerulonephritis
B. Vascular disorders induce renal disease by increasing B. Membranous glomerulonephritis
renal perfusion. C. Minimal change disease
C. All structural components of the kidney are equally D. Rapidly progressive glomerulonephritis
susceptible to disease. 8. Which of the following disorders is the major cause of the
D. Tubulointerstitial renal diseases usually result from nephrotic syndrome in adults?
antibody-antigen and complement interactions. A. IgA nephropathy
2. In glomerular diseases, morphologic changes in the glo- B. Membranoproliferative glomerulonephritis
meruli include all of the following except C. Membranous glomerulonephritis
A. cellular proliferation. D. Rapidly progressive glomerulonephritis
B. erythrocyte congestion. 9. Which of the following glomerular diseases is the major
C. leukocyte infiltration. cause of the nephrotic syndrome in children?
D. glomerular basement membrane thickening. A. IgA nephropathy
3. When all renal glomeruli are affected by a morphologic B. Minimal change disease
change, this change is described as C. Membranous glomerulonephritis
A. diffuse. D. Rapidly progressive glomerulonephritis
B. focal. 10. Which of the following statements regarding IgA
C. differentiated. nephropathy is true?
D. segmental. A. It often follows a mucosal infection.
4. In glomerular renal disease, glomerular damage results B. It is associated with the nephrotic syndrome.
from C. It is characterized by leukocyte infiltration of the
A. deposition of infectious agents. glomeruli.
B. a decrease in glomerular perfusion. D. It often occurs secondary to systemic lupus
C. changes in glomerular hemodynamics. erythematosus.
D. toxic substances induced by immune complex 11. Eighty percent of patients who develop chronic
formation. glomerulonephritis previously had some type of glomer-
5. Clinical features that are characteristic of glomerular ular disease. Which of the following disorders is impli-
damage include all of the following except cated most frequently in the development of chronic
A. edema. glomerulonephritis?
B. hematuria. A. IgA nephropathy
C. proteinuria. B. Membranous glomerulonephritis
D. polyuria. C. Poststreptococcal glomerulonephritis
6. Which of the following disorders frequently occurs after a D. Rapidly progressive glomerulonephritis
bacterial infection of the skin or throat? 12. Chronic renal failure often develops in each of the follow-
A. Acute glomerulonephritis ing diseases except
B. Chronic glomerulonephritis A. amyloidosis.
C. Membranous glomerulonephritis B. diabetes mellitus.
D. Rapidly progressive glomerulonephritis C. diabetes insipidus.
D. systemic lupus erythematosus.
236 CHAPTER 8 Renal and Metabolic Disease

13. Which of the following features characterize(s) the 21. Most urinary tract infections are caused by
nephrotic syndrome? A. yeast, such as Candida spp.
1. Proteinuria B. gram-negative rods.
2. Edema C. gram-positive rods.
3. Hypoalbuminemia D. gram-positive cocci.
4. Hyperlipidemia 22. Which of the following formed elements when present in
A. 1, 2, and 3 are correct. urine sediment is most indicative of an upper urinary
B. 1 and 3 are correct. tract infection?
C. 4 is correct. A. Bacteria
D. All are correct. B. Casts
14. When a patient has the nephrotic syndrome, microscopic C. Erythrocytes
examination of the urine sediment often reveals D. Leukocytes
A. granular casts. 23. The most common cause of chronic pyelonephritis is
B. leukocyte casts. A. cystitis.
C. red blood cell casts. B. bacterial sepsis.
D. waxy casts. C. drug-induced nephropathies.
15. Which of the following has not been associated with D. reflux nephropathies.
acute tubular necrosis? 24. Eosinophiluria, fever, and skin rash are characteristic
A. Antibiotics clinical features of
B. Galactosuria A. acute pyelonephritis.
C. Hemoglobinuria B. acute interstitial nephritis.
D. Surgical procedures C. acute glomerulonephritis.
16. Which formed element in urine sediment is characteristic D. chronic glomerulonephritis.
of toxic acute tubular necrosis and aids in its differenti- 25. Cessation of the administration of a drug is the fastest
ation from ischemic acute tubular necrosis? and most effective treatment for
A. Collecting tubular cells A. acute pyelonephritis.
B. Granular casts B. acute interstitial nephritis.
C. Proximal tubular cells C. acute glomerulonephritis.
D. Waxy casts D. chronic glomerulonephritis.
17. Which of the following disorders is characterized by the 26. Yeast is considered part of the normal flora in each of the
urinary excretion of large amounts of arginine, cystine, following locations except in the
lysine, and ornithine? A. gastrointestinal tract.
A. Cystinosis B. oral cavity.
B. Cystinuria C. urinary tract.
C. Lysinuria D. vagina.
D. Tyrosinuria 27. Acute renal failure can be caused by all of the following
18. Generalized loss of proximal tubular function is a char- except
acteristic of A. hemorrhage.
A. Fanconi’s syndrome. B. acute tubular necrosis.
B. nephrotic syndrome. C. acute pyelonephritis.
C. renal glucosuria. D. urinary tract obstruction.
D. renal tubular acidosis. 28. Which of the following statements about chronic renal
19. Which of the following changes is not associated with failure is true?
renal tubular acidosis? A. It can be reversed by appropriate treatment
A. Decreased glomerular filtration rate regimens.
B. Decreased renal tubular secretion of hydrogen ions B. It eventually progresses to end-stage renal disease.
C. Decreased proximal tubular reabsorption of C. It is monitored by periodic determinations of renal
bicarbonate blood flow.
D. Increased back-diffusion of hydrogen ions in the D. Its onset involves a sudden decrease in the glomeru-
distal tubules lar filtration rate.
20. Which of the following disorders is considered a lower 29. Isosthenuria, significant proteinuria, and numerous casts of
urinary tract infection? all types describe the urinalysis findings from a patient with
A. Cystitis A. acute renal failure.
B. Glomerulonephritis B. acute tubular necrosis.
C. Pyelitis C. chronic renal failure.
D. Pyelonephritis D. renal tubular acidosis.
CHAPTER 8 Renal and Metabolic Disease 237

30. Approximately 75% of the renal calculi that form in 37. Which of the following is a characteristic feature of type 2
patients contain diabetes mellitus?
A. calcium. A. Daily insulin injections are necessary.
B. cystine. B. Onset of the disease is usually sudden.
C. oxalate. C. Strong tendency to develop ketoacidosis.
D. uric acid. D. The disease usually presents after 40 years of age.
31. The formation of renal calculi is enhanced by 38. Which of the following abnormalities is not a clinical fea-
A. an increase in urine flow. ture of an infant with galactosuria?
B. the natural “acid-alkaline tide” of the body. A. Cataract formation
C. increases in protein in the urine ultrafiltrate. B. Liver dysfunction
D. increases in chemical salts in the urine ultrafiltrate. C. Mental retardation
32. An overflow mechanism is responsible for the aminoac- D. Polyuria
iduria present in 39. Galactose is produced in the normal metabolism of
A. cystinosis. A. fructose.
B. cystinuria. B. glucose.
C. tyrosinuria. C. lactose.
D. phenylketonuria. D. sucrose.
33. Which of the following hereditary diseases results in the 40. Which of the following features is not a characteristic of
accumulation and excretion of large amounts of homo- diabetes insipidus?
gentisic acid? A. Polyuria
A. Alkaptonuria B. Polydipsia
B. Melanuria C. Increased production of antidiuretic hormone
C. Phenylketonuria D. Urine with a low specific gravity
D. Tyrosinuria 41. Which of the following statements about porphobilino-
34. Which of the following substances oxidizes with expo- gen is true?
sure to air, causing the urine to turn brown or black? A. Porphobilinogen is red and fluoresces.
A. Melanin B. Normally, only trace amounts of porphobilinogen
B. Porphyrin are formed.
C. Tyrosine C. Porphobilinogen is an intermediate product in bili-
D. Urobilinogen rubin formation.
35. Which of the following diseases is related to tyrosine pro- D. Porphobilinogen production is the rate-limiting step
duction or metabolism? in heme synthesis.
1. Tyrosinuria 42. Porphyria is characterized by the body’s attempt to
2. Melanuria A. increase heme degradation.
3. Phenylketonuria B. increase heme formation.
4. Alkaptonuria C. decrease globin synthesis.
A. 1, 2, and 3 are correct. D. decrease iron catabolism.
B. 1 and 3 are correct. 43. Which of the following statements regarding porphyrin
C. 4 is correct. and porphyrin precursors is true?
D. All are correct. 1. Porphyria can be inherited or induced.
36. Which of the following diseases can result in severe men- 2. Porphyrin precursors are neurotoxins.
tal retardation if not detected and treated in the infant? 3. Porphyrins can be dark red or purple.
1. Phenylketonuria 4. Porphyrin precursor accumulation causes skin
2. Maple syrup urine disease photosensitivity.
3. Galactosuria A. 1, 2, and 3 are correct.
4. Alkaptonuria B. 1 and 3 are correct.
A. 1, 2, and 3 are correct. C. 4 is correct.
B. 1 and 3 are correct. D. All are correct.
C. 4 is correct.
D. All are correct.
238 CHAPTER 8 Renal and Metabolic Disease

Case 8.1
A 30-year-old woman is seen by her physician. She has a temper- 1. List any abnormal or discrepant urinalysis findings.
ature of 101°F and reports nausea and headache, with flank (below 2. Select the most probable diagnosis.
ribs and above iliac crest) tenderness and pain. When asked, she A. Normal urinalysis
states that urination is sometimes painful, that she must urinate B. Yeast infection
much more frequently than usual, and that she has a sensation C. Upper urinary tract infection (upper UTI)
of urgency. A random, midstream clean catch urine specimen is D. Lower urinary tract infection (lower UTI)
collected for a routine urinalysis and culture. 3. Which single microscopic finding is most helpful in differenti-
ating an upper UTI from a lower UTI?
Results A. Red blood cells
Physical Chemical B. White blood cells
Examination Examination Microscopic Examination C. Casts >
WBC casts

Color: yellow SG: 1.010 RBC/hpf: 0–2 D. Bacteria


E. Epithelial cells
- Clarity: cloudy pH: 6.5 WBC/hpf: 25–50
4. Another name for this condition is
Odor: — Blood: trace Casts/lpf: 0–2 granular;
Protein: 30 mg/dL 2–5 WBC A. urethritis.

8-
SSA: 1+ Epithelials: few SE cells/hpf B. acute cystitis.
LE: positive Crystals/hpf: few CaOx C. acute pyelonephritis.
Nitrite: positive Bacteria/hpf: moderate D. acute interstitial nephritis.
Glucose: negative 5. State two physiologic mechanisms that can lead to the devel-
Ketones: negative opment of this condition.
Bilirubin: negative 6. Suggest a reason for the trace blood result yet a normal num-
Urobilinogen: normal ber of red blood cells (0–2 per hpf) observed microscopically.
Ascorbic acid: negative 7. At this patient’s level of hematuria, is the blood that is present
most likely contributing to the reagent strip protein result of
30 mg/dL? Explain briefly.

CaOx, Calcium oxalate; hpf, high-power field; LE, leukocyte esterase; lpf, low-power field; RBC, red blood cell; SSA, sulfosalicylic acid precipitation
test; SE, squamous epithelial; WBC, white blood cell.

Case 8.2
A 58-year-old male is seen in the emergency department and 1. List any abnormal or discrepant urinalysis findings.
reports intermittent severe pain that radiates from his right side 2. For each discrepancy, list a test that could be performed to
to his abdomen and groin area (renal colic). He has a frequent confirm or deny the cause for the discrepancy.
need to urinate with little or no urine output. Other complaints 3. Based on the information provided, which of the following is
include a cold that he has been self-treating with over-the-counter the most probable cause of this patient’s condition?
medications and vitamin supplements for longer than a week. A. Renal calculi
B. Urinary tract infection
Results C. Acute glomerulonephritis
Physical Chemical Microscopic D. Drug-induced acute interstitial nephritis
Examination Examination Examination 4. State at least three factors that could influence the develop-
ment of this patient’s condition.

_⑤
Color: pink SG: >1.030 RBC/hpf: 10–25
Clarity: slightly Refractometry: 1.035 WBC/hpf: 5–10

changes in urine pH
cloudy pH: 5.5 Casts/lpf: 0–2 hyaline Urinary stasis

Odor: — Blood: negative Epithelials: few TE '

↑ conc -
oichemicalsalts
- Protein: trace cells/hpf
SSA: trace Crystals/hpf: many CaOx
LE: negative Bacteria/hpf: few
Nitrite: positive
Glucose: negative
Ketones: negative
Bilirubin: negative
Urobilinogen: normal

CaOx, Calcium oxalate; hpf, high-power field; LE, leukocyte esterase; lpf, low-power field; RBC, red blood cell; SSA, sulfosalicylic acid precipitation
test; SE, squamous epithelial; WBC, white blood cell.
CHAPTER 8 Renal and Metabolic Disease 239

Case 8.3
A 14-day-old baby girl is admitted to the hospital with lethargy, 1. List any abnormal or discrepant urinalysis findings.
diarrhea, vomiting, and difficulty in feeding. Physical examination 2. Which results may have been modified by the specimen col-
reveals jaundice, an enlarged liver, and neurologic abnormalities lection conditions?
(e.g., increased muscular tonus). No blood group incompatibility 3. What substance is most likely causing the yellow coloration of
is found. She has lost 1.8 lb since birth. The infant is fitted with a the foam? Bilirubin
collection bag to obtain a urine specimen. The collection takes 4. What is the most likely explanation for the discrepancy in the
place over several hours, and the baby’s urine is sent to the lab- glucose screening results?
oratory for routine urinalysis. 5. What is a possible diagnosis for this patient? How could this
diagnosis be confirmed?
Results 6. Does this patient have a urinary tract infection? Why or
Physical Chemical Microscopic why not?
Examination Examination Examination

%
Color: amber SG: 1.025 RBC/hpf: 0–2
Clarity: cloudy pH: 8.0 WBC/hpf: 0–2
Odor: — Blood: negative Casts/lpf: 0–2 hyaline;
Yellow foam Protein: trace 0–2 granular
noted. SSA: 1+ Epithelials: few SE cells/lpf
LE: negative Crystals/hpf: moderate
Nitrite: negative triple
Glucose: negative phosphate
Clinitest: 1000 mg/dL Bacteria/hpf: few
Ketones: negative
Bilirubin: positive
Ictotest: positive
Urobilinogen: normal

hpf, High-power field; LE, leukocyte esterase; lpf, low-power field; RBC, red blood cell; SSA, sulfosalicylic acid precipitation test; SE, squamous
epithelial; WBC, white blood cell.
240 CHAPTER 8 Renal and Metabolic Disease

Case 8.4
A 6-year-old boy is brought to the hospital emergency depart- 1. List any abnormal or discrepant urinalysis findings.
ment by his mother. This morning after his bath, she noticed that 2. What urinary substance is responsible for the white foam
his scrotum appeared swollen. In addition, for the past several noted during the urinalysis? Protein
days her son has been has been tired—that is, definitely not 3. If the glomerular filtration barrier loses its “shield of negativ-
his active self—and has been complaining of a headache. Phys- ity,” which plasma protein is usually lost first? Explain your
ical examination is unremarkable except for mild peripheral selection. Albumin
edema of the eyelids, scrotum, and lower limbs. No skin rash 4. Explain the physiologic processes responsible for the edema
or fever is present. During the examination, the boy reveals that exhibited in this patient.
he “doesn’t need to go potty much anymore.” The previous 5. The proteinuria in this patient should be classified as
week, the boy had a routine wellness physical and received a A. glomerular proteinuria.
booster of the diphtheria-pertussis-tetanus vaccine. The follow- B. tubular proteinuria.
ing blood tests and routine urinalysis were obtained: C. overflow proteinuria.
D. postrenal proteinuria.
Urinalysis Results 6. Based on the patient’s symptoms (edema) and laboratory
Physical Chemical Microscopic results (proteinuria, hypoalbuminemia, hyperlipidemia), select
Examination Examination Examination the most probable disorder.
A. acute glomerulonephritis.
Color: dark SG: 1.030 RBC/hpf: 0–2
B. acute pyelonephritis.
yellow pH: 6.0 WBC/hpf: 0–2
C. acute renal failure.
Clarity: clear Blood: negative Casts/lpf: 0–2 hyaline
D. nephrotic syndrome.
Odor: — - Protein: 500 mg/dL Epithelials: none seen
By exclusion, it was determined this patient had minimal
SSA: 3+ Crystals/hpf: few
change disease that presented as (see answer to Question 6),

White foam LE: negative amorphous
and he was treated promptly with corticosteroids (i.e., oral pred-
that Nitrite: negative urates
nisone). In 8 days, his urine output increased significantly, and
does not Glucose: negative Bacteria: none seen
his urine protein result decreased to 30 mg/dL. A routine urinal-
dissipate Ketones: negative
ysis another 24 hours later was negative for urine protein.
was noted. Bilirubin: negative
Urobilinogen: normal

Blood Results Reference Range


Sodium: 136 mmol/L 136–145 mmol/L
Potassium: 4.2 mmol/L 3.5–5.0 mmol/L
Glucose: 92 mg/dL 70–105 mg/dL
Urea nitrogen: 25 mg/dL 11–23 mg/dL
Creatinine: 0.8 mg/dL 0.6–1.2 mg/dL
Total protein: 4.8 g/dL 6.0–8.0 g/dL
Albumin: 1.3 g/dL 3.5–5.5 g/dL
Cholesterol: 282 mg/dL <200 mg/dL
Triglyceride: 255 mg/dL <150 mg/dL

hpf, High-power field; LE, leukocyte esterase; lpf, low-power field; RBC, red blood cell; SSA, sulfosalicylic acid precipitation test; WBC, white
blood cell.
CHAPTER 8 Renal and Metabolic Disease 241

Case 8.5
Two days previous, a 26-year-old woman saw her primary care 1. List any abnormal or discrepant urinalysis findings.
physician, and it was determined that she had a urinary tract 2. A cytospin preparation of the urine sediment is performed and
infection. A conventional 10-day regimen of ampicillin was pre- stained using Hansel stain. A differential white cell count of
scribed. Today, she returns to the clinic with a fever and urticarial the sediment reveals 12% eosinophils. Based on this finding
rash on her chest, back, face, and hands. The following routine and the urinalysis results, the most likely diagnosis is
urinalysis is obtained: A. acute glomerulonephritis.
B. acute interstitial nephritis.
Urinalysis Results C. acute pyelonephritis.
Physical Chemical Microscopic D. nephrotic syndrome.
Examination Examination Examination 3. The proteinuria in this patient should be classified as
A. glomerular proteinuria.

078%0
Color: dark SG: 1.015 RBC/hpf: 5–10
yellow pH: 6.5 WBC/hpf: 10–25 B. tubular proteinuria.
Clarity: Blood: small Casts/lpf: 0–2 WBC; 0–2 C. overflow proteinuria.
cloudy Protein: 100 mg/dL RTE; 2–5 D. postrenal proteinuria.
Odor: — LE: positive (1+) granular 4. Which microscopic findings indicate that the inflammatory
Nitrite: negative Epithelials: moderate RTE process is in the kidneys?
Glucose: negative cells/hpf; few 5. State three reasons why the nitrite test is negative despite the
Ketones: negative TE cells/hpf; presence of bacteria in the urine sediment.
Bilirubin: negative few SE cells/lpf
Urobilinogen: normal Crystals/hpf: few urates
Bacteria/hpf: few

hpf, High-power field; LE, leukocyte esterase; lpf, low-power field; RBC, red blood cell; RTE, renal tubular epithelial; SE, squamous epithelial; TE,
transitional epithelial; WBC, white blood cell.

Case 8.6
A 43-year-old woman with a 15-year history of systemic lupus 1. List any abnormal or discrepant urinalysis findings.
erythematosus is transferred to a university hospital because 2. What substance most likely accounts for the brown color of
of significant deterioration of renal function. The following rou- this urine? Oxidized hemoglobin
tine urinalysis is obtained on admission: 3. State two reasons to explain why the leukocyte esterase test
is negative despite increased numbers of white blood cells in
Urinalysis Results the urine sediment.
Physical Chemical Microscopic 4. At this patient’s level of hematuria, is the blood that is present
Examination Examination Examination contributing to the reagent strip protein result? Explain briefly.
5. List the microscopic finding(s) that indicate whether hematu-

•←
So
Color: brown SG: 1.010 RBC/hpf: 25–50;
Clarity: pH: 7.0 dysmorphic ria/hemorrhage is occurring in the nephrons?
cloudy Blood: large forms present 6. The proteinuria in this patient should be classified as
Odor: — Protein: 100 mg/dL WBC/hpf: 5–10 A. glomerular proteinuria.
LE: negative Casts/lpf: 0–2 RBC; B. tubular proteinuria.
Nitrite: negative 5–10 granular; C. overflow proteinuria.
Glucose: negative 0–2 hyaline D. postrenal proteinuria.
Ketones: negative Epithelials: few TE cells/hpf 7. Based on the information provided and the results obtained,
Bilirubin: negative Crystals/hpf: few CaOx the most likely diagnosis is
Urobilinogen: normal Bacteria: none seen A. acute glomerulonephritis.
B. acute interstitial nephritis.
C. acute pyelonephritis.
D. nephrotic syndrome.
8. Briefly describe a physiologic mechanism to account for the
development of this patient’s condition (as cited in
Question 7).
CaOx, Calcium oxalate; hpf, high-power field; LE, leukocyte esterase; lpf, low-power field; RBC, red blood cell; TE, transitional epithelial; WBC, white
blood cell.
242 CHAPTER 8 Renal and Metabolic Disease

Case 8.7
A 23-year-old woman is seen in the emergency room with acute 1. Circle any abnormal or discrepant urinalysis findings.
abdominal pain, nausea, and hypertension. She had a previous 2. For academic reasons, the urine specimen was refrigerated
admission 1 year ago for intestinal problems and neurologic and was examined the next day for any change in color.
symptoms (depression). At that time, gastrointestinal and neuro- The yellow urine now had a pink hue; however, it was not
logic examinations were negative. She recently started taking prominent. What substance most likely is causing the pink
oral contraceptives and states that she is taking no other medi- hue now observed in the urine? Por pho bitin
cations. In addition, she has a family history of acute intermittent 3. See Appendix E. If the results of the Hoesch test were ques-
porphyria. Routine hematologic and chemistry tests are ordered, tionable or if reagents were not available, what test could be
and all results are normal. A routine urinalysis is performed. performed to confirm the presence of this substance? Watson Schwartz test
-

4. Explain the physiologic process that results in the appearance


Results of this substance in the urine.
Physical Chemical 5. What is this patient’s most likely diagnosis? Acute intermittent porphyria
Examination Examination Confirmatory Tests 6. In addition to the substance observed, this patient would most
likely have increased levels of
Color: yellow SG: 1.015
A. blood porphyrins.
Clarity: clear pH: 5.0
B. fecal porphyrins.
Odor: — Blood: negative
C. urinary coproporphyrin.
Protein: negative
D. urinary δ-aminolevulinic acid.
LE: negative
7. Are any reagent strip tests capable of detecting the substance
Nitrite: negative
identified in Question 2 in urine? NO
Glucose: negative
Ketones: negative
Bilirubin: negative
Urobilinogen: normal
-
Hoesch test: positive

LE, Leukocyte esterase.


9
Cerebrospinal Fluid Analysis

LEARNING OBJECTIVES
After studying this chapter, the student should be able to: • Albumin
1. Describe the formation of cerebrospinal fluid (CSF) and • Glucose
state at least three functions that the CSF performs. • Immunoglobulin G
2. Describe the procedure for lumbar puncture and the • Lactate
proper collection technique for CSF. • Total protein
3. Discuss the importance of timely processing and testing 7. Describe briefly protein electrophoretic patterns of CSF
of CSF and state at least three adverse effects of time delay and the abnormal presence of oligoclonal banding.
on CSF specimens. 8. Calculate the CSF/serum albumin index and the CSF/
4. State the physical characteristics of normal CSF and immunoglobulin G index and state the clinical
discuss how each characteristic can be modified in disease importance of each index.
states. 9. Discuss the proper microbiological examination of CSF
5. Discuss the clinical importance of the microscopic and its importance in the diagnosis of infectious diseases
examination of CSF. of the central nervous system.
6. Compare and contrast the concentrations of the 10. Explain briefly the role of CSF immunologic tests in the
following constituents of CSF in health and in disease diagnosis of meningitis.
states:

CHAPTER OUTLINE
Physiology and Composition, 243 Chemical Examination, 254
Specimen Collection, 246 Protein, 254
Physical Examination, 247 Glucose, 256
Microscopic Examination, 248 Lactate, 256
Total Cell Count, 248 Microbiological Examination, 256
Red Blood Cell (Erythrocyte) Count, 248 Microscopic Examination of CSF Smears, 256
White Blood Cell (Leukocyte) Count, 249 Culture, 257
Differential Cell Count, 250 Immunologic Methods, 257

K E Y T E R M S*
blood-brain barrier oligoclonal bands
cerebrospinal fluid pleocytosis
choroid plexus stat
intrathecal subarachnoid space
meninges ventricles
meningitis xanthochromia

*Definitions are provided in the chapter and glossary.

cells that line the brain and spinal cord also play a minor role
PHYSIOLOGY AND COMPOSITION in the production of CSF. The formation of CSF can be
Cerebrospinal fluid (CSF) bathes the brain and spinal cord. described as a selective secretion from plasma, not as an ultra-
CSF is produced primarily (70%) from secretions into the four filtrate. This is evidenced by higher CSF concentrations of
ventricles of the brain by the highly vascular choroid plexus some solutes (e.g., sodium, chloride, magnesium) and lower
(vascular fringe–like folds in the pia mater). The ependymal CSF concentrations of other solutes (e.g., potassium, total

243
258 CHAPTER 9 Cerebrospinal Fluid Analysis

13. Normasell DE, Stacy EK, Booker CF, et al: Detection of beta-2 15. Myint T, Ribes JA, Stadler LP: Ten-year old with fever, headache,
transferrin in otorrhea and rhinorrhea in a routine clinical and neck stiffness; primary amebic meningoencephalitis. Clin
laboratory setting. Clin Diagn Lab Immunol 1:68, 1994. Infect Dis 55(12):1677, 1737–1738, 2012.
14. Murray PR, Hampton CM: Recovery of pathogenic bacteria
from cerebrospinal fluid. J Clin Microbiol 12:554–557, 1980.

STUDY QUESTIONS
1. Cerebrospinal fluid (CSF) is produced primarily from 9. In CSF, which of the following findings indicates a trau-
A. secretions by the choroid plexus. matic puncture?
B. diffusion from plasma into the central nervous system. A. The presence of erythrophagocytic cells in the CSF
C. ultrafiltration of plasma in the ventricles of the brain. B. Hemosiderin granules within macrophages in the
D. excretions from ependymal cells lining the brain and CSF sediment
spinal cord. C. An uneven distribution of blood in the CSF col-
2. Cerebrospinal fluid is found between the lection tubes
A. arachnoid and dura mater. D. A xanthochromic supernatant after CSF centrifugation
B. arachnoid and pia mater. 10. How many leukocytes are normally present in the CSF
C. pia mater and dura mater. obtained from an adult?
D. pia mater and choroid plexus. A. 0 to 5 cells/μL
3. Which of the following statements regarding CSF is true? B. 0 to 10 cells/μL
A. Cerebrospinal fluid is constantly produced. C. 0 to 20 cells/μL
B. Cerebrospinal fluid is reabsorbed into the blood at D. 0 to 30 cells/μL
the choroid plexus. 11. Which of the following cells can be present in small num-
C. Cerebrospinal fluid is essentially composed of diluted bers in normal CSF?
plasma. A. Erythrocytes
D. Cerebrospinal fluid circulates through the brain and B. Lymphocytes
spinal cord because of active and passive diffusion C. Macrophages
processes. D. Plasma cells
4. Which of the following substances does not normally 12. Which of the following cell types predominate in CSF
pass through the blood-brain barrier? during a classic case of bacterial meningitis?

=
A. PO2 A. Lymphocytes
B. Albumin B. Macrophages
C. Glucose C. Monocytes
D. Fibrinogen D. Neutrophils
5. During a lumbar puncture procedure, the first collection 13. Which of the following cell types predominate in CSF
tube of CSF removed should be used for during a classic case of viral meningitis?
A. chemistry tests. A. Lymphocytes
B. cytologic studies. B. Macrophages
C. hematologic tests. C. Monocytes
D. microbiological studies. D. Neutrophils
6. Which of the following is not an analytical concern when 14. When choroid plexus cells and ependymal cells are pres-
the processing and testing of CSF are delayed? ent in CSF, they
A. The viability of microorganisms A. are often clinically significant.
B. The lability of the immunoglobulins B. represent the demyelination of nerve tissue.
C. The lysis of leukocytes and erythrocytes C. can closely resemble clusters of malignant cells.
D. Alterations in the chemical composition D. indicate breakdown of the blood-brain barrier.
7. Pleocytosis is a term used to describe 15. All of the following proteins are normally present in the
A. an increased number of cells in the CSF. CSF except
B. a pink, orange, or yellow CSF specimen. A. albumin.
C. an increased protein content in the CSF caused by B. fibrinogen.
cellular lysis. C. transthyretin.
D. inflammation and sloughing of cells from the cho- D. transferrin.
roid plexus. 16. Which of the following events does not result in an
8. All of the following can cause xanthochromia in CSF except increased CSF total protein?
A. high concentrations of protein. A. A traumatic puncture procedure
B. high concentrations of bilirubin. B. Alterations in the blood-brain barrier
C. increased numbers of leukocytes. C. Trauma to the central nervous system, resulting in
D. erythrocytes from a traumatic tap. fluid loss
D. Decreased reabsorption of CSF into the peripheral
blood
CHAPTER 9 Cerebrospinal Fluid Analysis 259

17. Which of the following proteins in the CSF is used to 21. Which of the following statements about CSF glucose is
monitor the integrity of the blood-brain barrier? false?
A. Albumin A. Increased CSF glucose values are diagnostically
B. Transthyretin significant.
C. Transferrin B. Glucose enters the CSF by active transport and sim-
D. Immunoglobulin G ple diffusion.
18. An immunoglobulin G index greater than 0.70 indicates C. Decreased CSF glucose values reflect a defective
A. intrathecal synthesis of immunoglobulin G. blood-brain barrier and increased glycolysis.
B. a compromised blood-brain barrier. D. CSF glucose values reflect the plasma glucose con-
C. active demyelination of neural proteins. centration 30 to 90 minutes preceding collection.
D. increased transport of immunoglobulin G from 22. Normal CSF lactate levels (less than 25 mg/dL) are com-
plasma into the CSF. monly found in patients with
19. An unknown fluid can be positively identified as CSF by A. bacterial meningitis.
determining the B. fungal meningitis.
A. lactate concentration. C. tuberculous meningitis.
B. albumin concentration. D. viral meningitis.
C. presence of oligoclonal banding on electrophoresis. 23. Which of the following procedures frequently provides a
D. presence of carbohydrate-deficient transferrin on rapid presumptive diagnosis of bacterial meningitis?
electrophoresis. A. A blood culture
20. Which of the following statements about oligoclonal B. A CSF culture
bands is false? C. A CSF Gram stain
A. In the CSF, these bands indicate increased intrathecal D. Immunologic tests on CSF for microbial antigens
concentrations of immunoglobulin G. 24. India ink preparations and microbial antigen tests on
B. The bands usually correlate with the stage of disease CSF can aid in the diagnosis of
and can be used to predict disease progression. A. bacterial meningitis.
C. The bands are often present in the CSF and serum of B. fungal meningitis.
individuals with a lymphoproliferative disease. C. tuberculous meningitis.
D. The bands are often present in the CSF but not in the D. viral meningitis.
serum of individuals with multiple sclerosis.

Case 9.1
A 4-year-old girl is brought to the emergency room by her par- 1. List any abnormal results.
ents. She is lethargic, reports that her head hurts, and shows 2. Calculate the CSF/plasma glucose ratio.
signs of stiffness in her neck. Her mother states that she has 3. These results are most consistent with a preliminary diagno-
had “a temperature” for the past 2 days; her current tempera- sis of
ture is determined to be 104°C. She is admitted to the hospital, A. viral meningitis.
where blood is drawn and a lumbar puncture performed. Cere- B. bacterial meningitis.
brospinal fluid and pertinent blood chemistry results follow. C. Guillain-Barre! syndrome.
D. acute lymphocytic leukemia.
Blood Chemistry Results Reference Interval 4. Does the CSF lactate value assist in determining a diagnosis
Glucose, fasting: 90 mg/dL <110 mg/dL for this patient?
5. Situation: If Gram stain results are negative (i.e., no organisms
seen), would you change the diagnosis selected in Question 3?
Why or why not?
Cerebrospinal Fluid Results 6. Explain briefly the physiologic mechanisms that account for
Microscopic the CSF total protein and glucose values.
Physical Examination Examination Chemical Examination


Color: colorless Leukocyte count: Total protein: 130 mg/dL
Clarity: cloudy (3 +) 7300 cells/μL Glucose: 32 mg/dL
Gram stain: results Differential count: Lactate: 33 mg/dL
pending Monocytes: 7%
Lymphocytes: 6%
Neutrophils: 87%
260 CHAPTER 9 Cerebrospinal Fluid Analysis

Case 9.2
A 39-year-old woman noticed numbness in her left leg and diffi- 1. List any abnormal results.
culty walking approximately 3 months ago. Since that time, the 2. Calculate CSF/serum albumin index as follows:
numbness seems to come and go, along with episodes of dizzi-
AlbuminCSF ðmg=dLÞ
ness. More recently, she has experienced numbness on the CSF=serum albumin index ¼
right side of her face and “blurred” vision in her right eye that Albuminserum ðg=dLÞ
comes and goes. She gets tired easily and often feels unsteady ðreference interval :< 9:0Þ
while upright and walking. She is admitted to the hospital for
3. Why is the CSF/serum albumin index a good indicator of the
tests. Cerebrospinal fluid and pertinent blood chemistry results
integrity of the blood-brain barrier?
follow.
4. Calculate the CSF IgG index as follows:
Blood Chemistry Results Reference Interval
IgGCSF ðmg=dLÞ Albuminserum ðg=dLÞ
Glucose, fasting: 85 mg/dL <110 mg/dL CSF IgG index ¼ $
IgGserum ðg=dLÞ AlbuminCSF ðmg=dLÞ
Albumin: 4.6 g/dL 3.5–5.0 g/dL
Immunoglobulin G: 1.4 g/dL 0.65–1.50 g/dL ðreference interval : 0:30 % 0:70Þ

5. Suggest a diagnosis that is consistent with the results


obtained and the patient history.
6. Based on the diagnosis chosen, state two additional chemical
Cerebrospinal Fluid Results tests that could be performed to confirm this diagnosis, and
indicate the results expected.
Physical Microscopic Chemical
Examination Examination Examination


Color: colorless Leukocyte count: Total protein: 45 mg/dL
Clarity: clear 3 cells/μL Albumin: 28 mg/dL
Gram stain: no Differential count: IgG: 12.4 mg/dL
organisms Monocytes: 24% Glucose: 72 mg/dL
seen Lymphocytes: 75% Lactate: 18 mg/dL
Neutrophils: 1%
10
Pleural, Pericardial, and
Peritoneal Fluid Analysis

LEARNING OBJECTIVES
After studying this chapter, the student should be able to: 5. Compare and contrast chylous and pseudochylous
1. Describe the function of serous membranes as they relate effusions.
to the formation and absorption of serous fluid. 6. Correlate the microscopic examination and differential
2. Describe four pathologic changes that lead to the formation cell count of serous fluid analyses with diseases that
of an effusion. affect the serous membranes.
3. Discuss appropriate collection requirements for serous 7. Correlate the concentrations of selected chemical
fluid specimens. constituents of serous fluids with various disease states.
4. Classify a serous fluid effusion as a transudate or an 8. Discuss the microbiological examination of serous fluids
exudate based on the examination of its physical, and its importance in the diagnosis of infectious
microscopic, and chemical characteristics. diseases.

CHAPTER OUTLINE
Physiology and Composition, 261 Glucose, 269
Specimen Collection, 263 Amylase, 269
Transudates and Exudates, 264 Lipids (Triglyceride and Cholesterol), 270
Physical Examination, 265 pH, 270
Microscopic Examination, 266 Carcinoembryonic Antigen, 270
Total Cell Counts, 266 Microbiological Examination, 270
Differential Cell Count, 266 Staining Techniques, 270
Cytologic Examination, 269 Culture, 270
Chemical Examination, 269
Total Protein and Lactate Dehydrogenase Ratios, 269

K E Y T E R M S*
ascites mesothelial cells
chyle paracentesis
chylous effusion pseudochylous effusion
effusion serous fluid
exudate transudate

*Definitions are provided in the chapter and glossary.

PHYSIOLOGY AND COMPOSITION cavity. The serous membranes that line these cavities consist
of a thin layer of connective tissue covered by a single layer of
The lungs, heart, and abdominal organs are surrounded by a flat mesothelial cells. Within the membrane is an intricate
thin, continuous, serous membrane, as well as by the internal network of capillary and lymphatic vessels. Each membrane
surfaces of the body cavity wall. A space or cavity filled with is attached firmly to the body wall and the organ it surrounds;
fluid lies between the membrane that covers the organ (vis- however, the opposing surfaces of the membrane—despite
ceral membrane) and the membrane that lines the body wall close contact—are not attached to each other. Instead, the
(parietal membrane) (Fig. 10.1). Each cavity is separate and is space between the opposing surfaces (i.e., between the visceral
named for the organ or organs it encloses. The lungs are indi- and parietal membranes) is filled with a small amount of fluid
vidually surrounded by a pleural cavity, the heart by the peri- that serves as a lubricant between the membranes, which per-
cardial cavity, and the abdominal organs by the peritoneal mits free movement of the enclosed organ. The cavity fluid is

261
CHAPTER 10 Pleural, Pericardial, and Peritoneal Fluid Analysis 271

BIBLIOGRAPHY editors: Henry’s clinical diagnosis and management by


laboratory methods, ed 22, Philadelphia, 2011, Saunders.
Clinical and Laboratory Standards Institute (CLSI): Body fluid Kjeldsberg CR, Knight JA: Body Fluids, ed 3, Chicago, 1993,
analysis for cellular composition: approved guideline, CLSI American Society for Clinical Pathology Press.
Document H56-A, Wayne, PA, 2006, CLSI.
Karcher DS, McPherson RA: Cerebrospinal, synovial, serous body
fluids, and alternative specimens. In McPherson RA, Pincus MR,

/
STUDY QUESTIONS
1. Which of the following statements about serous fluid– 6. Thoracentesis refers specifically to the removal of fluid
filled body cavities is true? from the
1. A parietal membrane is attached firmly to the A. abdominal cavity. -

body cavity wall. B. pericardial cavity.


.
_

periocm til
2. Serous fluid acts as a lubricant between opposing C. peritoneal cavity. -

peritine
membranes. - D. pleural cavity.
3. A serous membrane is composed of a single layer 7. Which of the following tubes could be used for a bacterial
of flat mesothelial cells. culture of serous fluid?
4. The visceral and parietal membranes of an organ A. EDTA
are actually a single continuous membrane. B. Sodium citrate
A. 1, 2, and 3 are correct. C. Sodium fluoride
B. 1 and 3 are correct. ✓D. Sodium heparin
C. 4 is correct. 8. Serous fluid for bacterial culture should be stored at
D. All are correct. ✓ A. !20°C.

/2. Which of the following mechanisms is responsible for the


formation of serous fluid in body cavities?
B. 2°C to 8°C.
C. 20°C to 24°C. -
A. Ultrafiltration of circulating blood plasma .
D. 36°C to 38°C.
B. Selective absorption of fluid from the lymphatic 9. Which of the following parameters best identifies a fluid
system as a transudate or an exudate?
C. Diuresis of solutes and water across a concentration A. Color and clarity
gradient B. Leukocyte and differential counts
D. Active secretion by mesothelial cells that line the C. Total protein and specific gravity measurements
serous membranes -D. Total protein ratio and lactate dehydrogenase ratio
3. Which of the following conditions enhances the forma- 10. Chylous and pseudochylous effusions are differentiated
r tion of serous fluid in a body cavity? by their
A. Increased lymphatic absorption A. physical examinations.
B. Increased capillary permeability - B. cholesterol concentrations.
C. Increased plasma oncotic pressure rtriglyceride concentrations. r
C.
D. Decreased capillary hydrostatic pressure D. leukocyte and differential counts.
4. The pathologic accumulation of fluid in a body cavity is 11. Which of the following conditions is most often associ-
called ated with the formation of a transudate?
A. an abscess. A. Pancreatitis
B. an effusion. r B. Surgical procedures
C. pleocytosis. -C. Congestive heart failure -

D. paracentesis. D. Metastatic neoplasm


5. Paracentesis and serous fluid testing are performed to 12. Match the type of serous effusion most often associated
1. remove serous fluids that may be compressing a with each pathologic condition.

I vital organ.
2. determine the pathologic cause of an effusion.
3. identify an effusion as a transudate or an exudate. Pathologic Condition
I
__ A. Neoplasms
Type of Serous
Effusion
1. Exudate

: I
4. prevent volume depletion caused by the accumu-
lation of fluid in body cavities. __
2 B. Hepatic cirrhosis 2. Transudate

0
__
I C. Infection
A. 1, 2, and 3 are correct. 1 D. Rheumatoid arthritis -
__
B. 1 and 3 are correct. __
I
E. Trauma
C. 4 is correct. 2 F. Nephrotic syndrome
__
D. All are correct.
272 CHAPTER 10 Pleural, Pericardial, and Peritoneal Fluid Analysis

malignant cells? O
13. Which of the following features is not a characteristic of 16. A pleural or peritoneal fluid amylase level two times
higher than the serum amylase level can be found in effu-
A. Irregular nuclear membrane sions resulting from

-
B. Uneven nuclear chromatin distribution
C. Less than normal nucleus-to-cytoplasm ratio
O A. pancreatitis.
B. hepatic cirrhosis.
D. Multiple prominent nucleoli with irregular borders C. rheumatoid arthritis.
14. Which of the following laboratory findings on an effusion D. lymphatic obstruction.
does not indicate a specific diagnosis?
A. LE cells found during the microscopic examination
O
17. A glucose concentration difference greater than 30 mg/
dL between the serum and an effusion is associated with
-B. A serous fluid glucose concentration less than 60
mg/dL O A. pancreatitis.
B. hepatic cirrhosis.
C. Microorganisms identified by Gram or acid-fast stain C. rheumatoid arthritis. -
D. Malignant cells identified during the microscopic or
cytologic examination
15. An abnormally low fluid pH value is useful when evalu-
D. lymphatic obstruction.
O
18. Which of the following actions can adversely affect the
chances of obtaining a positive stain or culture when per-
ating conditions associated with forming microbiological studies on infectious serous
rA. pleural effusions. fluid?
B. pleural and pericardial effusions. A. Using a large volume of serous fluid for the inoculum
C. pericardial and peritoneal effusions. r B. Storing serous fluid specimens at refrigerator
D. pleural, pericardial, and peritoneal effusions. temperatures
C. Using an anticoagulant in the serous fluid collection
container
D. Concentrating the serous fluid before preparing
smears for staining

Case 10.1
A 51-year-old man with a history of tuberculosis is found to have a unilateral pleural effusion. A pleural fluid specimen is obtained
by thoracentesis and is sent to the laboratory for evaluation.

Blood Chemistry Results Pleural Fluid Results


Reference Physical Microscopic Chemical
Interval Examination Examination Examination
Total protein: 7.0 g/dL 6.0–8.3 mg/dL Color: yellow Leukocyte count: 1100 Total protein: 4.2 g/dL
Lactate dehydrogenase: 520 U/L 275–645 U/L Clarity: cloudy cells/μL Lactate
Glucose, fasting: 75 mg/dL "110 mg/dL Clots present: no Differential count: dehydrogenase: 345 U/L
Mononuclear: 93% Glucose: 55 mg/dL
Neutrophils: 3%
Gram stain: no organisms
seen;
leukocytes
present

0008%0
1. Calculate the fluid-to-serum total protein ratio. 0-60
2. Calculate the fluid-to-serum lactate dehydrogenase ratio. 0.66
3. Classify this pleural fluid specimen as a transudate or an exudate, and state two physiologic mechanisms that can cause this
type of effusion.
CHAPTER 10 Pleural, Pericardial, and Peritoneal Fluid Analysis 273

Case 10.2
A 48-year-old woman has ascites and pleural effusion. Blood is drawn and a peritoneal fluid specimen is obtained by paracentesis
and sent to the laboratory for evaluation.

Blood Chemistry Results Peritoneal Fluid Results


Reference Physical Microscopic Chemical
Interval Examination Examination Examination
Total protein: 6.5 g/dL 6.0–8.3 mg/dL Color: yellow Leukocyte count: 8 cells/μL Total protein: 2.9 g/dL
Lactate dehydrogenase: 300 U/L 275–645 U/L Clarity: clear Cytologic Lactate
Glucose, fasting: 82 mg/dL "110 mg/dL Clots examination: no malignant dehydrogenase: 125 U/L
Liver function tests*: normal present: no cells seen Glucose: 67 mg/dL
Gram stain: no organisms
seen

00
1. Calculate the fluid-to-serum total protein ratio. 0.45
2. Calculate the fluid-to-serum lactate dehydrogenase ratio. 0.42
3. Classify this peritoneal fluid specimen as a transudate or an exudate, and state two physiologic mechanisms that can cause
this type of effusion.

*Alanine aminotransferase, aspartate aminotransferase, γ-glutamyltransferase, alkaline phosphatase, and bilirubin.


11
Synovial Fluid Analysis

LEARNING OBJECTIVES
After studying this chapter, the student should be able to: 5. State physical characteristics of normal synovial fluid and
1. Describe the formation and function of synovial fluid. discuss how each characteristic is modified in disease states.
2. Summarize the four principal classifications of joint disease. 6. Correlate the cells and crystals observed during microscopic
3. Classify synovial fluid as normal, noninflammatory, examination of synovial fluid with various joint diseases.
inflammatory, septic, or hemorrhagic using various 7. Compare and contrast concentrations of selected
laboratory results. chemical constituents of synovial fluid from healthy
4. Discuss appropriate tubes for the collection and joints with that from diseased joints.
distribution of synovial fluid specimens; discuss 8. Discuss the microbiological examination of synovial fluid
the importance of timely specimen processing and and its importance in the diagnosis of infectious joint disease.
testing.

CHAPTER OUTLINE
Physiology and Composition, 274 Differential Cell Count, 278
Classification of Joint Disorders, 275 Crystal Identification, 278
Specimen Collection, 275 Chemical Examination, 282
Physical Examination, 277 Glucose, 282
Color, 277 Total Protein, 283
Clarity, 277 Uric Acid, 283
Viscosity, 277 Lactate, 283
Clot Formation, 278 Microbiological Examination, 283
Microscopic Examination, 278 Gram Stain, 283
Total Cell Count, 278 Culture and Molecular Methods, 283

K E Y T E R M S*
arthritis synovial fluid
arthrocentesis synoviocytes
hyaluronate

*Definitions are provided in the chapter and glossary.

PHYSIOLOGY AND COMPOSITION (e.g., collagenases). The second type of synoviocyte synthe-
sizes hyaluronate, a mucopolysaccharide linked with approx-
In areas of the skeleton where friction could develop, such as imately 2% protein. The synoviocytes are loosely organized in
the joints, bursae, and tendon sheaths, viscous synovial fluid the synovial membrane and differ from cells in other lining
is present. Within articulated diarthrodial joints (e.g., the membranes in that they have no basement membrane, and
knee), the ends of apposing bones are covered with articular adjacent synovial cells are not joined with desmosomes.
cartilage, the joint space is lined by a synovial membrane Beneath the synoviocytes is a thin layer of loose connective
(except in weight-bearing areas), and synovial fluid bathes tissue containing a vast network of blood vessels, lymphatics,
and lubricates the joint (Fig. 11.1). The surface of the synovial and nerves. Variable numbers of mononuclear cells are also
membrane surrounding the joint consists of numerous found in this connective tissue layer.
microvilli with a layer, one to three cells deep, of synovial cells Synovial fluid is formed by the ultrafiltration of plasma
called synoviocytes (Fig. 11.2). Two types of synoviocytes are across the synovial membrane and from secretions by syno-
present in the synovial membrane. The most prevalent type is viocytes. The resultant viscous fluid serves as a lubricant for
actively phagocytic and synthesizes degradative enzymes the joint and is the sole nutrient source for the metabolically

274
284 CHAPTER 11 Synovial Fluid Analysis

7. Villanueva TG, Schumacher HR, Jr: Cytologic examination of BIBLIOGRAPHY


synovial fluid, Diagn Cytopathol 3:141–147, 1987.
8. Cohen AS, Goldenberg D: Synovial fluid, Laboratory diagnostic Eisenberg JM, Schumacher HR, Davidson PK, Kaufmann L:
procedures in the rheumatic diseases, New York, 1985, Grune & Usefulness of synovial fluid analysis in the evaluation of joint
Stratton. effusions, Arch Intern Med 144:715–719, 1984.
9. Galagan KA, Blomberg D, Cornbleet PJ, Glassy EF, editors: Gatter RA: A practical handbook of joint fluid analysis, Philadelphia,
Color atlas of body fluids, Northfield, IL, 2006, College of 1991, Lea & Febiger.
American Pathologists (CAP). Karcher DS, McPherson RA: Cerebrospinal, synovial,
10. Glasser L: Reading the signs in synovia, Diagn Med 3:35–50, serous body fluids, and alternative specimens.
1980. In McPherson RA, Pincus MR, editors: Henry’s clinical
11. Carter JD, Hudson AP: Reactive arthritis: clinical aspects and diagnosis and management by laboratory methods, ed 22,
medical management, Rheum Dis Clin N Am 35:21–44, 2009. Philadelphia, 2011, Saunders.
12. Laboratory Clinical: Body fluid analysis for cellular composition: Pal B, Nash J, Oppenheim B, et al: Is routine synovial fluid analysis
approved guideline, CLSI Document H56-A, Wayne, PA, 2006, necessary? Lessons and recommendations from an audit,
CLSI. Rheumatol Int 18:181–182, 1999.
13. Garcia-Arias M, Balsa A, Mola EM: Septic arthritis, Best Pract Shmerling RH: Synovial fluid analysis: a critical reappraisal, Rheum
Res Clin Rheumatol 25:407–421, 2011. Dis Clin North Am 20:503–512, 1994.
Shmerling RH, Delbanco TL, Tosteson ANA, et al: Synovial fluid
tests: what should be ordered? JAMA 264:1009–1014, 1990.

STUDY QUESTIONS
1. Which of the following tasks is a function of synovial 6. A synovial fluid specimen is received in the laboratory
fluid? 2 hours after collection. Which of the following changes
1. Providing lubrication for a joint to the fluid will most likely have taken place?
= 2. Assisting in the structural support of a joint
3. Transporting nutrients to articular cartilage
A. The specimen will have clotted.
B. The uric acid concentration will have decreased.
4. Synthesizing hyaluronate and degradative enzymes - C. Crystals may have precipitated or dissolved.
A. 1, 2, and 3 are correct. D. The lactate concentration will have decreased
B. 1 and 3 are correct. because of anaerobic glycolysis.
C. 4 is correct. 7. Which of the following anticoagulants does not have the
D. All are correct. potential to precipitate out in crystalline form when used
2. Which of the following statements is a characteristic of for synovial fluid specimens?
normal synovial fluid? A. Sodium citrate
- A. Synovial fluid is viscous. -B. Sodium heparin
B. Synovial fluid is slightly turbid. C. Lithium heparin
C. Synovial fluid is dark yellow. D. Potassium oxalate
D. Synovial fluid forms small clots on standing. 8. A synovial fluid specimen has a high cell count and
3. Which of the following components is not normally requires dilution to be counted. Which of the following
present in synovial fluid? diluents should be used?
- A. Fibrinogen
B. Neutrophils
- A. Normal saline
B. Dilute acetic acid (2%)
C. Protein C. Dilute methanol (1%)
D. Uric acid D. Phosphate buffer solution (0.050 mol/L)
4. Which of the following substances will not increase the 9. Which of the following results from synovial fluid anal-
turbidity of synovial fluid? ysis indicates a joint disease process?
A. Fat A. A few synoviocytes present in the fluid
B. Crystals B. A WBC count lower than 200 cells/μL
- C. Hyaluronate C. An RBC count lower than 2000 cells/μL
D. WBCs - D. A differential count showing greater than 25%
5. Abnormally decreased viscosity in synovial fluid results from neutrophils
A. mucin degradation by leukocytic lysosomes. 10. Differentiation of synovial fluid crystals, based on their
B. overproduction of synovial fluid by synoviocytes. birefringence, is achieved using
C. autoimmune response of synoviocytes in joint disease. A. transmission electron microscopy.
-
D. depolymerization of hyaluronate by neutrophilic B. phase-contrast microscopy.
enzymes. C. direct polarizing microscopy.

-D. compensated polarizing microscopy.


CHAPTER 11 Synovial Fluid Analysis 285

11. The microscopic examination of synovial fluid for crys- 15. Which of the following findings provides a definitive
tals can be difficult because diagnosis of a specific joint condition?
1. numerous artifacts are also birefringent. - A. Staphylococcal bacteria identified by Gram stain

E
2. few crystals may be present. B. Corticosteroid crystals identified during the micro-
3. free-floating crystals can become enmeshed or scopic examination
hidden in fibrin. C. A plasma–synovial fluid glucose difference exceeding
4. different crystals can closely resemble each other 20 mg/dL
morphologically. D. Greater than 25 WBCs/μL observed during the
A. 1, 2, and 3 are correct. microscopic examination
B. 1 and 3 are correct. 16. Analysis of a synovial fluid specimen reveals the following:
C. 4 is correct. • Cloudy, yellow-green fluid of low viscosity
- D. All are correct. • Total leukocyte count of 98,000 cells/μL
12. Which of the following crystals characteristically occurs • Plasma–synovial fluid glucose difference of 47 mg/dL
in patients with gout? Based on the information provided and Table 11.2, this
A. Cholesterol crystals -

specimen most likely would be classified as


B. Hydroxyapatite crystals -
A. noninflammatory. -

yellow
✓C. Monosodium urate crystals B. inflammatory. _ yellow white
D. Calcium pyrophosphate dihydrate crystals /C. septic. -

yellow green
13. In synovial fluid, which of the following crystals is not D. hemorrhagic. - red / brown
birefringent? 17. An analysis of a synovial fluid specimen reveals the
A. Cholesterol crystals - chronic inHg following:

÷
B. Hydroxyapatite crystals • Yellow fluid of high viscosity

-

C. Monosodium urate crystals gouty • Total leukocyte count of 300 cells/μL


D. Calcium pyrophosphate dihydrate crystals pseudo • Plasma–synovial fluid glucose difference of 17 mg/dL
14. Assuming that a patient is fasting, which of the Based on the information provided and Table 11.2, this
following analytes is normally present in the synovial specimen would most likely be classified as
fluid in essentially the same concentration as in the blood
plasma?
/ A. noninflammatory.
B. inflammatory.
1. Glucose C. septic.
2. Lactate D. hemorrhagic.
3. Uric acid
4. Protein
A. 1, 2, and 3 are correct.
B. 1 and 3 are correct.
C. 4 is correct.
D. All are correct.
286 CHAPTER 11 Synovial Fluid Analysis

Case 11.1

0
A 51-year-old man has painful swelling in both knees. He is hos- 1. List any abnormal results.
pitalized, and an arthrocentesis is scheduled for the next morn- 2. Calculate the plasma–synovial fluid glucose difference.30mg / dl
ing. In the morning, a fasting blood sample is drawn for routine 3. Based on the results obtained, this synovial fluid specimen
chemistry tests. Synovial fluid is aspirated from the right knee should be classified as
and submitted to the laboratory. A. normal.
B. noninflammatory (group I).
Blood Chemistry Results C. inflammatory (group II). -
Reference Interval D. septic (group III).
E. hemorrhagic (group IV).
Glucose, fasting: 85 mg/dL !110 mg/dL
4. What is the most likely identity of the crystals observed in the
-
Uric acid: 12.7 mg/dL 2.6–8.0 mg/dL
synovial fluid?
Synovial Fluid Results A. Cholesterol
Physical Microscopic Chemical B. Corticosteroid
Examination Examination Examination C. Hydroxyapatite
D. Calcium pyrophosphate dihydrate

=
888
Color: yellow WBC count: 43,000 cells/μL Glucose: E. Monosodium urate
Clarity: Differential count: 55 mg/dL 5. These results are most consistent with a diagnosis of
cloudy Monocytes: 24% Uric acid: A. gouty arthritis.
Viscosity: Lymphocytes: 13% 12.4 mg/dL B. pseudogout.
decreased Neutrophils: 63% Total protein: C. rheumatoid arthritis.
Crystals: many 4.0 g/dL D. bacterial infection.
intracellular Lactate: E. traumatic arthritis, with previous corticosteroid injection.
needle-shaped 19 mg/dL
=
6. If no crystals were observed in the microscopic examination,
crystals; negative would you change the diagnosis? Why or why not?
birefringence
Gram stain: no organisms
seen; many leukocytes
present

Case 11.2
A 37-year-old woman has persistent and painful swelling in her 1. List any abnormal results.
left knee several days after arthroscopic repair of a torn ligament 2. Calculate the plasma–synovial fluid glucose difference. 44mg 1dL
(i.e., medial meniscus). A fasting blood sample is drawn for rou- 3. Based on the results obtained, this synovial fluid specimen
tine chemistry tests. Arthrocentesis is performed, and the syno- should be classified as
vial fluid is submitted to the laboratory. A. normal.
B. noninflammatory (group I).
Blood Chemistry Results C. inflammatory (group II).
Reference Interval D. septic (group III). -
E. hemorrhagic (group IV).
Glucose, fasting: 79 mg/dL !110 mg/dL
4. These results are most consistent with a diagnosis of
Uric acid: 6.2 mg/dL 2.6–8.0 mg/dL
A. gouty arthritis.
Synovial Fluid Results B. pseudogout.
C. rheumatoid arthritis.
Physical Chemical
D. bacterial infection.
/
Examination Microscopic Examination Examination
E. traumatic arthritis, with previous corticosteroid injection.
Color: yellow WBC count: 97,000 cells/μL Glucose:
Clarity: cloudy Differential count: 35 mg/dL
Viscosity: Monocytes: 13% Uric acid:
decreased Lymphocytes: 5% 5.9 mg/dL

8
Neutrophils: 82% Total protein:
Crystals: none 5.3 g/dL
present Lactate:
•Gram stain: gram-positive 35 mg/dL
cocci present; many
leukocytes present
CHAPTER 11 Synovial Fluid Analysis 287

Case 11.3

÷
A 25-year-old professional football player is in an automobile 1. List any abnormal results.
accident. His recovery is complete except for persistent swelling 2. Calculate the plasma–synovial fluid glucose difference. 15mg / dl
in his left knee. This same knee had been a chronic problem 3. Based on the results obtained, this synovial fluid specimen
before the accident, and he had received a corticosteroid intraar- should be classified as
ticular injection 6 months earlier. An arthrocentesis is scheduled, A. normal.
along with the collection of a fasting blood sample for routine B. noninflammatory (group I).
chemistry tests. The blood and synovial fluid are submitted to C. inflammatory (group II).
the laboratory. D. septic (group III).
E. hemorrhagic (group IV).
Blood Chemistry Results 4. Based on the results obtained, what is the most likely identity
Reference Interval of the crystals observed in the synovial fluid?
A. Cholesterol
Glucose, fasting: 95 mg/dL !110 mg/dL
B. Corticosteroid
Uric acid: 5.7 mg/dL 2.6–8.0 mg/dL
C. Hydroxyapatite
Synovial Fluid Results D. Calcium pyrophosphate dihydrate
Physical Microscopic Chemical E. Monosodium urate
Examination Examination Examination 5. These results are most consistent with a diagnosis of

078
A. gouty arthritis.
Color: yellow WBC count: 950 cells/μL Glucose: B. pseudogout.
Clarity: Differential count: 80 mg/dL C. rheumatoid arthritis.
slightly Monocytes: 65% Uric acid: D. bacterial infection.
cloudy Lymphocytes: 17% 5.7 mg/dL E. traumatic arthritis, with previous corticosteroid injection.
Viscosity: Neutrophils: 18% Total protein:
normal Crystals: few needle- 5.5 g/dL
shaped crystals; Lactate:
negative birefringence 21 mg/dL
Gram stain: no organisms
seen; few leukocytes
present
12
Seminal Fluid Analysis

LEARNING OBJECTIVES
After studying this chapter, the student should be able to: normal range for each parameter, and relate each function
1. Discuss the composition of seminal fluid and briefly to male fertility:
describe the function of each of the following structures • Agglutination
in seminal fluid formation: • Concentration
• Epididymis • Morphology
• Interstitial cells of Leydig • Motility
• Prostate gland • Viability
• Seminal vesicles 6. Identify and describe the morphologic appearance
• Seminiferous tubules of normal and abnormal forms of spermatozoa.
2. Outline the maturation of sperm (spermatozoa) and 7. Discuss the origin and clinical significance of cells
identify the morphologic structures in which each other than sperm in the seminal fluid.
maturation phase occurs. 8. Discuss briefly the role of quantifying the following
3. Summarize the collection of seminal fluid for analysis, biochemical substances in seminal fluid and identify the
including the importance of timing and recovery of the structure evaluated by each substance:
complete specimen. • Acid phosphatase
4. Describe the performance of the physical examination • Citric acid
(appearance, volume, and viscosity) of seminal fluid • Fructose
and the results expected from a normal specimen. • pH
5. Describe the procedures used to evaluate the following • Zinc
characteristics of sperm in seminal fluid, state the

CHAPTER OUTLINE
Physiology, 289 Morphology, 294
Specimen Collection, 291 Automated Semen Analysis Systems, 295
Physical Examination, 291 Vitality, 295
Appearance, 291 Cells Other Than Spermatozoa, 296
Volume, 291 Agglutination, 296
Viscosity, 292 Chemical Examination, 296
Microscopic Examination, 292 pH, 296
Motility, 292 Fructose, 297
Concentration and Sperm Count, 293 Other Biochemical Markers, 297
Postvasectomy Sperm Counts, 293

K E Y T E R M S*
epididymis seminal vesicles
interstitial cells of Leydig seminiferous tubules
liquefaction sperm (also called spermatozoa)
prostate gland vasectomy
seminal fluid (also called semen) viscosity

*Definitions are provided in the chapter and glossary.

288
298 CHAPTER 12 Seminal Fluid Analysis

BIBLIOGRAPHY Keel BA, Quinn P, Schmidt CF, et al: Results of the American
Association of Bioanalysts national proficiency testing
Amelar RD: The semen analysis, Infertility in men: diagnosis and programme in andrology, Hum Reprod 15:680, 2000.
treatment, Philadelphia, 1966, FA Davis. Makler A: The improved ten-micrometer chamber for rapid sperm
Barrosos G, Mercan R, Oxgur K, et al: Intra- and inter-laboratory count and motility evaluation, Fertil Steril 33:337, 1980.
variability in the assessment of sperm morphology by Overstreet JW, Katz DF: Semen analysis, Urol Clin North Am
strict criteria: impact of semen preparation, staining 14:441, 1987.
techniques and manual versus computerized analysis, Hum Tomlinson MJ, Kessopoulou E, Barratt CL: The diagnostic and
Reprod 14:2036, 1999. prognostic value of traditional semen parameters, J Androl
Freund M: Standards for the rating of human sperm morphology, Int 20:588, 1999.
J Fertil 11:97, 1966. World Health Organization: (WHO) laboratory manual for the
Jeyendran RS: Sperm collection and processing: a practical guide, examination of human semen and sperm-cervical mucus
New York, 2003, Cambridge University Press. interaction, ed 4, New York, 1999, Cambridge University Press.

STUDY QUESTIONS
1. Seminal fluid analysis is routinely performed to evaluate 6. Which of the following is a requirement when collecting
which of the following? semen specimens?
A. Prostate cancer A. The patient should abstain from sexual intercourse
B. Postvasectomy status for at least 2 days after the collection.
C. Penile implant status B. Only complete collections of the entire ejaculate are
D. Premature ejaculation acceptable for analysis.
2. Which of the following structures contribute(s) secretions C. A single semen specimen is sufficient for the evalu-
to semen? ation of male fertility.
1. Epididymis D. Semen specimens must be evaluated within 3 hours
2. Prostate gland after collection.
3. Seminal vesicles 7. Which of the following conditions adversely affects the
4. Seminiferous tubules quality of a semen specimen?
A. 1, 2, and 3 are correct. A. The use of Silastic condoms
B. 1 and 3 are correct. B. The time of day the collection is obtained
C. 4 is correct. C. The collection of the specimen in a glass container
D. All are correct. D. The storage of the specimen at refrigerator temperatures
3. Which of the following structures performs an endocrine 8. Which of the following statements regarding semen is true?
and an exocrine function? A. Semen usually coagulates within 30 minutes after
A. Testes ejaculation.
B. Epididymis B. For semen to liquefy before 60 minutes is abnormal.
C. Prostate gland C. After liquefaction, the viscosity of normal semen is
D. Seminal vesicles similar to that of water.
4. The primary function of semen is to D. After liquefaction, the presence of particulate matter
A. nourish the spermatozoa. is highly indicative of a bacterial infection.
B. coagulate the ejaculate. 9. Which of the following statements regarding the manual
C. transport the spermatozoa. evaluation of sperm motility is not true?
D. stimulate sperm maturation. A. Sperm motility most often is graded subjectively.
5. Match the number of the structure to the feature that B. Sperm motility is affected adversely by temperature.
best describes it. Only one structure is correct for each C. Sperm motility assesses speed and forward
feature. progression.
D. Sperm motility should be evaluated initially and at
Descriptive Feature Structure 2 hours after collection.
4 A. Produces and secretes
__ 1. Bulbourethral 10. Which of the following statements regarding sperm con-
__
7- B. Site of spermatogenesis testosterone gland centration is true?
__
3 C. Concentrates and stores 2. Ejaculatory duct A. Sperm concentration within a single individual is
sperm 3. Epididymis
usually constant.
__
b- D. Secretes fluid rich in zinc 4. Interstitial cells of
__
6 E. Secretes fluid high in Leydig
B. Sperm concentration depends solely on the period of
fructose 5. Prostate gland abstinence.
8
__ F. Transports sperm to the 6. Seminal vesicles C. In a normal ejaculate, sperm concentration ranges
ejaculatory duct 7. Seminiferous tubules from 20 million to 250 million per milliliter.
8. Vas deferens D. For fertility purposes, sperm concentration is more
important than sperm motility.
CHAPTER 12 Seminal Fluid Analysis 299

11. Which of the following statements regarding sperm mor- 15. Fructose in semen assists in the evaluation of which of the
phology is true? following?
A. Sperm morphology is usually evaluated using a 1. The secretory function of the seminal vesicles
peroxidase stain. 2. The functional integrity of the epididymis
B. Stained smears of fresh semen can be used to evaluate 3. The functional integrity of the vas deferens
sperm morphology. 4. The secretory function of the prostate gland
C. Sperm morphology is evaluated using 400! (high- A. 1, 2, and 3 are correct.
power) magnification. B. 1 and 3 are correct.
D. Normal semen contains at least 80% sperm with nor- C. 4 is correct.
mal morphology. D. All are correct.
12. Which of the following parameters directly relates to and 16. Which of the following substances can be used to evaluate
provides a check of the sperm motility evaluation? the secretory function of the prostate gland?
A. Agglutination evaluation A. Carnitine
B. Concentration determination B. Fructose
C. Morphology assessment C. pH
D. Vitality assessment D. Zinc
13. Microscopically, immature spermatogenic cells are often 17. The concentration of which of the following substances
difficult to distinguish from can be used to positively identify a fluid as seminal fluid?
A. bacteria. A. Acid phosphatase
B. erythrocytes. B. Citric acid
C. leukocytes. C. Fructose
D. epithelial cells. D. Zinc
14. A semen pH greater than 7.8 is associated with
A. premature ejaculation.
B. obstruction of the vas deferens.
C. abnormal seminal vesicle function.
D. infection of the male reproductive tract.

Case 12.1 Case 12.2


A 36-year-old man and his 32-year-old wife are undergoing an A semen specimen is collected by a 45-year-old man for eval-
evaluation for infertility. A semen specimen is collected at uation of a vasectomy performed 12 weeks earlier.
home and is brought to the laboratory for routine testing.
Semen Analysis
Semen Analysis
Physical Examination Microscopic Examination
Physical Examination Microscopic Examination Color: white Motility: 50%
Color: gray Motility: 70% Volume: 3 mL -
Concentration: 1 ! 106 sperm/mL
Volume: 4.5 mL -
Concentration: 15 ! 106 sperm/mL Liquefaction: 40 minutes Morphology: 80% normal
Liquefaction: 50 minutes Morphology: 70% normal Viscosity: 0 (watery) Vitality: 60%
Viscosity: 0 (watery) Vitality: 60%
Leukocytes: 0.8 ! 106 cells/mL →
Leukocytes: 2 ! 106 cells/mL
Other: moderate bacteria
1. List any abnormal or discrepant results. 1. List any abnormal or discrepant results.
2. Do any of the results obtained suggest improper specimen 2. Do any of the results obtained suggest improper specimen
collection or laboratory error? collection or laboratory error?
3. Are any of the results obtained associated with male 3. After an appropriate time interval, how many sperm should
infertility? be present in seminal fluid after a successful vasectomy?
4. Based on these results, what chemical test should be per-
formed to evaluate the functional integrity of the seminal
vesicles and ejaculatory ducts?
13
Analysis of Vaginal Secretions

LEARNING OBJECTIVES
After studying this chapter, the student should be able to: 3. Discuss vaginal secretion results associated with health,
1. Discuss the collection and proper handling of vaginal including the pH and microscopic entities.
secretion specimens. 4. Compare and contrast the causes, clinical features,
2. Describe the performance of each of the following tests typical vaginal secretion results, and treatments in the
and discuss the clinically significant entities: following conditions:
• Wet mount examination • Bacterial vaginosis
• Amine test • Candidiasis
• KOH preparation and examination • Trichomoniasis
• Atrophic vaginitis

CHAPTER OUTLINE
Specimen Collection and Handling, 301 Candidiasis, 306
pH, 302 Trichomoniasis, 306
Microscopic Examinations, 302 Atrophic Vaginitis, 307
Wet Mount Examinations, 302 Pregnancy-Associated Tests, 307
KOH Preparation and Amine Test, 304 Fetal Fibronectin, 307
Clinical Correlations, 305 Placental Alpha Microglobulin-1, 308
Bacterial Vaginosis, 305

K E Y T E R M S*
bacterial vaginosis KOH preparation
bactericidal vaginal fornix
cervicovaginal secretions vaginal pool
clue cells vaginitis
dyspareunia vulvovaginitis
dysuria

*Definitions are provided in the chapter and glossary.

The most common gynecologic complaints encountered by tests are simple and easy to perform, the accuracy of the
health care providers are vaginal discharge, vaginal discom- results obtained depends directly on the skill and expertise
fort, and vaginal odor. The three major causes for these symp- of the microscopist. This fact should not be minimized. If per-
toms are bacterial vaginosis, candidiasis, and trichomoniasis. sonnel with the necessary technical skills and expertise are
Whereas the causative agent for each of these conditions is unavailable, testing should be referred to a laboratory with
distinctly different, the clinical presentations can be nonspe- qualified personnel.
cific and similar (Table 13.1). Because treatment can differ The Clinical Laboratory Improvement Act has classified
significantly, determining the causative agent before initiating the wet mount examination and the KOH examination of
therapy is important, and in some cases, treating sexual part- vaginal secretions as provider-performed microscopy tests.
ners is also necessary to avoid reinfection. The act also states that when nonlaboratory personnel (i.e.,
These conditions are usually differentiated using a sample clinical practitioners such as physicians, physician assistants,
of vaginal secretions and a few direct microscopy tests: wet nurse practitioners, and nurses) perform these tests, the des-
mount examination, amine or “whiff” test, KOH examina- ignated laboratory director is responsible for ensuring the
tion, and Gram stain. Despite the fact that these microscopy accuracy and reliability of the testing performed. Timely

300
310 CHAPTER 13 Analysis of Vaginal Secretions

20. Krieger JN, Tam MR, Stevens CE, et al.: Diagnosis of 24. Goldenberg RL, Mercer BM, Iams JD, et al: The preterm
trichomoniasis: comparison of conventional wet-mount prediction study: patterns of cervicovaginal fetal fibronectin as
examination with cytologic studies, cultures, and monoclonal predictors of spontaneous preterm delivery, Am J Obstet
antibody staining of direct specimens, JAMA 259:1223–1227, Gynecol 177:8–12, 1997.
1988. 25. American College of Obstetrics and Gynecology: ACOG
21. DeMeo LR, Draper DL, McGregor JA, et al: Evaluation of a Practice Bulletin. Assessment of risk factors for preterm birth,
deoxyribonucleic acid probe for the detection of Trichomonas Obstet Gynecol 98:709–716, 2001.
vaginalis in vaginal secretions, Am J Obstet Gynecol 26. Rapid fFN Cassette Kit product insert, REF 01200, AW-03520-
174:1339–1342, 1996. 002 Rev. 003, 2009. Hologic, Inc. 1240 Elko Drive, Sunnyvale,
22. Matsura H, Hakomori SI: The oncofetal domain of fibronectin CA 94089-2212. www.hologic.com.
defined by monoclonal antibody FDC-6: its presence in 27. Lee SE, Park JS, Norwitz ER, Kim KW, Park HS, Jun JK:
fibronectins from fetal and tumor tissues and its absence in those Measurement of placental alpha-microglobulin-1 in
from normal adult tissues and plasma, Proc Natl Acad Sci cervicovaginal discharge to diagnose rupture of
82:6517–6521, 1985. membranes, Obstet Gynecol 109:634–640, 2007.
23. Ashwood ER, Grenache DG, Lambert-Messerlian G: Pregnancy 28. AmniSure ROM (rupture of [fetal] membranes) test
and its disorders. In Burtis CA, Ashwood ER, Bruns DE, editors: product insert, 1081763 Rev.03, 2014. AmniSure
Tietz textbook of clinical chemistry and molecular diagnostics, International LLC, 24 School Street, Boston, MA 02108. www.
ed 5, St. Louis, 2012, Elsevier Saunders. amnisure.com.

STUDY QUESTIONS
1. Which of the following devices should be used to collect a 6. Which of the following statements best describes a clue cell?
sample of vaginal secretions? A. Degenerating squamous epithelial cells with distinc-
A. Cervical brush on a Teflon shaft tive keratohyalin granulation
B. Cotton-tipped swab on a wooden shaft B. Budding yeast (e.g., blastoconidia) with small cocco-
C. Polyester-tipped swab on a plastic shaft bacilli adhering to their surfaces
D. Wool-tipped swab on a wooden shaft C. Squamous epithelial cells with numerous bacteria
2. Which of the following organisms is adversely affected if adhering to their outer cell membranes
a vaginal secretion specimen is refrigerated? D. White blood cells with numerous bacteria completely
A. Chlamydia trachomatis covering them such that they appear as floating
B. Candida albicans spherical orbs of bacteria
C. Gardnerella vaginalis 7. Which of the following vaginal secretion results correlate
D. Trichomonas vaginalis with health?
3. Which range of pH values is associated with secretions A. pH 3.9; white blood cells, 3 +
from a healthy vagina? B. pH 4.2; white blood cells, 1 +
A. 3.8 to 4.5 C. pH 4.8; white blood cells, rare
B. 4.5 to 5.8 D. pH 5.5; white blood cells, 2 +
C. 5.8 to 6.5 8. Which of the following statements best describes the
D. 7.0 to 7.4 microbial flora of a healthy vagina?
4. Which of the following elements is considered abnormal A. Large gram-positive rods predominate.
when present in vaginal secretions? B. Large gram-positive cocci predominate.
A. Bacteria C. Small gram-negative rods predominate.
B. Pseudohyphae D. Small gram-variable coccobacilli predominate.
C. Yeast 9. Which of the following tests is most helpful in differen-
D. White blood cells tiating red blood cells from yeast in vaginal secretions?
5. Which of the following organisms and substances is A. pH
responsible for the normal pH of the vagina? B. Amine test
A. Gardnerella vaginalis and its metabolic byproduct C. Wet mount examination
succinic acid D. KOH preparation and examination
B. Lactobacilli spp. and their metabolic byproduct 10. Which of the following vaginal secretion findings is most
lactic acid diagnostic for bacterial vaginosis?
C. Mobiluncus spp. and their metabolic byproduct A. pH 5.0
acetic acid B. Clue cells
D. Prevotella spp. and their metabolic byproduct C. Pseudohyphae
phenylacetic acid D. Parabasal cells
CHAPTER 13 Analysis of Vaginal Secretions 311

11. Which of the following substances is responsible for the 14. Which of the following proteins is used as a marker of
foul, fishy odor obtained when the “whiff” test is per- rupture of membranes in pregnant women?
formed on vaginal secretions? A. Fetal fibrinogen
A. Lactic acid B. Fetal fibronectin
B. Polyamine C. Alpha-1 microglobulin
C. Trimethylamine D. Placental alpha microglobulin-1
D. Hydrogen peroxide 15. Scenario: A pregnant woman of 30 weeks’ gestation
12. Select the condition that correlates best with the follow- complains of low back pain and occasional abdominal
ing vaginal secretion results: cramping. Which of the following tests is indicated and
why?
pH: 5.9 A. Fetal fibronectin test to determine whether mem-
Amine test: positive branes are ruptured
KOH examination: negative
B. Fetal fibronectin test to determine whether at risk of
Wet mount examination: bacteria, mixed bacterial flora
WBC: 4+
preterm delivery
C. Placental alpha microglobulin-1 test to determine
A. Normal, indicating a healthy vagina whether at risk of preterm delivery
B. Bacterial vaginosis D. Placental alpha microglobulin-1 test to determine
C. Candidiasis whether membranes are ruptured
D. Trichomoniasis
13. Select the condition that correlates best with the follow-
ing vaginal secretion results:

pH: 4.6
Amine test: negative
KOH examination: negative
Wet mount examination: bacteria, large rods predominate
WBC: 1+
A. Normal, indicating a healthy vagina
B. Bacterial vaginosis
C. Candidiasis
D. Trichomoniasis

Case 13.1
A 49-year-old perimenopausal female is seen by her gynecolo- 1. Based on the patient information and the vaginal fluid results
gist for a routine annual Pap smear. Before the examination, provided, what is the most likely diagnosis?
the health care provider asked if she had any concerns, to which 2. Discuss the formation of the substance responsible for the
the patient stated that she has been noticing a foul vaginal odor, foul odor described by this patient.
particularly after intercourse with her husband. A sample of her 3. Explain why the foul odor is more noticeable after unprotected
vaginal secretions was collected, and the following results were intercourse.
obtained. 4. Briefly describe the development of this disorder in a typical
woman.
Vaginal Secretion Results 5. When performing vaginal secretion analysis, what is the sin-
Wet Mount KOH Preparation and gle most diagnostic finding associated with this condition?
Examination Examination 6. Why are so few white blood cells present in this condition?

pH: 5.0 Amine test: positive


Bacteria: rare large rods, KOH examination: negative
few small rods, many
coccobacilli
WBCs: rare
Other: clue cells present
WBCs, White blood cells.
14
Amniotic Fluid Analysis

LEARNING OBJECTIVES
After studying this chapter, the student should be able to: 5. Compare and contrast the following tests for fetal
1. Discuss amniotic fluid formation and the interactive role pulmonary maturity:
the fetus has in the composition of the amniotic fluid. • Lecithin/sphingomyelin ratio
2. State at least four indications for performing an amniocen- • Phosphatidylglycerol
tesis and the stage in pregnancy best suited for each analysis. • Foam stability index
3. Identify at least four sources of error in amniotic fluid • Lamellar body count
testing caused by inappropriate specimen handling or 6. Describe the analysis of bilirubin in the amniotic fluid
chemical contamination. (ΔA450) and the relationship of this value to fetal status
4. Differentiate amniotic fluid from urine. and the need for medical intervention.

CHAPTER OUTLINE
Physiology and Composition, 312 Differentiation From Urine, 314
Function, 312 Physical Examination, 314
Formation, 313 Color, 314
Volume, 313 Turbidity, 314
Specimen Collection, 313 Chemical Examination, 315
Timing of and Indications for Amniocentesis, 313 Tests to Determine Fetal Lung Maturity, 315
Collection and Specimen Containers, 314 Test to Detect Blood Type Incompatibility, 317
Specimen Transport, Storage, and Handling, 314

K E Y T E R M S*
erythroblastosis fetalis meconium
isoimmune disease oligohydramnios
lamellar bodies polyhydramnios (also called hydramnios)

*Definitions are provided in the chapter and glossary.

With the use of ultrasound, amniocentesis is now a common beyond the scope and intent of this text and therefore are
and relatively safe obstetric procedure. Advancements in not discussed.
technology have provided new technical methods and clinical
applications for amniotic fluid analysis. The study of amniotic PHYSIOLOGY AND COMPOSITION
fluid is performed primarily for three reasons: (1) to enable
antenatal diagnosis of genetic and congenital disorders early Function
in fetal gestation (15 to 18 weeks), (2) to assess fetal pulmo- Amniotic fluid is the liquid medium that bathes a fetus
nary maturity later in the pregnancy (32 to 42 weeks), and throughout its gestation (Fig. 14.1). The amnion, a membrane
(3) to estimate and monitor the degree of fetal anemia caused composed of a single layer of cuboidal epithelial cells, sur-
by isoimmunization or infection. rounds the fetus and is filled with this fluid. Amniotic fluid
By far the most frequently performed tests on amniotic protects the fetus while enabling fetal movement and plays
fluid in the clinical laboratory are used to assess fetal lung an important role in numerous biochemical processes. Fetal
maturity and fetal anemia resulting from an isoimmune dis- cells and many biochemical compounds, such as electrolytes,
ease, which are discussed in this chapter. The specialized lab- nitrogenous compounds, proteins, enzymes, lipids, and hor-
oratory techniques required to detect numerous and varied mones, are present in the amniotic fluid. Although studies
genetic and metabolic disorders using amniotic fluid are have investigated many substances as potential biochemical

312
320 CHAPTER 14 Amniotic Fluid Analysis

12. Dilena BA, Ku F, Doyle I, et al: Six alternative methods to the 16. Liley AW: Liquor amnii analysis in the management of
lecithin/sphingomyelin ratio in amniotic fluid for assessing fetal pregnancy complicated by rhesus sensitization. Am J Obstet
lung maturity. Ann Clin Biochem 34:106, 1997. Gynecol 82:1359, 1961.
13. Neerhof MG, Dohnal JC, Ashwood ER, et al: Lamellar body 17. Queenan JT, Tomai TP, Ural SH, King JC: Deviation in amniotic
counts: a consensus on protocol. Obstet Gynecol 97:318, 2001. fluid optical density at a wavelength of 450 nm in Rh-
14. DeRoche ME, Ingardia CJ, Guerette PJ, et al: The use of lamellar immunized pregnancies from 14 to 40 weeks’ gestation:
body counts to predict fetal lung maturity in pregnancies a proposal for clinical management. Am J Obstet Gynecol
complicated by diabetes mellitus. Am J Obstet Gynecol 187:908, 168:1370, 1993.
2002. 18. Oepkes D, Seaward G, Vandenbussche F, et al: Doppler
15. Ashwood ER, Grenache DG, Lambert-Messerlian G: Pregnancy ultrasonography versus amniocentesis to predict fetal anemia.
and its disorders. In Burtis CA, Ashwood ER, Bruns DE, editors: N Engl J Med 355:156, 2006.
Tietz textbook of clinical chemistry and molecular diagnostics,
ed 5, St. Louis, 2012, Elsevier Saunders.

STUDY QUESTIONS
1. Which of the following is not a function of the amniotic 7. When processing amniotic fluid, high centrifugation
fluid surrounding a developing fetus? speeds are used to clear the fluid of turbidity for
A. Amniotic fluid provides protection of the fetus. -
A. bilirubin analysis. -
B. Amniotic fluid enables fetal movement. B. culturing of fetal cells.
- C. Amniotic fluid is a medium for oxygen exchange.
D. Amniotic fluid is a source of water and solute
C. meconium detection.
D. phospholipid analysis.
exchange. 8. Analysis for which of the following substances can aid in
2. Amniocentesis is usually performed at 15 to 18 weeks’ the differentiation of amniotic fluid from urine?
gestation to determine which of the following conditions? 1. Urea
A. Fetal distress 2. Glucose
B. Fetal maturity 3. Creatinine
r
C. Genetic disorders 4. Protein
D. Infections in the amniotic fluid A. 1, 2, and 3 are correct.
3. Through which of the following mechanism(s) does sol- B. 1 and 3 are correct.
ute and water exchange occur between the fetus and the C. 4 is correct.
amniotic fluid? -
D. All are correct.
1. Fetal swallowing of the amniotic fluid 9. Which of the following statements about amniotic fluid is
=2. Transudation across the fetal skin true?
3. Fetal urination into the amniotic fluid A. Amniotic fluid is normally clear and colorless.
=4. Respiration of amniotic fluid into the fetal pulmo-
-
B. Normally amniotic fluid contains fetal hair, cells, and
nary system vernix.
A. 1, 2, and 3 are correct. C. Amniotic fluid and urine can be differentiated by a
B. 1 and 3 are correct. physical examination of the fluid.
C. 4 is correct. D. When contaminated with meconium, amniotic fluid
/D. All are correct. takes on a yellow or amber coloration.
4. Select the term used to describe a decreased volume of 10. Which of the following isO not a test to evaluate the sur-
-

amniotic fluid present in the amniotic sac. factants present in the fetal pulmonary system?
A. Anhydramnios
B. Hydramnios
T A. ΔA450
B. Lecithin/sphingomyelin ratio
pew 900ns
C. Oligohydramnios C. Phosphatidylglycerol detection
D. Polyhydramnios exceeds to 1200mL D. Foam stability index
5. Amniotic fluid specimens are immediately protected
from light to preserve which of the following substances? lung immaturity?
0
11. Which of the following test results would indicate fetal

1. A lecithin/sphingomyelin ratio of less than 2.0

=
A. Bilirubin
B. Fetal cells 2. A lecithin/sphingomyelin ratio of more than 2.0
C. Meconium 3. A lecithin/sphingomyelin ratio of more than 2.0,
D. Phospholipids with phosphatidylglycerol absent
6. Which of the following substances, when present in 4. A lecithin/sphingomyelin ratio of less than 2.0,
amniotic fluid, is affected adversely by refrigeration? with phosphatidylglycerol present
A. Bilirubin A. 1, 2, and 3 are correct.
-B. Fetal cells B. 1 and 3 are correct.
C. Protein C. 4 is correct.
D. Phospholipids D. All are correct.
CHAPTER 14 Amniotic Fluid Analysis 321

12. Which of the following conditions can cause erythroblas- 14. The ΔA450 value is determined using amniotic fluid from
tosis fetalis? a mother at 20 weeks’ gestation. Which chart(s) should be
A. Immaturity of the fetal liver used to assess this value and the status of the fetus?
B. Decreased amounts of amniotic fluid A. Liley chart
C. Inadequate fetal pulmonary surfactants B. Queenan chart -
D. Maternal immunization by fetal antigens _ C. Either chart can be used.
13. A ΔA450 value that falls into zone III on the Liley chart D. More information is needed.
indicates that the fetus is experiencing
A. no hemolysis.
B. mild hemolysis.
C. moderate hemolysis.
D. severe hemolysis. ,

Case 14.1
A 32-year-old pregnant woman is seen by an obstetrician for the Spectrophotometer scan for ΔA450 determination at 35 weeks’
first time during her third pregnancy. She thinks she is around gestation
33 weeks’ gestation. She is from a Third World country and
3 months ago relocated to the United States with her husband 0.6
and family. A patient history reveals that she has two chil- 0.5
dren—a boy 7 years old and a girl 5 years old. Both births were
normal and uncomplicated; however, she states that her daugh- 0.4
ter had become yellow shortly after birth and that she was given
a blood transfusion. 0.3
Absorbance

Routine prenatal blood work is performed. The mother is deter-


mined to be type O Rh-negative, and an antibody screen reveals 0.2
the presence of an anti-Rh0(D). Her antibody titer is positive to a
1:32 dilution. Her husband is determined to be type A Rh-
positive. To assess and monitor the severity of the suspected 0.1
hemolytic process taking place, weekly amniocenteses are
scheduled.

Amniotic Fluid Results

33 Weeks’ 34 Weeks’ 35 Weeks’ 350 400 450 500 550 600 mm


Gestation Gestation Gestation
1. Calculate the ΔA450 for the amniotic fluid specimen obtained
ΔA450: 0.200 ΔA450: 0.245 Lecithin: 4.7 mg/dL
at 35 weeks’ gestation using the spectrophotometer scan
L/S ratio: 1.1 L/S ratio: 1.5 Sphingomyelin:
provided.
2.3 mg/dL
2. Using the Liley chart in Fig. 14.4, determine the zone in which
PG: absent PG: absent PG: present
the ΔA450 value falls at 35 weeks.
3. Describe the clinical implications that accompany a result in
this zone.
4. Using the values for lecithin and sphingomyelin provided at
35 weeks, calculate the lecithin/sphingomyelin ratio.
5. Based on the fetal lung maturity tests performed each week,
state whether the fetal lungs are mature or immature.
L/S ratio, Lecithin/sphingomyelin ratio; PG, phosphatidylglycerol.
15
Fecal Analysis

LEARNING OBJECTIVES
After studying this chapter, the student should be able to: 12. Describe the macroscopic characteristics of normal feces.
1. Describe the composition and formation of 13. List the major causes of abnormal fecal color, consistency,
normal feces. and odor.
2. Describe the effect of abnormal intestinal water 14. State the primary purpose for the detection of fecal
reabsorption on the consistency of feces. neutrophils.
3. Explain the three physiologic mechanisms that cause 15. Discuss the qualitative assessment of fecal fat using a
diarrhea. microscopic examination and the clinical utility of
4. Differentiate between secretory and osmotic diarrhea quantitative fecal fat tests.
using the fecal osmolality. 16. List at least five causes of blood in feces and state the
5. Identify at least three causes of secretory and osmotic importance of fecal occult blood detection.
diarrhea. 17. Discuss the advantages and disadvantages of the different
6. Compare and contrast the mechanisms of maldigestion indicators used on commercial slide tests for fecal
and malabsorption and the relationship of each to diarrhea. occult blood.
7. Differentiate inflammatory from noninflammatory acute 18. Compare and contrast the following methods for the
diarrhea based on symptoms, diarrheal mechanisms, and detection of fecal blood:
fecal laboratory tests. • Slide tests
8. Identify pathogens associated with acute diarrhea and • Quantitative chemical tests
their mode of transmission. • Immunologic assays
9. Categorize diseases associated with chronic diarrhea as • Radiometric assays
inflammatory or noninflammatory, and state the 19. Describe the chemical principle used for screening feces
predominant diarrheal mechanism. or vomitus for fetal hemoglobin.
10. Differentiate between steatorrhea and diarrhea, and discuss 20. Discuss the effect that disaccharidase deficiency has on
the physiologic conditions that result in steatorrhea. fecal characteristics and formation.
11. Describe the following types of fecal collections and give 21. State two methods for the qualitative detection of
an example of a test requiring each type: abnormal quantities of fecal carbohydrates.
• A random stool collection, with and without dietary 22. State the purpose and describe the principle of the xylose
restrictions absorption test.
• A 3-day fecal collection, with and without dietary
restrictions

CHAPTER OUTLINE
Fecal Formation, 323 Consistency and Form, 328
Diarrhea, 323 Mucus, 328
Acute Diarrhea, 324 Odor, 328
Chronic Diarrhea, 324 Microscopic Examination, 329
Steatorrhea, 326 Fecal White Blood Cells, 329
Specimen Collection, 327 Qualitative Fecal Fat, 330
Patient Education, 327 Meat Fibers, 331
Specimen Containers, 328 Chemical Examination, 331
Type and Amount Collected, 328 Fecal Blood, 331
Contaminants to Avoid, 328 Fetal Hemoglobin in Feces (Apt Test), 333
Gas Formation, 328 Quantitative Fecal Fat, 334
Macroscopic Examination, 328 Fecal Carbohydrates, 334
Color, 328

322
CHAPTER 15 Fecal Analysis 335

carbohydrate malabsorption from carbohydrate maldiges- 4. Drummey GD, Benson JA, Jones GM: Microscopical examination
tion, a xylose absorption test is performed. Xylose is a pentose of the stool for steatorrhea. N Engl J Med 264:85, 1961.
that does not depend on liver or pancreatic function for diges- 5. Mandel JS, Church TR, Bond JH, et al: The effect of fecal occult-
tion and is readily absorbed in the small intestine. Normally, blood screening on the incidence of colorectal cancer. N Engl J
Med 343:1603, 2000.
xylose is not present at significant levels in the blood, and the
6. Ahlquist DA, McGill DB, Schwartz S, et al: HemoQuant: a new
body does not metabolize it. In addition, xylose readily passes
quantitative assay for fecal hemoglobin. Ann Intern Med
through the glomerular filtration barrier and is excreted in the 101:297, 1984.
urine. The xylose absorption test involves the patient’s inges- 7. Rockey DC, Auslander A, Greenberg PD: Detection of upper
tion of a dose of xylose, followed by the collection of a 2-hour gastrointestinal blood with fecal occult blood tests. Am J
blood sample and a 5-hour urine specimen. The concentra- Gastroenterol 94:344, 1999.
tion of xylose is measured in the blood and urine. Depending 8. Hemoccult ICT immunochemical fecal occult blood test (product
on the size of the initial oral dose, at least 16% to 24% of the instructions). Fullerton, CA: Beckman Coulter Inc; 2008.
ingested dose of xylose is normally excreted by adults. 9. Apt L, Downey WS: Melena neonatorum: the swallowed blood
syndrome—a simple test for the differentiation of adult and fetal
hemoglobin in bloody stools. J Pediatr 47:5, 1955.
REFERENCES 10. Van de Kamer JH, Ten Bokel Huinink H, Weyers HW: Rapid
1. Semrad CE: Approach to the patient with diarrhea and method for the determination of fat in feces, J Biol Chem
malabsorption. In Goldman L, Schafer AI, editors: 177:347, 1949.
Goldman-Cecil medicine, ed 25, Philadelphia, 2016, Saunders. 11. Korpi-Steiner N, Ward JN, Kumar V, McConnell JP:
2. Kao YS, Liu FJ: Laboratory diagnosis of gastrointestinal tract and Comparative analysis of fecal fat quantitation via nuclear
exocrine pancreatic disorders. In Henry JB, editor: Clinical magnetic resonance spectroscopy (1H NMR) and gravimetry.
diagnosis and management by laboratory methods, ed 18, Clin Chim Acta 400:33, 2009.
Philadelphia, 1991, Saunders. 12. Heisig DG, Threatte GA, Henry JB: Laboratory diagnosis of
3. Vernon SW: Participation in colorectal cancer screening: a gastrointestinal tract and pancreatic disorders. In Henry JB,
review. J Natl Cancer Inst 89:1406, 1997. editor: Clinical diagnosis and management by laboratory
methods, ed 20, Philadelphia, 2001, Saunders.

STUDY QUESTIONS
1. Which of the following substances is not a component of 5. The inability to convert dietary foodstuffs into readily
normal feces? absorbable substances is called intestinal
A. Bacteria A. inadequacy.
B. Blood B. hypermotility.
C. Electrolytes C. malabsorption.
D. Water D. maldigestion.
2. All of the following actions can result in watery or diar- 6. Intestinal motility is stimulated by each of the following
rheal stools except except
A. decreased intestinal motility. A. castor oil.
B. inhibition of water reabsorption. B. dietary fiber.
C. inadequate time allowed for water reabsorption. C. intestinal distention.
D. an excessive volume of fluid presented for reabsorption. D. sympathetic nerve activity.
3. Lactose intolerance caused by the lack of sufficient lactase 7. Which of the following conditions is characterized by the
primarily presents with excretion of greasy, pale, foul-smelling feces?
A. steatorrhea. A. Steatorrhea
B. osmotic diarrhea. B. Osmotic diarrhea
C. secretory diarrhea. C. Secretory diarrhea
D. intestinal hypermotility. D. Intestinal hypermotility
4. Which of the following tests assists most in the differen- 8. The daily amount of fat excreted in the feces is normally
tiation of secretory and osmotic diarrhea? less than
A. Fecal fat A. 0.7 g.
B. Fecal carbohydrates B. 7.0 g.
C. Fecal occult blood C. 70 g.
D. Fecal osmolality D. 700 g.
336 CHAPTER 15 Fecal Analysis

9. Which of the following tests is used to diagnose 15. Which of the following types of fat require acidification
steatorrhea? and heat before they stain with Sudan III or Oil Red O
A. Fecal fat stain?
B. Fecal carbohydrates 1. Fatty acids
C. Fecal occult blood 2. Cholesterol
D. Fecal osmolality 3. Soaps (fatty acid salts)
10. Which of the following statements about feces is true? 4. Neutral fats (triglycerides)
A. The normal color of feces is primarily due to A. 1, 2, and 3 are correct.
urobilinogens. B. 1 and 3 are correct.
B. The amount of feces produced in 24 hours correlates C. 4 is correct.
poorly with food intake. D. All are correct.
C. The normal odor of feces is usually due to metabolic 16. With the two-slide qualitative fecal fat determination, the
byproducts of intestinal protozoa. first slide produces a normal amount of staining fat pre-
D. The consistency of feces is primarily determined by sent, whereas the second slide, after acid addition and
the amount of fluid intake. heat, produces an abnormally increased amount of fat.
11. Fecal specimens may be tested for each of the following These results indicate
except A. malabsorption.
A. fat. B. maldigestion.
B. blood. C. parasitic infestation.
C. bilirubin. D. disaccharidase deficiency.
D. carbohydrates. 17. Mass screening in adults for fecal occult blood is per-
12. Which of the following substances is responsible for the formed primarily to detect
characteristic color of normal feces? A. ulcers.
A. Bilirubin B. hemorrhoids.
B. Hemoglobin C. colorectal cancer.
C. Urobilins D. esophageal varices.
D. Urobilinogens 18. Which of the following dietary substances can cause a
13. Which of the following statements about fecal tests is false-negative guaiac-based fecal occult blood slide test?
true? A. Fish
A. A fecal fat determination identifies the cause of B. Red meat
steatorrhea. C. Ascorbic acid
B. A fecal leukocyte determination aids in differentiat- D. Fruits and vegetables
ing the cause of diarrhea. 19. Which of the following actions can cause a false-positive
C. A fecal Clinitest identifies the enzyme deficiency that guaiac-based fecal occult blood slide test?
prevents sugar digestion. A. Rehydration of the specimen on the slide before testing
D. A fecal blood screen aids in differentiating bacterial B. Degradation of hemoglobin to porphyrin
from parasitic infestations. C. Storage of fecal specimens before testing
14. Which of the following types of fat readily stain with D. Storage of slides with the specimen already applied
Sudan III or Oil Red O stain? 20. Select the true statement about fecal occult blood tests
1. Fatty acids (FOBTs)?
2. Cholesterol A. Guaiac-based FOBTs are more specific than
3. Soaps (fatty acid salts) immunochemical-based FOBTs.
4. Neutral fats (triglycerides) B. Guaiac-based FOBTs are more expensive than
A. 1, 2, and 3 are correct. immunochemical-based FOBTs.
B. 1 and 3 are correct. C. Dietary restrictions are not required when
C. 4 is correct. immunochemical-based FOBTs are used.
D. All are correct. D. Hemoglobin from nonhuman sources (e.g., red
meat) can cause false-positive results when
immunochemical-based FOBTs are used.
CHAPTER 15 Fecal Analysis 337

21. Which of the following conditions can result in the excre- 23. Which of the following are clinical manifestations of a
tion of small amounts of occult blood in the feces? disaccharidase deficiency?
1. Hemorrhoids 1. A positive fecal Clinitest
2. Bleeding gums 2. Constipation and gas
3. Peptic ulcers 3. A fecal pH of 5.0
4. Intake of iron supplements 4. A positive fecal occult blood test
A. 1, 2, and 3 are correct. A. 1, 2, and 3 are correct.
B. 1 and 3 are correct. B. 1 and 3 are correct.
C. 4 is correct. C. 4 is correct.
D. All are correct. D. All are correct.
22. Which of the following statements regarding the test for 24. Which of the following tests can differentiate inadequate
fetal hemoglobin in feces (the Apt test) is true? carbohydrate metabolism from inadequate carbohydrate
A. Any adult hemoglobin present should resist alkali absorption?
treatment. A. Fecal Clinitest
B. The Apt test is used to differentiate various hemoglo- B. Xylose absorption test
binopathies in the newborn. C. Oral carbohydrate tolerance tests
C. Hemoglobin degraded to hematin usually produces a D. Carbohydrate thin-layer chromatography
positive test result.
D. A pink color after alkali treatment indicates the pres-
ence of fetal hemoglobin.

Case 15.1
A 45-year-old traveling salesman sees his physician and reports Microbiological Examination
diarrhea, weight loss, and back pain for the past month. Physical Stool cultures: negative for Salmonella, Shigella, Campylobacter,
examination reveals yellowing of the sclera of the eyes (jaundice) enteropathogenic Escherichia coli, and Yersinia.
but no hepatomegaly or splenomegaly. Further tests support a Ova and parasites: negative for ova, cysts, and parasites.
diagnosis of pancreatic cancer. The results of a routine urinalysis,
72-hour stool collection for fecal fat, stool for Salmonella/Shigella Blood Chemistry Results
culture and ova and parasites, and xylose absorption follow. Xylose absorption test: normal
1. List any abnormal results.
Urinalysis Results
2. This patient’s condition should be classified as
Physical Chemical Microscopic A. oncotic diuresis.
Examination Examination Examination B. osmotic diarrhea.
C. secretory diarrhea.
8 Color: amber
Clarity: slightly
SG: 1.015
pH: 5.5
RBC/hpf: 0–2 cells
WBC/hpf: 0–2 cells D. intestinal hypermotility.
cloudy Blood: negative Casts/lpf: 0–2 hyaline; 3. What is the term used for an increased amount of fat in
Odor: — o Protein: negative - 2–5 granular the feces?
-
Yellow foam LE: negative Epithelials: few TE/hpf 4. The most likely mechanism responsible for this patient’s diar-
noted. Nitrite: negative rhea is
Glucose: negative A. malabsorption.
Ketones: negative B. maldigestion.
Bilirubin: large C. malexcretion.

8 Ictotest: positive
Urobilinogen: normal
D. malsecretion.
5. Explain the physiologic mechanisms responsible for the
increased fat and acholic stools excreted by this patient.
Fecal Fecal Fecal
Macroscopic Microscopic Chemical 6. Why is this patient’s urine urobilinogen result normal and not
Examination Examination Examination decreased?


Color: pale, clay-colored Leukocytes: absent ◦
Fat: 10 g/day
(acholic)
Consistency: watery, greasy
Form: bulky

hpf, High-power field; LE, leukocyte esterase; lpf, low-power field; RBC, red blood cell; TE, transitional epithelial; WBC, white blood cell.
338 CHAPTER 15 Fecal Analysis

Case 15.2
A 23-year-old woman sees her physician and reports headache, 1. List any abnormal results.
nausea, fever, and diarrhea for the past week. She first experi- 2. Determine the “calculated” fecal osmolality using the for-
enced the diarrhea shortly after a summer picnic. She currently mula: Osmolality ¼ 2 " (Na+fecal + K+fecal).
has five to six bowel movements each day. The stool does not 3. Based on the difference between observed and calculated
appear bloody. A stool specimen is collected and the following osmolality, this patient’s condition would be classified as
test results obtained: A. oncotic diuresis.
B. osmotic diarrhea.
Fecal Fecal Fecal
C. secretory diarrhea.
Macroscopic Microscopic Chemical
D. intestinal hypermotility.
Examination Examination Examination

÷
Color: brown Leukocytes: present Sodium: 65 mmol/L
Consistency: watery Potassium: 98 mmol/L
Osmolality: 340 mOsm/kg

Microbiological Examination of Stool


Culture: Salmonella sp. present.
Ova and parasites: negative for ova, cysts, and parasites.

Case 15.3
A 60-year-old woman is seen by her physician for a routine annual 3. List at least two compounds other than hemoglobin that con-
examination. Her only complaints are a lack of stamina and that tain the heme moiety. Myoglobin.at cytochromes
she tires easily. Routine urinalysis and hematology tests are per- 4. Which of the following conditions could account for the occult
formed. She is sent home with instructions and supplies to collect blood results obtained?
three different fecal specimens for the detection of occult blood. 1. Ulcers
2. Bleeding gums
Urinalysis: normal 3. Hemorrhoids

0
Hematologic Test Reference Fecal Occult 4. Colorectal cancer
Results Range Blood Test A. 1, 2, and 3 are correct.
B. 1 and 3 are correct.
Hemoglobin: 9.8 g/dL Female: 12–16.0 g/dL Specimen #1: positive C. 4 is correct.
Hematocrit: 36% Female: 38%–47% Specimen #2: positive D. All are correct.
Specimen #3: positive 5. Which of the following tests could assist in differentiating an
upper GI bleed from a lower GI bleed?
1. List any abnormal results. A. Apt test
2. Ingestion of which of the following substances can cause a B. Guaiac-based fecal occult blood test
false-positive guaiac-based fecal occult blood test? C. Immunochemical-based fecal occult blood test
1. Fish D. Porphyrin-based fecal occult blood test
2. Bananas 6. In this case the limited information and data suggest
3. Cauliflower A. melena.
4. Vitamin C B. creatorrhea.
A. 1, 2, and 3 are correct. C. gastrointestinal bleeding.
B. 1 and 3 are correct. D. pancreatic cancer.
C. 4 is correct.
D. All are correct.
16
Automation of Urine and Body
Fluid Analysis

LEARNING OBJECTIVES
After studying this chapter, the student should be able to: 4. Compare and contrast the three technologies used to
1. Describe the principle of reflectance photometry. perform fully automated urine microscopy analysis—
2. Discuss and differentiate between semiautomated and digital flow microscopy, flow cytometry, and cuvette-based
fully automated urine chemistry analyzers. digital microscopy.
3. State advantages gained by performing automated urine 5. Discuss the advantages and disadvantages of current
sediment analysis. automated body fluid analyzers.

CHAPTER OUTLINE
Automation of Urinalysis, 339 Automation of Body Fluid Analysis, 348
Urine Chemistry Analyzers, 339 Body Fluid Cell Counts Using Hematology
Automated Microscopy Analyzers, 342 Analyzers, 349
77 Elektronika UriSed Analyzer, 345 Body Fluid Cell Counts Using iQ200, 349
Fully Automated Urinalysis Systems, 346

K E Y T E R M S*
fully automated urinalysis semiautomated
reflectance photometry semiautomated urinalysis

*Definitions are provided in the chapter and glossary.

AUTOMATION OF URINALYSIS As with all technology, new analyzers and methods are
A goal of the urinalysis laboratory is to maximize productivity constantly being developed and modified. The combinations
and testing quality while keeping costs and turnaround time of analyzers or urinalysis workstations available through the
at a minimum. The first reagent strip tests to determine the collaboration of manufacturers are dynamic and change with
chemical composition of urine were developed in the 1950s time. Note that despite our global economy, instruments that
in an effort to achieve these goals. Since that time, reagent are available in Europe or Asia may not be available in the
strips have streamlined the chemical examination, signifi- United States, and vice versa. The instruments presented in
cantly reducing the time required and increasing the number this chapter are limited to those most commonly encountered
of specimens that can be analyzed in a given time period. in US laboratories and one available outside the United States.
Efforts next focused on ensuring consistency in reagent strip Although manufacturers use similar formats for their urine
reading (e.g., color interpretation, timing), reducing the chemical analyzers, the approach used for automated micros-
amount of specimen handling, and increasing specimen copy varies among three principles—digital flow microscopy,
throughput. These efforts have resulted in the development flow cytometry, and cuvette-based digital microscopy.
of instruments that assess reagent strip results and automate
evaluation of the physical characteristics of urine. In the early Urine Chemistry Analyzers
1980s, automation of the microscopic examination was Semiautomation of the chemical examination of urine was
achieved by the development of a urine microscopy analyzer developed to standardize the interpretation of reagent strip
(i.e., Yellow Iris). Today, automated urine chemistry ana- results. Consistent, unbiased, and accurate color interpreta-
lyzers and urine microscopy analyzers are available that can tion was the goal when urine chemistry analyzers were devel-
be used as standalone instruments or linked together to oped. All reagent strip reading instruments, regardless of
enable a fully automated urinalysis system. manufacturer, use reflectance photometry to interpret the

339
350 CHAPTER 16 Automation of Urine and Body Fluid Analysis

bd.com/vacutainer/uap/pdfs/UAP_Tech_Talk_VS8026.pdf. 10. US Food and Drug Administration 510(k) Premarket


Accessed June 25, 2010. Notification: ADVIA 2120/2120i Decision Summary k090346,
7. Fogazzi GB, Garigali G: The urinary sediment by UriSed July 28, 2010: http://www.accessdata.fda.gov/scripts/cdrh/
Technology—a new approach to urinary sediment examination, cfdocs/cfPMN/PMNSimpleSearch.cfm?db¼PMN&
Milan, 2012, Elsevier Srl. ID¼K090346. Accessed June 28, 2011.
8. Clinical and Laboratory Standards Institute (CLSI): Body fluid 11. de Jonge R, Brouwer R, de Graaf MT, et al: Evaluation of the new
analysis for cellular composition: approved guideline, CLSI body fluid mode on the Sysmex XE-5000 for counting leukocytes
Document H56-A, Wayne, PA, 2007, CLSI. and erythrocytes in cerebrospinal fluid and other body fluids.
9. Harris N, Kunicka J, Kratz A: The ADVIA 2120 hematology Clin Chem Lab Med 48:665–675, 2010.
system: flow cytometry-based analysis of blood and body fluids in
the routine hematology laboratory. Lab Hematol 11:47–61, 2005.

STUDY QUESTIONS
1. When semiautomated urine chemistry analyzers are used, 5. The benefits of performing automated urine microscopy
the color that develops on the reaction pads is measured by include all of the following except it
A. spectrophotometry. A. increases precision of microscopy results.
B. reflectance photometry. B. decreases exposure to urine, a potential biohazard.
C. fluorescence photometry. C. increases the time required for the microscopic
D. comparing reaction pads with a color chart. examination.
2. What is the purpose of the color compensation pad on D. decreases manual entry and potential transcription
reagent strips? errors.
A. To compensate for the effect of specific gravity on 6. Which of the following statements about the iQ200
urine color microscopy analyzer is true?
B. To calibrate the instrument for color assessment of A. Particle analysis is performed using flow cytometry.
reaction pads B. Urine particles are automatically classified into 12
C. To account for the contribution of urine color to the categories.
colors on the reaction pads C. Concentrated urine sediments must be prepared
D. To detect substances (e.g., phenazopyridine) that before analysis by the analyzer.
mask color development on the reaction pads D. It cannot be used as a stand-alone instrument (i.e., it
3. Select the true statement regarding reflectance photometry. must be attached to a urine chemistry analyzer
A. The amount of light that is absorbed is detected and for use).
measured. 7. Which of the following statements about the UF-100 and
B. The same wavelength of light is used to evaluate all UF-1000i urine particle analyzers is true?
reaction pads. A. A separate channel is used to detect bacteria.
C. The intensity of light reflected from a polished surface B. Digital images of each urine particle are available for
is quantified. review and archival storage.
D. The relationship between reflectance and concentra- C. The analyzers can specifically identify pathologic casts
tion is not linear. and renal epithelial cells.
4. Select the true statement regarding semiautomated urine D. Impedance technology is the primary method by
chemistry analyzers. which these analyzers detect and categorize particles.
A. Results cannot be automatically transmitted to 8. Which of the following statements is not an issue for the
an LIS. instruments used to perform body fluid analysis?
B. Specific gravity is usually determined by refractive A. Unable to perform five-part WBC differentials
index. B. Have difficulty detecting and enumerating RBCs
C. Urine color and clarity are manually determined and C. Unable to detect and specifically identify malignant
entered into the analyzer. cells
D. Well-mixed uncentrifuged urine is placed onto the D. Unable to perform accurate and precise counting of
intake platform for analysis. low WBC numbers (<20 cells/μL).
17
Body Fluid Analysis:
Manual Hemacytometer Counts and
Differential Slide Preparation

LEARNING OBJECTIVES
After studying this chapter, the student should be able to: 4. Describe step-by-step how to perform a manual cell
1. State four factors that adversely affect manual cell count using a hemacytometer.
counts performed using a hemacytometer. 5. Calculate the cell count in a body fluid when provided
2. Discuss advantages and disadvantages of each diluent with the necessary information.
used to perform body fluid cell counts. 6. Explain cytocentrifugation and its use in preparing
3. Discuss the challenges associated with cell counting slides of body fluid for differential analysis,
of viscous fluids—synovial fluid, semen—including including the equipment needed, advantages,
pretreatment options and their effects if any on cell counts. and disadvantages.

CHAPTER OUTLINE
Using a Hemacytometer, 351 Hemacytometer Calculation Examples, 354
Diluents and Dilutions, 351 Preparation of Slides for Differential, 356
Hemacytometer Cell Counts, 353 Cytocentrifugation, 356
Calculations, 353 Slide Preparations, 358

USING A HEMACYTOMETER provider to provide valid, useful information. This may


include performing a cell count and including on the report
Manual methods using a hemacytometer are often used to a statement such as “Specimen clotted; cell counts must be
perform cell counts on body fluids such as cerebrospinal interpreted with caution.”
fluid, synovial fluid, pleural fluid, pericardial fluid, and peri- Manual cell counts using a hemacytometer are time-
toneal fluid, as well as peritoneal dialysates, bronchoalveolar consuming, require advanced technical skills, have poor pre-
lavages, and semen. In health, the numbers of red blood cells cision (reproducibility), and are subject to numerous errors as
(RBCs) and white blood cells (WBCs) in these body fluids a result of the multiple steps involved. Therefore it is imper-
are low, and other cells or cellular debris can be present. ative that well-trained and technically proficient laboratorians
As discussed in Chapter 16, automated cell counting ana- perform them and that appropriate materials are used to
lyzers can produce erroneous results when the cell count verify the achievement of quality goals.
is low. It is the responsibility of each laboratory to define
its lower limit for cell counts (RBCs and WBCs) and to have
a protocol for performing manual cell counts using a hema- Diluents and Dilutions
cytometer when cell counts are below the laboratory-defined The visual appearance of the body fluid aids in determining
lower limit.1 whether a dilution should be made for cell counting and what
Highly viscous body fluids (e.g., synovial fluid) and fluids dilution should be prepared. Body fluids that are clear do not
that fail to appropriately liquefy (e.g., semen) require pretreat- require a dilution, and the fluid can be loaded directly onto a
ment before cell counting by manual or automated methods. hemacytometer. Fluids that are visibly cloudy or bloody must
Note that cell counts using a clotted body fluid are inaccurate. be diluted to obtain accurate cell counts. Table 17.1 is pro-
Because it may not be possible to obtain another body fluid vided as a guide to dilution selection based on visual appear-
specimen, every effort is made to work with the health care ance. When diluents that do not lyse RBCs are used, a higher

351
358 CHAPTER 17 Body Fluid Analysis

For specimens that have a low protein content (e.g., CSF), The WBC differential can be performed using any area of
adding a drop of 22% albumin to the sample chamber before the cell button. A systematic approach to viewing should be
adding the body fluid enhances adherence of cells to the glass used (similar to that used with blood smears) to prevent erro-
slide and reduces cell distortion (smudging) or disintegration.1,3 neous repeat counting of the same cells. Ideally, 100 to 300
cells should be evaluated.
Slide Preparations
Slide preparations are stained using Wright’s or Wright-Giemsa REFERENCES
stain performed manually or automatically using a slide stainer.
The hand-drawn or premarked circle on the microscope slide 1. Clinical and Laboratory Standards Institute (CLSI): Body fluid
indicates the location of the cell button (see Fig. 17.4). analysis for cellular composition: approved guideline, CLSI
Adjust the microscope to low-power (100 !) magnifica- Document H56-A, Wayne, PA, 2006, CLSI.
2. World Health Organization: WHO laboratory manual for the
tion, and thoroughly scan the entire cell button looking for
examination and processing of human semen, ed 5, Geneva,
cell clumps, which are characteristic of malignancies. Note Switzerland, 2010, World Health Organization.
that malignant cells can be present in low numbers, and even 3. Kjeldsberg CR, Knight JA: Laboratory methods. In Body fluids,
a single malignant cell is clinically significant. Also, not all cell ed 3, Chicago, 1993, American Society of Clinical Pathologists
clumps are composed of malignant cells. Press.

STUDY QUESTIONS
1. Which of the following statements is not associated with the 6. In the pretreatment of a synovial fluid with hyaluronidase,
performance of cell counts using a manual hemacytometer? a 1:10 dilution is made, after which a WBC count is per-
A. The procedure is time-consuming. formed using a 1:20 dilution of this fluid. The WBCs in the
B. Quantitative pipetting is required. four large corner squares (“W”) and the center square are
C. Body fluids with low cell counts cannot be analyzed. counted in each chamber (i.e., 5 mm2). Both sides of the
D. High variability in results is obtained between hemacytometer were evaluated with 37 cells and 43 cells
laboratorians. counted in chamber 1 and chamber 2, respectively. What
2. Which of the following diluents will cause synovial fluid to is the average cell count that should be reported?
form a mucin clot? A. 160 WBCs/μL
A. Hyaluronidase buffer solution B. 1600 WBCs/μL
B. Hypotonic saline C. 16,000 WBCs/μL
C. Isotonic saline D. 160,000 WBCs/μL
D. Turk’s solution 7. A WBC count is performed using a 1:2 dilution of CSF,
3. Which of the following diluents should be used when an and the four large corner squares (“W”) and the center
RBC count is requested? square are counted in each chamber (i.e., 5 mm2). Both
A. Dilute acetic acid sides of the hemacytometer were evaluated with 31 cells
B. Hypotonic saline and 23 cells counted in chamber 1 and chamber 2, respec-
C. Isotonic saline tively. What should be done next?
D. Turk’s solution A. Clean hemacytometer and repeat counts.
4. An air displacement pipette cannot accurately dispense B. Recount both sides of the hemacytometer.
A. CSF. C. Calculate the average WBC count and report.
B. pleural fluid. D. Reload the same dilutions onto a clean hemacytome-
C. peritoneal fluid. ter, and repeat the counts.
D. synovial fluid. 8. Distortions observed on cytocentrifuge slide preparations
5. Which of the following actions will adversely affect the cell have been associated with
count obtained using a hemacytometer? A. viscous fluids.
A. Counting six “W” squares instead of the usual five B. high cell counts.
B. Preparing a dilution of semen using an air displace- C. use of fresh body fluid specimens.
ment pipette D. fluids that have a high protein concentration.
C. Mixing the fluid for 3 minutes before loading it onto
the hemacytometer
D. Making three dilutions but using only two to load the
hemacytometer
18
Microscopy

LEARNING OBJECTIVES
After studying this chapter, the student should be able to: 3. Describe the daily care and preventive maintenance
1. Identify and explain the functions of the following routines for microscopes.
components of a microscope: 4. Compare and contrast the principles of the following
• Aperture diaphragm types of microscopy:
• Condenser • Brightfield
• Eyepiece (ocular) • Phase-contrast
• Field diaphragm • Polarizing
• Mechanical stage • Interference contrast
• Objective • Darkfield
2. Describe K€ohler illumination and the microscope • Fluorescence
adjustment procedure used to ensure optimal specimen 5. List an advantage of and an application for each type
imaging. of microscopy discussed.

CHAPTER OUTLINE
Brightfield Microscope, 360 Care and Preventive Maintenance, 366
Eyepiece, 361 Types of Microscopy, 366
Mechanical Stage, 362 Brightfield Microscopy, 367
Condenser, 362 Phase-Contrast Microscopy, 367
Illumination System, 363 Polarizing Microscopy, 368
Objectives, 363 Interference Contrast Microscopy, 372
Ocular Field Number, 365 Darkfield Microscopy, 374
Microscope Adjustment Procedure, 365 Fluorescence Microscopy, 375

K E Y T E R M S*
aperture diaphragm interference contrast microscopy
birefringent (also called doubly refractile) K€ohler illumination
brightfield microscopy mechanical stage
chromatic aberration numerical aperture
condenser objective
darkfield microscopy parcentered
eyepiece (also called ocular) parfocal
field diaphragm phase-contrast microscopy
field number polarizing microscopy
field of view resolution
fluorescence microscopy spherical aberration

*Definitions are provided in the chapter and glossary.

A high-quality brightfield microscope is required for the microscopes can be modified to allow several types of micros-
microscopic examination of urine and other body fluids. copy from a single instrument—brightfield, phase-contrast,
One must give considerable care to its selection because its polarization—good planning ensures selection of the most
use is an integral part of laboratory work, and microscopes with appropriate instrument. Whereas acquiring a suitable micro-
quality objective lenses are costly. Because some brightfield scope is of utmost importance, appropriate training on its use

359
CHAPTER 18 Microscopy 377

REFERENCES Abramowitz MJ: Darkfield illumination. Am Lab 23:60, 1991.


Brown B: Basic laboratory techniques. In Hematology: principles
1. Nikon Microscopy U: Microscope alignment for K€ohler and procedures, ed 5, Philadelphia, 1988, Lea & Febiger.
illumination (website): http://www.microscopyu.com/tutorials/ Foster B, ASCLS: Optimizing light microscopy for biological and
java/kohler/index.html. Accessed February 9, 2016. clinical laboratories, Dubuque, IA, 1997, Kendall/Hunt
2. Hoffman R: The modulation contrast microscope: principles and Publishing.
performance. J Microsc 110:205–222, 1977. Mollring FK: Microscopy from the very beginning, Oberkochen,
West Germany, 1981, Carl Zeiss.
BIBLIOGRAPHY Olympus Instruction Manual: Differential interference contrast
attachment for transmitted light model BH2-NIC, AX5349,
Abramowitz MJ: Koehler illumination. Am Lab 21:106, 1989. Tokyo, 1988, Olympus Optical Company.
Abramowitz MJ: Microscope objectives. Am Lab 21:81, 1989. Olympus Microscopy Resource Center: Interactive Java tutorials
Abramowitz MJ: The first order red compensator. Am Lab 21:110, (website): http://olympus.magnet.fsu.edu/primer/java/
1989. index.html. Accessed February 9, 2016.
Abramowitz MJ: Fluorescence filters. Am Lab 22:168, 1990. Smith RF: Microscopy and photomicroscopy: a working manual,
Abramowitz MJ: The polarizing microscope. Am Lab 22:72, 1990. ed 2, Boca Raton, FL, 1994, CRC Press.

STUDY QUESTIONS
1. In a brightfield microscope, which lens produces the pri- 6. Match the microscope component with its primary
mary image magnification? function.
A. Condenser
B. Eyepiece (ocular) Microscope
C. Numerical aperture Function Component
7 A. Produces primary image
__ 1. Aperture
D. Objective
2. A microscope has a 10 ! magnification eyepiece and magnification diaphragm
__
3 B. Produces secondary 2. Condenser
a 100! objective lens. What is the total magnification of
image magnification 3. Eyepiece
the specimen when viewed using this microscope? 5 C. Moves the specimen for
__ 4. Field diaphragm
A. 0.1 ! viewing 5. Mechanical stage
B. 10! __
2 D. Optimally focuses light 6. Light source
C. 100! onto the specimen 7. Objective
D. 1000 ! I E. Controls the angle of light
__
3. Select the numerical aperture that has the ability to dis- presented to the specimen
4 F. Controls the diameter of
__
tinguish the smallest distance between two distinct
points, that is, the greatest resolving power (R). light rays that strike the
A. 0.25 specimen
B. 0.65
7. Which of the following components should be adjusted
C. 0.85
to decrease the illumination light or field brightness?
D. 1.25
A. Aperture diaphragm
4. The numerical aperture of a lens can be increased by
B. Condenser
A. decreasing the angle of light made by the lens.
C. Field diaphragm
B. increasing the refractive index of the optical
D. Light source
medium.
8. Which lens characteristic is described as the ability to
C. increasing the illumination intensity.
keep a specimen image in focus regardless of which
D. decreasing the interpupillary distance.
objective lens is used?
A. Parcentered
B. Parfocal
C. Chromatic aberration
5. Which parameter(s) will increase with an increase in the
D. Spherical aberration
numerical aperture of an objective lens?
9. To achieve maximal image magnification and reso-
A. Magnification and resolution
lution, the
B. Field of view and resolution
A. condenser should be in its lowest position.
C. Magnification and field of view
B. condenser numerical aperture must be equal to or
D. Magnification
greater than the objective numerical aperture.
C. aperture diaphragm should be used to decrease field
brightness.
D. field diaphragm should be opened fully.
378 CHAPTER 18 Microscopy

10. Various inscriptions may be found on an objective lens. 15. A birefringent substance is one that
Select the objective lens inscription that indicates a A. vibrates light in all directions.
numerical aperture of 0.25. B. vibrates light at two different wavelengths.
A. SPlan40PL C. refracts light in two different directions.
0.65 D. shifts light one-half wavelength out of phase.
160/025 16. Which type of microscopy is able to produce three-
B. 25 dimensional images and perform optical sectioning?
0.10 A. Brightfield
160/0.17 B. Interference contrast
C. E10 C. Phase-contrast
0.25 D. Polarizing
160/0.20 17. The principle of fluorescence microscopy is based on
D. DPlan25 A. a substance that causes the rotation of polarized light.
0.10 B. differences in the optical light path being converted
25/160 to intensity differences.
11. When a microscope with K€ ohler illumination is adjus- C. differences in refractive index being converted into
ted, the variations in light intensity.
A. condenser is adjusted up or down until the field dia- D. the absorption of light and its subsequent emission at
phragm is focused sharply. a longer wavelength.
B. field diaphragm is opened until it is slightly smaller 18. Converting a brightfield microscope for polarizing
than the field of view. microscopy requires
C. illumination intensity is adjusted using the field and A. two polarizing filters—one placed below the con-
aperture diaphragms. denser and one placed between the objective and
D. aperture diaphragm is opened until 25% of the field is the eyepiece.
in view. B. a special condenser, two polarizing filters, and a Wol-
12. Microscope lenses should be cleaned or polished using laston prism between the objective and the eyepiece.
1. gauze. C. an annular diaphragm in the condenser and a phase-
2. facial tissue. shifting element in the objective.
3. lint-free tissues. D. a slit aperture below the condenser, a polarizing filter,
4. lens paper. and a modulator.
A. 1, 2, and 3 are correct. 19. Which type of microscopy uses a special condenser to
B. 1 and 3 are correct. direct light onto the specimen from oblique angles only?
C. 4 is correct. A. Darkfield
D. All are correct. B. Interference contrast
13. When viewing a focused specimen in the microscope, C. Phase-contrast
the user sees a speck in the field of view. The speck D. Polarizing
remains in view when the objective is changed and when 20. Match the type of microscopy with the characteristic.
the specimen is moved. The speck is most likely located
on the Microscopy
A. condenser. Characteristic Type
2 A. Is the preferred technique for
__ 1. Brightfield
B. eyepiece.
C. objective. identifying spirochetes. 2. Darkfield
3
__ B. Is used often to visualize 3. Fluorescence
D. specimen coverslip.
antigens, antibodies, and 4. Phase-contrast
14. Which type of microscopy converts differences in refrac- viruses. 5. Polarizing
tive index into variations in light intensity to obtain the 6 C. Enables three-dimensional
__ 6. Interference
specimen image? viewing of unstained, low- contrast
A. Brightfield refractile specimens.
B. Interference contrast 5
__ D. Is used to identify negative
C. Phase-contrast and positive birefringence.
D. Polarizing 4 E. Produces less haloing with
__
thin, flat specimens.
APPENDIX C
Reference Intervals

Urine (Random Specimen) Fecal Reference Intervals


Reference Intervals Physical Examination
Physical Examination Color Brown
Consistency Firm, formed
Component Result
Form Tubular, cylindrical
Color Colorless to amber (varies with
state of hydration, diet, health) Chemical Examination
Clarity Clear Total fat, quantitative <6 g/day and <20% of stool
Specific gravity 1.002–1.035 (physiologically (72-hour specimen)
possible 1.002–1.040) Osmolality 285–430 mOsm/kg H2O
Osmolality 275–900 mOsm/kg H2O Potassium 30–140 mmol/L
(physiologically possible Sodium 40–110 mmol/L
50–1400 mOsm/kg H2O)
Microscopic Examination
Volume 600–1800 mL/day (varies with
Fat, qualitative assessment
state of hydration, diet, health)
Neutral fat Few globules present per
Chemical Examination high-power field
Component Result Total fat <100 fat globules (diameter
Bilirubin Negative !4 microns) per high-
Glucose Negative power field
Ketones Negative Leukocytes (qualitative) None present
Leukocyte esterase Negative Meat and vegetable fibers Few
Nitrite Negative (qualitative)
pH 4.5–8.0
Protein Negative
Urobilinogen !1 mg/dL
Microscopic Examination*
Component Amount Magnification
Red blood cells 0–3 Per high-power field
White blood cells 0–8 Per high-power field
Casts 0–2 hyaline Per low-power field
or finely
granular†
Epithelial cells
Squamous Few Per low-power field
Transitional Few Per high-power field
Renal Few (0–1) Per high-power field
Bacteria and yeast Negative Per high-power field
*
Using the UriSystem. Note that values vary with the standardization
system used (i.e., the concentration of urine sediment, the microscope
slide technique, and the optical properties of the microscope).

After physical exercise, cast numbers increase and finely granular
casts are included (Haber, 1991).

385
386 APPENDIX C Reference Intervals

Semen Characteristics Associated With Fertility


Parameter Reference Interval* Lower Reference Limit†
Physical Examination
Appearance Gray-white, opalescent, opaque
Volume 2–5 mL 1.5 mL (1.4–1.7)
Viscosity/liquefaction Discrete droplets (watery) within 60 minutes
Microscopic Examination
Motility 50% or more with moderate to rapid linear (forward) progression 40% (38%–42%)
Concentration 20 to 250 ! 106 sperm per mL 15 ! 106 sperm per mL
Morphology 14% or more have normal morphology 4% normal forms
Vitality 75% or more are alive 58% (55%–63%)
Leukocytes <1 ! 106 per mL
Chemical Examination
pH 7.2–7.8 "7.2
Acid phosphatase (total) "200 U per ejaculate at 37°C (p-nitrophenylphosphate)
Citric acid (total) "52 μmol per ejaculate
Fructose (total) "13 μmol per ejaculate "13 μmol per ejaculate
Zinc (total) "2.4 μmol per ejaculate "2.4 μmol per ejaculate
*
Based on the strict criteria evaluation recommended by the World Health Organization (1999) for assessing sperm morphology for fertility purposes.
The one-sided 5th centile lower reference limit recommended by the WHO (2010) for assessing semen characteristics.

Synovial Fluid Reference Intervals*


Physical Examination
Total volume 0.1–3.5 mL
Color Pale yellow
Clarity Clear
Viscosity High; forms “strings” 4–6 cm long
Spontaneous clot formation No

Microscopic Examination
Erythrocyte count <2000 cells/μL
Leukocyte count <200 cells/μL
Differential cell count
Monocytes and macrophages #60%
Lymphocytes # 30%
Neutrophils #10%
Crystals None present

Chemical Examination
Glucose Equivalent to plasma values†
Glucose: P-SF difference <10 mg/dL†
Uric acid Equivalent to plasma values†
Total protein 1–3 g/dL
Lactate 9–33 mg/dL{
Hyaluronate 0.3–0.4 g/dL
*
Values for fluid obtained from a knee joint.

Synovial fluid values are equivalent to blood plasma values if obtained from a fasting patient.
{
Normal lactate values are assumed to be similar to those in blood and cerebrospinal fluid; actual reference intervals have
yet to be established.
Data from Kjeldsberg CR, Knight JA: Synovial fluid. In Body fluids, ed 2, Chicago, 1986, American Society of Clinical
Pathology Press, pp 129–152.
APPENDIX C Reference Intervals 387

Cerebrospinal Fluid Reference Intervals*


Physical Examination
Component Result
Color Colorless
Clarity Clear
Chemical Examination
Component Conventional Units Conversion Factor SI Units
Electrolytes
Calcium 2.0–2.8 mEq/L 0.5 1.00–1.40 mmol/L
Chloride 115–130 mEq/L 1 115–130 mmol/L
Lactate 10–22 mg/dL 0.111 1.1–2.4 mmol/L
Magnesium 2.4–3.0 mEq/L 0.5 1.2–1.5 mmol/L
Potassium 2.6–3.0 mEq/L 1 2.6–3.0 mmol/L
Sodium 135–150 mEq/L 1 135–150 mmol/L
Glucose 50–80 mg/dL 0.5551 2.8–4.4 mmol/L
Total protein 15–45 mg/dL 10 150–450 mg/L
Albumin 10–30 mg/dL 10 100–300 mg/L
IgG 1–4 mg/dL 10 10–40 mg/L
Protein Electrophoresis Percent of Total Protein
Transthyretin (prealbumin) 2%–7%
Albumin 56%–76%
α1-Globulin 2%–7%
α2-Globulin 4%–12%
β-Globulin 8%–18%
γ-Globulin 3%–12%
Microscopic Examination
Component Conventional Units Conversion Factor SI Units
Neonates (<1 year old) 0–30 cells/μL 106 0 to 30 ! 106 cells/L
1-4 years old 0–20 cells/μL 106 0 to 20 ! 106 cells/L
5-18 years old 0–10 cells/μL 106 0 to 10 ! 106 cells/L
Adults (>18 years old) 0–5 cells/μL 106 0 to 5 ! 106 cells/L

Differential Cell Count Percent of Total Count


Neonates
Lymphocytes 5%–35%
Monocytes 50%–90%
Neutrophils 0%–8%
Adults
Lymphocytes 40%–80%
Monocytes 15%–45%
Neutrophils 0%–6%
*
For cerebrospinal fluid specimens obtained by lumbar puncture.
388 APPENDIX C Reference Intervals

Serous Fluid* Effusion: Differentiation as Transudate or Exudate


Parameter Transudates Exudates
Causes
Increased hydrostatic pressure Increased capillary permeability
Decreased oncotic pressure Decreased lymphatic absorption
Physical Examination
Clarity Clear Cloudy
Color Pale yellow Variable (yellow, greenish, pink, red)
Clots spontaneously No Variable; often yes
Microscopic Examination
WBC count <1000 cells/μL (pleural) Variable, usually
<300 cells/μL (peritoneal) >1000 cells/μL (pleural)
>500 cells/μL (peritoneal)
Differential count Mononuclear cells predominate Early, neutrophils predominate; late, mononuclear cells
Chemical Examination
Bilirubin ratio (fluid-to-serum) "0.6 >0.6
Glucose Equal to serum level " Serum level
TP concentration <50% of serum >50% of serum
TP ratio (fluid-to-serum) "0.5 >0.5
LD activity <60% of serum >60% of serum
LD ratio (fluid-to-serum) "0.6 >0.6
Cholesterol ratio (fluid-to-serum) "0.3 >0.3
LD, Lactate dehydrogenase; TP, total protein; WBC, white blood cell.
*
Serous fluids include pleural, peritoneal, and pericardial fluids.

Serous Fluid* Effusion: Differentiation as Chylous and Pseudochylous


Parameter Chylous Effusion Pseudochylous Effusion
Physical Examination Milky Milky
Chemical Examination
Chylomicrons Present Absent
Triglycerides >110 mg/dL (1.2 mmol/L) <110 mg/dL (1.2 mmol/L)
Cholesterol Usually <200 mg/dL (5.2 mmol/L) Usually >200 mg/dL (5.2 mmol/L)
Microscopic Findings Lymphocytes Variety of cell types
Lipid-laden macrophages
Cholesterol crystals†
Conditions Pleural effusions due to Chronic diseases such as
• Trauma or surgery (caused damage to thoracic duct) • Tuberculosis
• Obstruction of lymphatic system: tumors (lymphomas), • Rheumatoid arthritis
fibrosis • Collagen vascular disease
Peritoneal effusions due to
• Hepatic cirrhosis
• Portal vein thrombosis
*
Serous fluids include pleural, peritoneal, and pericardial fluids.
Presence confirms or establishes fluid as pseudochylous effusion.

APPENDIX D
Body Fluid Diluents and
Pretreatment Solutions

When the diluents and pretreatment solutions provided


2. Gradually add 3.0 mL glacial acetic acid to the flask.
in this section are prepared in the laboratory, appropriate
(Note: ALWAYS add acid to water.)
personal protective equipment should be used. Table D.1 3. After acid addition, add CLRW to bring the volume to the
summarizes the common uses of these diluents and the lim- calibration mark.
itations associated with some body fluids (e.g., synovial fluid). 4. Store at room temperature.

COMMERCIAL ISOTONIC DILUENTS


The commercial diluents used by automated hematology or TURK’S SOLUTION2
body fluid analyzers can be used to prepare manual dilutions This diluent lyses red blood cells (RBCs) and stains nucleated
of body fluids. It is important when obtaining an aliquot for cells. As with other diluents that contain acetic acid, clumping
use that precautions are taken so that the primary container of the body fluid will occur when a high level of protein is
(used by the analyzer) is not contaminated. present. DO NOT use this solution as a diluent for synovial
fluids because it will cause a mucin clot.
SALINE, ISOTONIC (0.85%) OR CAUTION: Acid.
1. To a 100-mL volumetric flask, add approximately 60 mL Clin-
“NORMAL SALINE”
ical Laboratory Reagent Water (CLRW).1
1. Into a 100-mL volumetric flask, add 0.85 g NaCl. 2. Gradually add 3.0 mL glacial acetic acid to the flask.
2. Add approximately 60 mL Clinical Laboratory Reagent (Note: ALWAYS add acid to water.)
Water (CLRW)1 and swirl to dissolve. 3. Add 1 mL of 0.5% methylene blue solution (see instructions
3. Add CLRW to bring the volume to the calibration mark. that follow for making this solution) or 1 mL of 1% gentian
4. Store at room temperature. violet (aqueous solution).
4. Add CLRW to bring volume in flask to the calibration mark.
5. Store at room temperature.

SALINE, HYPOTONIC (0.30%) 0.5% Methylene Blue Solution (Used to Prepare Turk’s
1. Into a 100-mL volumetric flask, add 0.30 g NaCl. Solution)
2. Add approximately 60 mL Clinical Laboratory Reagent CAUTION: Acid.
Water (CLRW)1 and swirl to dissolve. 1. To a 100-mL volumetric flask, add 0.5 g methylene blue.
3. Add CLRW to bring the volume to the calibration mark. 2. Add approximately 60 mL Clinical Laboratory Reagent
4. Store at room temperature. Water (CLRW).1
3. Add 0.5 mL glacial acetic acid to the flask.
(Note: ALWAYS add acid to water.)
4. Swirl flask to mix.
DILUTE ACETIC ACID (3.0%) 5. Add CLRW to bring volume in flask to the calibration mark.
6. Store at room temperature.
This diluent lyses red blood cells (RBCs) and stains nucleated
cells. As with other diluents that contain acetic acid, clumping
of the body fluid will occur when a high level of protein is
present. DO NOT use this solution as a diluent for synovial
fluids because it will cause a mucin clot.
CAUTION: Acid.
1. Into a 100-mL volumetric flask, add approximately 60 mL
Clinical Laboratory Reagent Water. (CLRW).1

389
390 APPENDIX D Body Fluid Diluents and Pretreatment Solutions

SYNOVIAL FLUID SOLUTIONS TABLE D.1 Common Uses and Limitations


Hyaluronidase Pretreatment for Synovial Fluid 3 of Diluents
For the analysis of highly viscous synovial fluids, pretreatment Solution Use Comments
with hyaluronidase may be necessary before performing
Commercial WBC count Diluent used in
cell counts or chemical and immunologic tests. Several
isotonic RBC count hematology analyzers
approaches are available.
diluents for cell counting
1. Approach A: Hyaluronidase (EC 3.2.1.35) alone
Isotonic saline WBC count Also known as “normal”
a. Add approximately 400 units of hyaluronidase (powder or
liquid) to approximately 1 mL of fluid. (0.85%) RBC count saline
b. Mix and incubate at 37°C for 10 minutes.3 Note that if liq- Hypotonic WBC count • Lyses RBCs
uid hyaluronidase is used, the volume used must be saline
accounted for in the cell count calculations. (0.30%)
2. Approach B: Buffered hyaluronidase2 Dilute acetic WBC count • Lyses RBCs
For each milliliter of synovial fluid, add 1 drop of 0.05% acid (3.0%)* • Do not use with
buffered hyaluronidase. Mix and incubate at room temper- synovial fluids; causes
ature for 4 minutes.2 mucin clot and cell
clumping
0.05% Buffered Hyaluronidase Turk’s solution WBC count • Lyses RBCs
a. Into a 100-mL volumetric flask, add 50 mg Type 1-S hyal- • Do not use with
uronidase (EC 3.2.1.35). synovial fluids; causes
b. Add 0.912 g potassium phosphate monobasic (FW mucin clot and cell
136.09). clumping
c. Add approximately 60 mL Clinical Laboratory Reagent
Hyaluronidase Pretreatment Reduces viscosity,
Water (CLRW)1 and swirl to dissolve.
pretreatment of synovial enabling accurate cell
d. Add CLRW to bring the volume to the calibration mark.
solutions fluid counts and crystal
e. Store at 2°C to 5°C.
detection/identification
3. Approach C: Add 0.1 mL Type 1-S hyaluronidase (EC
3.2.1.35) to 0.9 mL synovial fluid. Mix well and incubate at Hyaluronidase Synovial fluid • Prevents mucin clot
37°C for 4 hours.2 (0.1 g/L) WBC count formation in synovial
diluent fluids
Hyaluronidase (0.1 g/L) Diluent for Cell Counts • Stain enhances
in Synovial Fluid2 nucleated cell
Before diluting a synovial fluid, the specimen should be mixed identification
for 5 to 10 minutes.3 Note that for turbid specimens, the base Semen Pretreatment Reduces viscosity of
of the tube should be flicked several times to dislodge cells pretreatment of semen semen that failed to
before mixing. For accurate cell counts, turbid specimens solutions liquefy to enable
must be thoroughly mixed. accurate sperm counts
To prepare 100 mL of hyaluronidase diluent (and cell counts, when
1. Prepare 0.067 mol/L phosphate buffer as follows: performed)
a. Prepare 250 mL of Solution A: In a 250-mL volumetric Semen diluent Sperm count Prepare with or without
flask, dissolve 2.279 g monobasic potassium phosphate stain; stain enhances
(MW 136.09) in Clinical Laboratory Reagent Water visualization of sperm
(CLRW).1 *
b. Store at 2°C to 5°C. Stable for 3 months. Other concentrations of acetic acid can also be used (e.g., 5%, 10%).
Note: At all times when reagents are prepared, appropriate personal
c. Prepare 500 mL of Solution B: In a 500-mL volumetric
protective equipment (PPE) should be used.
flask, dissolve 4.756 g dibasic sodium phosphate
(MW 141.96) in CLRW. Store at 2°C to 5°C. Stable for
3 months.
d. Combine 13-mL solution A, 87-mL solution B, and 13-mL
absolute methanol. Store at 2°C to 5°C. Stable for
3 months.
2. In a 100-mL volumetric flask, add the following chemicals:
a. 10.0 mg hyaluronidase, Type 1-S (EC 3.2.1.35)
b. 40.0 mg dextrose
c. 8.0 mg toluidine blue O (CAS 92-31-9)
3. Fill flask approximately half full using 0.067 mol/L phosphate
buffer. Swirl flask to dissolve chemicals. Bring volume to
calibration mark using buffer.
4. Store at 2°C to 5°C. Stable for 3 months. Filter before use if
necessary.
APPENDIX D Body Fluid Diluents and Pretreatment Solutions 391

SEMEN SOLUTIONS
Semen Diluent for Sperm Counts
Semen Pretreatment Solutions 1. Into a 100-mL volumetric flask, add the following
Two treatment solutions are provided that can be used to substances:
reduce the viscosity of mucoid semen specimens that fail to !60 mL Clinical Laboratory Reagent Water (CLRW)1
liquefy adequately after 60 minutes. The effects of these treat- 5 g sodium bicarbonate
ments on sperm motility, sperm morphology, and biochemical 1 mL 35% (v/v) formalin
tests of seminal plasma are not known. Document use of Optional (to enhance visualization of sperm heads): 25 mg
these solutions, and account for the volume used when Trypan blue or 5 mL saturated gentian violet (>4 mg/mL)4
performing sperm concentration calculations. 2. Swirl to dissolve.
3. Bring volume to 100 mL calibration mark using CLRW.
A. Dilution With Physiologic Solution 4. Store at 4°C. If crystals form, filter before use.
Prepare a 1-to-2 dilution of the semen (one part semen + one
part diluent) using Dulbecco’s phosphate-buffered saline
(pH 7.4). After the dilution is prepared, liquefaction can be
enhanced by repeatedly pipetting the mixture.
REFERENCES
1. Clinical and Laboratory Standards Institute (CLSI): Preparation
Dulbecco’s Phosphate-Buffered Saline (pH 7.4)4 and testing of reagent water in the clinical laboratory: approved
1. Into a 1-L volumetric flask, add the following substances: guideline, ed 4, Wayne, PA, 2006, CLSI. CLSI Document GP40-
Approximately 750 mL Clinical Laboratory Reagent Water A4-AMD.
(CLRW)1 2. Kjeldsberg CR, Knight JA: Laboratory methods. In Body fluids, ed
8.00 g sodium chloride (NaCl) 3, Chicago, 1993, American Society of Clinical Pathology Press.
1.00 g D-glucose 3. Clinical and Laboratory Standards Institute (CLSI): Body fluid
0.20 g potassium chloride (KCl) analysis for cellular composition: approved guideline, Wayne, PA,
0.20 g potassium dihydrogen phosphate (KH2PO4) 2007, CLSI. CLSI Document H56-A.
2.16 g disodium hydrogen phosphate heptahydrate 4. World Health Organization: WHO laboratory manual for the
(Na2HPO4 • 7H2O) examination and processing of human semen, ed 5, Geneva,
0.10 g magnesium chloride hexahydrate (MgCl2•6H2O) Switzerland, 2010, World Health Organization.
2. In a 10-mL volumetric flask, dissolve 0.132 g of calcium
chloride dihydrate (CaCl2•2H2O).
3. To prevent precipitation, add the calcium chloride dihydrate
solution (10 mL) to the 1-L flask slowly and with stirring.
4. Bring to within ! 3 mL of calibration mark using CLRW
and mix.
5. Adjust to pH 7.4 using 1 mol/L sodium hydroxide (NaOH).
6. After pH adjustment, bring volume to 1-L calibration mark
using CLRW.

B. Digestion With Bromelain


The proteolytic enzyme bromelain can be used to enzymati-
cally digest and liquefy the semen specimen. Prepare a
1-to-2 dilution of the semen (i.e., one part semen + one part
bromelain solution) using bromelain solution (see instructions
that follow for making this solution), and mix well. After the
dilution is prepared, incubate at 37°C for 10 minutes. Mix well
before analysis.

Bromelain Solution (10 IU/mL)4


1. Into a 100-mL volumetric flask, add 1000 IU of bromelain
(EC 3.4.22.32).
2. Add approximately 60 mL Dulbecco’s phosphate-buffered
saline (for preparation, see previous instructions).
3. Swirl flask to dissolve; then bring volume to calibration mark
using buffered saline.

You might also like